Aldehydes and Ketones - Wiley

90 downloads 9892 Views 3MB Size Report
2 CHAPTER 20. Aldehydes and Ketones. DO YOU REMEMBER? Before you go on, be sure you understand the following topics. If necessary, review the ...
Aldehydes and Ketones

20 20.1 Introduction to Aldehydes and Ketones

DID YOU EVER WONDER . . .

20.2 Nomenclature

why beta-carotene, which makes carrots orange, is reportedly good for your eyes?

20.3 Preparing Aldehydes and Ketones: A Review

T

his chapter will explore the reactivity of aldehydes and ketones. Specifically, we will see that a wide variety of nucleophiles will react with aldehydes and ketones. Many of these reactions are common in biological pathways, including the role that beta-carotene plays in promoting healthy vision. As we will see several times in this chapter, the reactions of aldehydes and ketones are also cleverly exploited in the design of drugs. The reactions and principles outlined in this chapter are central to the study of organic chemistry and will be used as guiding principles throughout the remaining chapters of this textbook.

klein_c20_001-056v1.4.indd 1

20.4 Introduction to Nucleophilic Addition Reactions 20.5 Oxygen Nucleophiles 20.6 Nitrogen Nucleophiles 20.7 Mechanism Strategies 20.8 Sulfur Nucleophiles 20.9 Hydrogen Nucleophiles 20.10 Carbon Nucleophiles 20.11 Baeyer-Villiger Oxidation of Aldehydes and Ketones 20.12 Synthesis Strategies 20.13 Spectroscopic Analysis of Aldehydes and Ketones

30/04/10 16.46

2

CHAPTER 20

Aldehydes and Ketones

DO YOU REMEMBER? Before you go on, be sure you understand the following topics. If necessary, review the suggested sections to prepare for this chapter: UÊ Àˆ}˜>À`ÊÀi>}i˜ÌÃÊ­-iV̈œ˜Ê£Î°È®ÊÊ

UÊ ,iÌÀœÃޘ̅ïVÊ>˜>ÞÈÃÊ­-iV̈œ˜Ê£Ó°Î®

UÊ "݈`>̈œ˜ÊœvÊ>Vœ…œÃÊ­-iV̈œ˜Ê£Î°£ä® 6ˆÃˆÌÊÜÜܰ܈iÞ«ÕðVœ“Ê̜ÊV…iVŽÊޜÕÀÊ՘`iÀÃÌ>˜`ˆ˜}Ê>˜`ÊvœÀÊÛ>Õ>LiÊ«À>V̈Vi°

20.1 Introduction to Aldehydes and Ketones Aldehydes (RCHO) and ketones (R2CO) are similar in structure in that both classes of compounds possess a C5O bond, called a carbonyl group: Carbonyl Group

O R

O H

R

An aldehyde

R

A ketone

The carbonyl group of an aldehyde is flanked by one carbon atom and one hydrogen atom, while the carbonyl group of a ketone is flanked by two carbon atoms. Aldehydes and ketones are responsible for many flavors and odors that you will readily recognize: O

H3CO

O

O

O H

HO

H

H Vanillin (Vanilla flavor)

Cinnamaldehyde (Cinnamon flavor)

(R )-Carvone (Spearmint flavor)

Benzaldehyde (Almond flavor)

Many important biological compounds also exhibit the carbonyl moiety, including progesterone and testosterone, the female and male sex hormones. O

OH H

H H

H

H

O

H

O Progesterone

Testosterone

Simple aldehydes and ketones are industrially important; for example: O H

O H

Formaldehyde

H3C

CH3

Acetone

Acetone is used as a solvent and is commonly found in nail polish remover, while formaldehyde is used as a preservative in some vaccine formulations. Aldehydes and ketones are also used as building blocks in the syntheses of commercially important compounds, including pharmaceuticals and polymers. Compounds containing a carbonyl group react with a large variety of nucleophiles, affording a wide range of possible products. Due to the versatile reactivity of the carbonyl group, aldehydes and ketones occupy a central role in organic chemistry.

klein_c20_001-056v1.4.indd 2

30/04/10 16.46

20.2

3

Nomenclature

20.2 Nomenclature Nomenclature of Aldehydes Recall that four discrete steps are required to name most classes of organic compounds (as we saw with alkanes, alkenes, alkynes, and alcohols): 1. 2. 3. 4.

Identify and name the parent. Identify and name the substituents. Assign a locant to each substituent. Assemble the substituents alphabetically.

Aldehydes are also named using the same four-step procedure. When applying this procedure for naming aldehydes, the following guidelines should be followed: When naming the parent, the suffix “-al” indicates the presence of an aldehyde group: O H Butane

Butanal

When choosing the parent of an aldehyde, identify the longest chain that includes the carbon atom of the aldehydic group: The parent must include this carbon atom

Parent=Octane

H

O

Parent=Hexanal

When numbering the parent chain of an aldehyde, the aldehydic carbon is assigned number 1, despite the presence of alkyl substituents, U bonds, or hydroxyl groups: Correct

H

1

O

2

3

4

Incorrect 5

6

H

7

OH

7

O

6

5

4

3

2

1

OH

It is not necessary to include the locant in the name, because it is understood that the aldehydic carbon is the number 1 position. As with all compounds, when a chirality center is present, the configuration is indicated at the beginning of the name; for example: O H Cl (R)-2-chloro-3-phenylpropanal

A cyclic compound containing an aldehyde group immediately adjacent to the ring is named as a carbaldehyde: O H

Cyclohexanecarbaldehyde

klein_c20_001-056v1.4.indd 3

30/04/10 16.46

4

CHAPTER 20

Aldehydes and Ketones

The International Union of Pure and Applied Chemistry (IUPAC) nomenclature also recognizes the common names of many simple aldehydes, including the three examples shown below: O H

O

O H

H3C

Formaldehyde

H

H

Acetaldehyde

Benzaldehyde

Nomenclature of Ketones Ketones, like aldehydes, are named using the same four-step procedure. When naming the parent, the suffix “-one” indicates the presence of a ketone group: O

Butane

Butanone

The position of the ketone group is indicated using a locant. The IUPAC rules published in 1979 dictate that this locant be placed immediately before the parent, while the IUPAC recommendations released in 1993 and 2004 allow for the locant to be placed immediately before the suffix “-one”: O 1

2

3

4

5

6

7

3-heptanone or heptan-3-one

Both names above are acceptable IUPAC names. IUPAC nomenclature recognizes the common names of many simple ketones, including the three examples shown below: O O H3C

O CH3

CH3

Acetone

Acetophenone

Benzophenone

Although rarely used, IUPAC rules also allow simple ketones to be named as alkyl alkyl ketones. For example, 3-hexanone can also be called ethyl propyl ketone: O C Ethyl propyl ketone

SKILLBUILDER 20.1 NAMING ALDEHYDES AND KETONES LEARN the skill

*ÀœÛˆ`iÊ>ÊÃÞÃÌi“>̈VÊ­1* ®Ê˜>“iÊvœÀÊ̅iÊvœœÜˆ˜}ÊVœ“«œÕ˜`\

O

SOLUTION STEP 1 `i˜ÌˆvÞÊ>˜`ʘ>“iÊ̅iÊ «>Ài˜Ì°

/…iÊwÀÃÌÊÃÌi«ÊˆÃÊ̜ʈ`i˜ÌˆvÞÊ>˜`ʘ>“iÊ̅iÊ«>Ài˜Ì°Ê …œœÃiÊ̅iʏœ˜}iÃÌÊV…>ˆ˜Ê̅>Ìʈ˜VÕ`iÃÊ̅iÊV>ÀLœ˜ÞÊ}ÀœÕ«]Ê>˜`Ê̅i˜Ê˜Õ“LiÀÊ̅iÊ V…>ˆ˜Ê̜Ê}ˆÛiÊ̅iÊV>ÀLœ˜ÞÊ}ÀœÕ«Ê̅iʏœÜiÃÌʘՓLiÀÊ«œÃÈLi\

8 7 2

4

1 3

9 6

5

O 3-nonanone

klein_c20_001-056v1.4.indd 4

30/04/10 16.46

20.2

5

Nomenclature

iÝÌ]ʈ`i˜ÌˆvÞÊ̅iÊÃÕLÃ̈ÌÕi˜ÌÃÊ>˜`Ê>ÃÈ}˜ÊœV>˜ÌÃ\ STEP 2 `i˜ÌˆvÞÊ>˜`ʘ>“iÊ̅iÊ ÃÕLÃ̈ÌÕi˜Ìð STEP 3 ÃÈ}˜Ê>ʏœV>˜ÌÊ̜Êi>V…Ê ÃÕLÃ̈ÌÕi˜Ì°

STEP 4 ÃÃi“LiÊ̅iÊ ÃÕLÃ̈ÌÕi˜ÌÃÊ >«…>LïV>Þ° STEP 5 ÃÈ}˜Ê̅iÊVœ˜w}ÕÀ>̈œ˜Ê œvÊ>˜ÞÊV…ˆÀ>ˆÌÞÊVi˜ÌiÀð

4,4-dimethyl

8

7 2

4

1 3

9 6

5

6-ethyl

O

ˆ˜>Þ]Ê>ÃÃi“LiÊ̅iÊÃÕLÃ̈ÌÕi˜ÌÃÊ>«…>LïV>Þ\Êȇi̅ޏ‡{]{‡`ˆ“i̅ޏ‡Î‡˜œ˜>˜œ˜i°Ê ivœÀiÊ Vœ˜VÕ`ˆ˜}]ÊÜiʓÕÃÌÊ>Ü>ÞÃÊV…iVŽÊ̜ÊÃiiʈvÊ̅iÀiÊ>ÀiÊ>˜ÞÊV…ˆÀ>ˆÌÞÊVi˜ÌiÀðÊ/…ˆÃÊVœ“«œÕ˜`Ê `œiÃÊ i݅ˆLˆÌÊ œ˜iÊ V…ˆÀ>ˆÌÞÊ Vi˜ÌiÀ°Ê 1Ș}Ê Ì…iÊ ÃŽˆÃÊ vÀœ“Ê -iV̈œ˜Ê x°Î]Ê Ì…iÊ RÊ Vœ˜w}ÕÀ>̈œ˜Ê ˆÃÊ >ÃÈ}˜i`Ê̜Ê̅ˆÃÊV…ˆÀ>ˆÌÞÊVi˜ÌiÀ\

R O

/…iÀivœÀi]Ê̅iÊVœ“«iÌiʘ>“iʈÃÊ(R)-6-ethyl-4,4-dimethyl-3-nonanone.

PRACTICE the skill 20.1 ÃÈ}˜Ê>ÊÃÞÃÌi“>̈VÊ­1* ®Ê˜>“iÊ̜Êi>V…ÊœvÊ̅iÊvœœÜˆ˜}ÊVœ“«œÕ˜`Ã\ O

O H

(a)

Br Br

O

(b)

(c) O

O

H H

(d)

APPLY the skill

20.2

(e) À>ÜÊ̅iÊÃÌÀÕVÌÕÀiʜvÊi>V…ÊœvÊ̅iÊvœœÜˆ˜}ÊVœ“«œÕ˜`Ã\

­>® ­S®‡Î]·`ˆLÀœ“œ‡{‡i̅ޏVÞVœ…iÝ>˜œ˜iÊ Ê ­L®Ê Ó]{‡`ˆ“i̅ޏ‡Î‡«i˜Ì>˜œ˜i ­V® ­R®‡Î‡LÀœ“œLÕÌ>˜> 20.3 *ÀœÛˆ`iÊ>ÊÃÞÃÌi“>̈VÊ­1* ®Ê˜>“iÊvœÀÊ̅iÊVœ“«œÕ˜`ÊLiœÜ°Ê iÊV>ÀivՏ\Ê/…ˆÃÊVœ“«œÕ˜`ʅ>ÃÊÌܜÊV…ˆÀ>ˆÌÞÊVi˜ÌiÀÃÊ­V>˜ÊޜÕÊw˜`Ê̅i“¶®° O

20.4

œ“«œÕ˜`ÃÊ܈̅ÊÌܜÊV>ÀLœ˜ÞÊ“œˆïiÃÊ>Àiʘ>“i`Ê>ÃÊ>Ž>˜iÊ`ˆœ˜iÃÆÊvœÀÊiÝ>“«i\ O

O

2,3-butanedione

/…iÊVœ“«œÕ˜`Ê>LœÛiʈÃÊ>˜Ê>À̈wVˆ>Êy>ۜÀÊ>``i`Ê̜ʓˆVÀœÜ>ÛiÊ«œ«VœÀ˜Ê>˜`ʓœÛˆi‡Ê ̅i>ÌiÀÊ«œ«VœÀ˜Ê̜ÊȓՏ>ÌiÊ̅iÊLÕÌÌiÀÊy>ۜÀ°Ê˜ÌiÀiÃ̈˜}Þ]Ê̅ˆÃÊÛiÀÞÊÃ>“iÊVœ“«œÕ˜`ʈÃÊ >ÃœÊŽ˜œÜ˜Ê̜ÊVœ˜ÌÀˆLÕÌiÊ̜ÊLœ`Þʜ`œÀ°Ê >“iÊ̅iÊvœœÜˆ˜}ÊVœ“«œÕ˜`Ã\ O

(a)

O

O

O

(b)

O

(c)

O

O

need more PRACTICE? Try Problems 20.44–20.49

klein_c20_001-056v1.4.indd 5

30/04/10 16.46

6

CHAPTER 20

Aldehydes and Ketones

20.3 Preparing Aldehydes and Ketones: A Review In previous chapters, we have studied a variety of methods for preparing aldehydes and ketones, which are summarized in Tables 20.1 and 20.2, respectively. TABLE 20.1 A SUMMARY OF ALDEHYDE PREPARATION METHODS COVERED IN PREVIOUS CHAPTERS

TABLE 20.2 A SUMMARY OF KETONE PREPARATION METHODS COVERED IN PREVIOUS CHAPTERS

REACTION

REACTION

SECTION

"݈`>̈œ˜ÊœvÊ*Àˆ“>ÀÞʏVœ…œÃÊ £Î°£äÊ Ê OH O Ê PCC Ê CH2Cl2 R H ÊR 7…i˜ÊÌÀi>Ìi`Ê܈̅Ê>ÊÃÌÀœ˜}ʜ݈`ˆâˆ˜}Ê>}i˜Ì]Ê«Àˆ“>ÀÞÊ>Vœ…œÃÊ >Àiʜ݈`ˆâi`Ê̜ÊV>ÀLœÝޏˆVÊ>Vˆ`ðÊœÀ“>̈œ˜ÊœvÊ>˜Ê>`i…Þ`iÊ ÀiµÕˆÀiÃÊ>˜ÊœÝˆ`ˆâˆ˜}Ê>}i˜Ì]ÊÃÕV…Ê>ÃÊ*

]Ê̅>ÌÊ܈Ê˜œÌÊvÕÀ̅iÀÊ œÝˆ`ˆâiÊ̅iÊÀiÃՏ̈˜}Ê>`i…Þ`i°

SECTION

"݈`>̈œ˜ÊœvÊ-iVœ˜`>ÀÞʏVœ…œÃÊ £Î°£äÊ Ê OH O Ê Na2Cr2O7 Ê H2SO4, H2O R R R R Ê ÊÊÛ>ÀˆiÌÞʜvÊÃÌÀœ˜}ʜÀʓˆ`ʜ݈`ˆâˆ˜}Ê>}i˜ÌÃÊV>˜ÊLiÊÕÃi`ÊÌœÊ œÝˆ`ˆâiÊÃiVœ˜`>ÀÞÊ>Vœ…œÃ°Ê/…iÊÀiÃՏ̈˜}ʎi̜˜iÊ`œiÃʘœÌÊ Õ˜`iÀ}œÊvÕÀ̅iÀʜ݈`>̈œ˜° "✘œÞÈÃʜvʏŽi˜iÃÊ ™°£™Ê ÊR R R R 1) O3 Ê O O Ê 2) DMS R R R R Ê Ê/iÌÀ>ÃÕLÃ̈ÌÕÌi`Ê>Ži˜iÃÊ>ÀiÊVi>Ûi`Ê̜ÊvœÀ“ÊŽi̜˜ið

"✘œÞÈÃʜvʏŽi˜iÃÊ ™°£™Ê Ê H H H ÊH 1) O3 Ê O O 2) DMS ÊR R R R Ê Ê"✘œÞÈÃÊ܈ÊVi>ÛiÊ>Ê 5 Ê`œÕLiÊLœ˜`°ÊvÊiˆÌ…iÀÊV>ÀLœ˜Ê >̜“ÊLi>ÀÃÊ>ʅÞ`Àœ}i˜Ê>̜“]Ê>˜Ê>`i…Þ`iÊ܈ÊLiÊvœÀ“i`°

Vˆ`‡ >Ì>Þâi`ÊÞ`À>̈œ˜ÊœvÊ/iÀ“ˆ˜>ÊŽÞ˜iÃÊ £ä°nÊ Ê O Ê H2SO4, H2O Ê HgSO4 CH3 R R Ê Ê/…ˆÃÊ«ÀœVi`ÕÀiÊÀiÃՏÌÃʈ˜Ê>Ê>ÀŽœÛ˜ˆŽœÛÊ>``ˆÌˆœ˜ÊœvÊÜ>ÌiÀÊ >VÀœÃÃÊ̅iÊUÊLœ˜`]ÊvœœÜi`ÊLÞÊÌ>Õ̜“iÀˆâ>̈œ˜Ê̜ÊvœÀ“Ê>Ê “i̅ޏʎi̜˜i°

Þ`ÀœLœÀ>̈œ˜‡"݈`>̈œ˜ÊœvÊ/iÀ“ˆ˜>ÊŽÞ˜iÃÊ £ä°nÊ Ê H 1) R2B H Ê R 2) H2O2, NaOH ÊR O Ê ÊÞ`ÀœLœÀ>̈œ˜‡œÝˆ`>̈œ˜ÊÀiÃՏÌÃʈ˜Ê>˜Ê>˜Ìˆ‡>ÀŽœÛ˜ˆŽœÛÊ>``ˆÌˆœ˜ÊœvÊÜ>ÌiÀÊ>VÀœÃÃÊ>ÊUÊLœ˜`]ÊvœœÜi`ÊLÞÊÌ>Õ̜“iÀˆâ>̈œ˜ÊœvÊ Ì…iÊÀiÃՏ̈˜}Êi˜œÊ̜ÊvœÀ“Ê>˜Ê>`i…Þ`i°Ê

Àˆi`i‡ À>vÌÃÊVޏ>̈œ˜Ê £™°ÈÊ Ê O O Ê Ê R Cl R Ê AlCl Ê 3 Ê Àœ“>̈VÊÀˆ˜}ÃÊ̅>ÌÊ>ÀiʘœÌÊ̜œÊÃÌÀœ˜}ÞÊ`i>V̈Û>Ìi`Ê܈ÊÀi>VÌÊ ÜˆÌ…Ê>˜Ê>Vˆ`ʅ>ˆ`iʈ˜Ê̅iÊ«ÀiÃi˜ViʜvÊ>Êi܈ÃÊ>Vˆ`Ê̜ʫÀœ`ÕViÊ >˜Ê>Àޏʎi̜˜i°

CONCEPTUAL CHECKPOINT 20.5 `i˜ÌˆvÞÊ̅iÊÀi>}i˜ÌÃʘiViÃÃ>ÀÞÊ̜Ê>V…ˆiÛiÊi>V…ÊœvÊ̅iÊvœœÜˆ˜}ÊÌÀ>˜ÃvœÀ“>̈œ˜Ã\

OH

O

O

OH O

H

(a)

(b) H

O

O

(d)

klein_c20_001-056v1.4.indd 6

(c)

O

O H

(e)

(f)

30/04/10 16.46

20.4

Introduction to Nucleophilic Addition Reactions

7

20.4 Introduction to Nucleophilic Addition Reactions The electrophilicity of a carbonyl group derives from resonance effects as well as inductive effects: Resonance

Induction -

O

d-

O

O

+



One of the resonance structures exhibits a positive charge on the carbon atom, indicating that the carbon atom is deficient in electron density (I+). Inductive effects also render the carbon atom deficient in electron density. As a result, this carbon atom is particularly electrophilic and is susceptible to attack by a nucleophile. Molecular orbital calculations suggest that nucleophilic attack occurs at an angle of approximately 107° to the plane of the carbonyl group, and in the process, the hybridization state of the carbon atom changes (Figure 20.1). 107° angle –

Nuc

Nuc

R

¡

O

FIGURE 20.1 7…i˜Ê>ÊV>ÀLœ˜ÞÊ}ÀœÕ«ÊˆÃÊ >ÌÌ>VŽi`ÊLÞÊ>ʘÕViœ«…ˆi]Ê Ì…iÊV>ÀLœ˜Ê>̜“Ê՘`iÀ}œiÃÊ >ÊV…>˜}iʈ˜Ê…ÞLÀˆ`ˆâ>̈œ˜Ê>˜`Ê }iœ“iÌÀÞ°

– O

R

R

R

sp 2 (Trigonal planar)

sp 3 (Tetrahedral )

The carbon atom is originally sp2 hybridized with a trigonal planar geometry. After the attack, the carbon atom is sp3 hybridized with a tetrahedral geometry. In recognition of this geometric change, the resulting alkoxide ion is often called a tetrahedral intermediate. This term appears many times throughout the remainder of this chapter. In general, aldehydes are more reactive than ketones toward nucleophilic attack. This observation can be explained in terms of both steric and electronic effects: 1.

Steric effects. A ketone has two alkyl groups (one on either side of the carbonyl) that contribute to steric hindrance in the transition state of a nucleophilic attack. In contrast, an aldehyde has only one alkyl group, so the transition state is less crowded and lower in energy.

2.

Electronic effects. Recall that alkyl groups are electron donating. A ketone has two electrondonating alkyl groups that can stabilize the I+ on the carbon atom of the carbonyl group. In contrast, aldehydes have only one electron-donating group: O

O

R d± R

R d± H

A ketone has two electron-donating alkyl groups that stabilize the partial positive charge

An aldehyde has only one electron-donating alkyl group that stabilizes the partial positive charge

The I+ charge of an aldehyde is less stabilized than a ketone. As a result, aldehydes are more electrophilic than ketones and therefore more reactive. Aldehydes and ketones react with a wide variety of nucleophiles. As we will see in the coming sections of this chapter, some nucleophiles require basic conditions, while others require acidic conditions. For example, recall from Chapter 13 that Grignard reagents are very strong nucleophiles that will attack aldehydes and ketones to produce alcohols: OH

O 1) MeMgBr

R

klein_c20_001-056v1.4.indd 7

R

2) H2O

R

R

Me

30/04/10 16.46

8

CHAPTER 20

Aldehydes and Ketones

The Grignard reagent itself provides for strongly basic conditions, because Grignard reagents are both strong nucleophiles and strong bases. This reaction cannot be achieved under acidic conditions, because, as explained in Section 13.6, Grignard reagents are destroyed in the presence of an acid. The Grignard reaction above follows a general mechanism for the reaction between a nucleophile and a carbonyl group under basic conditions (Mechanism 20.1). This general mechanism has two steps: (1) nucleophilic attack followed by (2) proton transfer.

MECHANISM 20.1 NUCLEOPHILIC ADDITION UNDER BASIC CONDITIONS Nucleophilic attack O

Proton transfer -

O

O

-

Nuc

H

OH

H

Nuc The carbonyl group is attacked by a nucleophile, forming a tetrahedral intermediate

Nuc

The tetrahedral intermediate is protonated upon treatment with a mild proton source

Aldehydes and ketones also react with a wide variety of other nucleophiles under acidic conditions. In acidic conditions, the same two mechanistic steps are observed, but in reverse order—that is, the carbonyl group is first protonated and then undergoes a nucleophilic attack (Mechanism 20.2).

MECHANISM 20.2 NUCLEOPHILIC ADDITION UNDER ACIDIC CONDITIONS Proton transfer O

Nucleophilic attack +

H

-

O

H A

OH

Nuc

Nuc The carbonyl group is first protonated, rendering it even more electrophilic

The protonated carbonyl group is then attacked by a nucleophile

In acidic conditions, the first step plays an important role. Specifically, protonating the carbonyl group generates a very powerful electrophile: +

O

H

H O

O

H¬A

·

+

Very powerful electrophile

It is true that the carbonyl group is already a fairly strong electrophile; however, a protonated carbonyl group bears a full positive charge, rendering the carbon atom even more electrophilic. This is especially important when weak nucleophiles, such as H2O or ROH, are employed, as we will see in the upcoming sections. When a nucleophile attacks a carbonyl group under either acidic or basic conditions, the position of equilibrium is highly dependent on the ability of the nucleophile to function as a leaving group. A Grignard reagent is a very strong nucleophile, but it does not function as a leaving group (a carbanion is too unstable to leave). As a result, the equilibrium so greatly favors products that the reaction effectively occurs in only one direction. With a sufficient amount of nucleophile present, the ketone is not observed in the product mixture. In contrast, halides are

klein_c20_001-056v1.4.indd 8

30/04/10 16.46

20.5

9

Oxygen Nucleophiles

good nucleophiles, but they are also good leaving groups. Therefore, when a halide functions as the nucleophile, the equilibrium actually favors the starting ketone: O

HO Cl

R

R

+ HCl

R

R

Once equilibrium has been achieved, the mixture consists primarily of the ketone, and only small quantities of the addition product. In this chapter, we will explore a wide variety of nucleophiles, which will be classified according to the nature of the attacking atom. Specifically, we will see nucleophiles based on oxygen, sulfur, nitrogen, hydrogen, and carbon (Figure 20.2). Oxygen Nucleophiles

Sulfur Nucleophiles

O

S

H

H

H

Nitrogen Nucleophiles

Hydrogen Nucleophiles

Carbon Nucleophiles

H

H

RMgBr

-

H Al

N

H R

H

H -

H

C

N

O R FIGURE 20.2 6>ÀˆœÕÃʘÕViœ«…ˆiÃÊ̅>ÌÊV>˜Ê >ÌÌ>VŽÊ>ÊV>ÀLœ˜ÞÊ}ÀœÕ«°

H

H

S

S

H

H

H

N H O

O

R

H

H R

Ph + H Ph P C Ph H

-

B H H

The remainder of the chapter will be a methodical survey of the reactions that occur between the reagents in Figure 20.2 and ketones and aldehydes. We will begin our survey with oxygen nucleophiles.

CONCEPTUAL CHECKPOINT O

20.6 À>ÜÊ>ʓiV…>˜ˆÃ“ÊvœÀÊi>V…ÊœvÊ̅iÊ Ài>V̈œ˜ÃÊLiœÜ\

HO

O

HO Cl

1) EtMgBr

+ HCl

2) H2O

(a)

(b)

20.5 Oxygen Nucleophiles Hydrate Formation When an aldehyde or ketone is treated with water, the carbonyl group can be converted into a hydrate: O

HO OH

+

H2O Hydrate

The position of equilibrium generally favors the carbonyl group rather than the hydrate, except in the case of very simple aldehydes, such as formaldehyde: O H3C

HO OH CH3

+

H2O

+

H2O

H3C

CH3

99.9%

O H

HO OH H

H

H

> 99.9%

klein_c20_001-056v1.4.indd 9

30/04/10 16.46

10

CHAPTER 20

Aldehydes and Ketones

The rate of reaction is relatively slow under neutral conditions but is readily enhanced in the presence of either acid or base. That is, the reaction can be either acid catalyzed or base catalyzed, allowing the equilibrium to be achieved much more rapidly. Consider the base-catalyzed hydration of formaldehyde (Mechanism 20.3).

MECHANISM 20.3 BASE-CATALYZED HYDRATION Nucleophilic attack -

-

O H

Proton transfer O

O

OH

H

H

H

H

OH

H

OH

The carbonyl group is attacked by hydroxide, forming a tetrahedral intermediate

H

H

OH

The tetrahedral intermediate is protonated by water to form the hydrate

In the first step, a hydroxide ion (rather than water) functions as a nucleophile. Then, in the second step, the tetrahedral intermediate is protonated with water, regenerating a hydroxide ion. In this way, hydroxide serves as a catalyst for the addition of water across the carbonyl group. Now consider the acid-catalyzed hydration of formaldehyde (Mechanism 20.4).

MECHANISM 20.4 ACID-CATALYZED HYDRATION Proton transfer

Nucleophilic attack

H H

O

+

O+

H

O

H

H

O

O

OH

H

H

H

OH

H

H

H

H

Proton transfer

H

H

H O+

H

H

OH

H The carbonyl group is protonated, rendering it more electrophilic

The protonated carbonyl group is attacked by water, forming a tetrahedral intermediate

The tetrahedral intermediate is deprotonated by water to form the hydrate

Under acid-catalyzed conditions, the carbonyl group is first protonated, generating a positively charged intermediate that is extremely electrophilic (it bears a full positive charge). This intermediate is then attacked by water to form a tetrahedral intermediate, which is deprotonated to give the product.

CONCEPTUAL CHECKPOINT 20.7 œÀʓœÃÌʎi̜˜iÃ]ʅÞ`À>ÌiÊvœÀ“>̈œ˜ÊˆÃÊ՘v>ۜÀ>Li]ÊLiV>ÕÃiÊ Ì…iÊiµÕˆˆLÀˆÕ“Êv>ۜÀÃÊ̅iʎi̜˜iÊÀ>̅iÀÊ̅>˜Ê̅iʅÞ`À>Ìi°ÊœÜiÛiÀ]Ê Ì…iÊiµÕˆˆLÀˆÕ“ÊvœÀʅÞ`À>̈œ˜ÊœvʅiÝ>y՜Àœ>Vi̜˜iÊv>ۜÀÃÊvœÀ“>̈œ˜Ê œvÊ̅iʅÞ`À>Ìi\Ê*ÀœÛˆ`iÊ>Ê«>ÕÈLiÊiÝ«>˜>̈œ˜ÊvœÀÊ̅ˆÃʜLÃiÀÛ>̈œ˜°

klein_c20_001-056v1.4.indd 10

O F3C

HO OH CF3

+

H2O F3 C

CF3

> 99.99%

30/04/10 16.46

20.5

11

Oxygen Nucleophiles

Acetal Formation The previous section discussed a reaction that can occur when water attacks an aldehyde or ketone. This section will explore a similar reaction, in which an alcohol attacks an aldehyde or ketone: O

+

[H+]

2 ROH

BY THE WAY

RO OR

+

H2O

Acetal

7…i˜Ê̅iÊÃÌ>À̈˜}ÊVœ“«œÕ˜`Ê ˆÃÊ>ʎi̜˜i]Ê̅iÊ«Àœ`ÕVÌÊV>˜Ê >ÃœÊLiÊV>i`Ê>ʺŽiÌ>°»Ê AcetalʈÃÊ>ʓœÀiÊ}i˜iÀ>Ê ÌiÀ“]Ê>˜`ʈÌÊ܈ÊLiÊÕÃi`Ê iÝVÕÈÛiÞÊvœÀÊ̅iÊÀi“>ˆ˜`iÀÊ œvÊ̅ˆÃÊ`ˆÃVÕÃȜ˜°

In acidic conditions, an aldehyde or ketone will react with two molecules of alcohol to form an acetal. The brackets surrounding the H+ indicate that the acid is a catalyst. Common acids used for this purpose include para-toluenesulfonic acid (TsOH) and sulfuric acid (H2SO4): O S

O OH

HO

S

O

OH

O

p-toluenesulfonic acid (TsOH)

Sulfuric acid

As mentioned earlier, the acid catalyst serves an important role in this reaction. Specifically, in the presence of an acid, the carbonyl group is protonated, rendering the carbon atom even more electrophilic. This is necessary because the nucleophile (an alcohol) is weak; it reacts with the carbonyl group more rapidly if the carbonyl group is first protonated. A mechanism for acetal formation is shown in Mechanism 20.5. This mechanism has many steps, and it is best to divide it conceptually into two parts: (1) The first three steps produce an intermediate called a hemiacetal and (2) the last four steps convert the hemiacetal into an acetal:

MECHANISM 20.5 ACETAL FORMATION Proton transfer

O

+

Nucleophilic attack

H

+

O

H A

Proton transfer

H O

OH

A

R

OH

H O+ The carbonyl group is protonated, rendering it more electrophilic

The alcohol attacks the protonated carbonyl to generate a tetrahedral intermediate

R

OR

The tetrahedral intermediate is deprotonated to form a hemiacetal

Hemiacetal +

H A

H + H O

Proton transfer The OH group is protonated, thereby converting it into an excellent leaving group

OR Loss of a leaving group Proton transfer

OR

A

Nucleophilic attack

H + R O

OR Acetal

klein_c20_001-056v1.4.indd 11

H

O R

–H2O

+

R

Water leaves to regenerate the C“ O double bond

O

OR The intermediate is deprotonated, generating an acetal

The second molecule of the alcohol attacks the C“ O double bond to generate another tetrahedral intermediate

30/04/10 16.46

12

CHAPTER 20

Aldehydes and Ketones

Let’s begin our analysis of this mechanism by focusing on the first part: formation of the hemiacetal, which involves the three steps in Figure 20.3. FIGURE 20.3 /…iÊÃiµÕi˜ViʜvÊÃÌi«Ãʈ˜ÛœÛi`Ê ˆ˜ÊvœÀ“>̈œ˜ÊœvÊ>ʅi“ˆ>ViÌ>°

Proton transfer

Nucleophilic attack

Proton transfer

Notice that the sequence of steps begins and ends with a proton transfer. This will be a recurring pattern in this chapter. Let’s focus on the details of these three steps: 1.

The carbonyl is protonated in the presence of an acid. The identity of the acid, HA+, is most likely a protonated alcohol, which received its extra proton from the acid catalyst:

H

+

A

H

H O+ R

2.

The protonated carbonyl is a very powerful electrophile and is attacked by a molecule of alcohol (ROH) to form a tetrahedral intermediate that bears a positive charge.

3.

The tetrahedral intermediate is deprotonated by a weak base (A), which is likely to be a molecule of alcohol present in solution.

Notice that the acid is not consumed in this process. A proton is used in step 1 and then returned in step 3, confirming the catalytic nature of the proton in the reaction. It is important to remember the specific order of these three steps, as we will soon encounter many other reactions that begin with the same three steps. These three steps are typical of reactions involving acid-catalyzed nucleophilic attack. Now let’s focus on the second part of the mechanism, conversion of the hemiacetal into an acetal, which is accomplished with the four steps in Figure 20.4. FIGURE 20.4 /…iÊÃiµÕi˜ViʜvÊÃÌi«ÃÊ̅>ÌÊ Vœ˜ÛiÀÌÊ>ʅi“ˆ>ViÌ>Êˆ˜ÌœÊ>˜Ê >ViÌ>°

Proton transfer

Loss of a leaving group

Nucleophilic attack

Proton transfer

Notice, once again, that the sequence of steps begins and ends with a proton transfer. A proton is used in the first step and then returned in the last step, but this time there are two middle steps rather than just one. When drawing the mechanism of acetal formation, make sure to draw these two steps separately. Combining these two steps is incorrect and represents one of the most common student errors when drawing this mechanism: Loss of a leaving group

+

OH2 OR

+ ROH

An SN 2 process cannot occur at this substrate

¡

Nucleophilic attack

These two steps cannot occur simultaneously, because that would represent an SN2 process occurring at a sterically hindered substrate. Such a process is disfavored and does not occur at an appreciable rate. Instead, the leaving group leaves first to form a resonance-stabilized intermediate, which is then attacked by the nucleophile in a separate step. The equilibrium arrows in the full mechanism of acetal formation indicate that the process is governed by an equilibrium. For many simple aldehydes, the equilibrium favors formation of the acetal, so aldehydes are readily converted into acetals by treatment with two equivalents of alcohol in acidic conditions: O H

H

+ 2 EtOH

[H+]

EtO

OEt

H

H

+ H2O

Products are favored at equilibrium

klein_c20_001-056v1.4.indd 12

30/04/10 16.46

20.5

13

Oxygen Nucleophiles

However, for most ketones, the equilibrium favors reactants rather than products:

BY THE WAY /…ˆÃÊÌiV…˜ˆµÕiÊiÝ«œˆÌÃÊiÊ

…@ÌiˆiÀ½ÃÊ«Àˆ˜Vˆ«i]Ê܅ˆV…Ê Ü>ÃÊVœÛiÀi`ʈ˜ÊޜÕÀÊ}i˜iÀ>Ê V…i“ˆÃÌÀÞÊVœÕÀÃi°ÊVVœÀ`ˆ˜}Ê ÌœÊiÊ …@ÌiˆiÀ½ÃÊ«Àˆ˜Vˆ«i]Ê ˆvÊ>ÊÃÞÃÌi“Ê>ÌÊiµÕˆˆLÀˆÕ“ʈÃÊ Õ«ÃiÌÊLÞÊܓiÊ`ˆÃÌÕÀL>˜Vi]Ê Ì…iÊÃÞÃÌi“Ê܈ÊV…>˜}iʈ˜Ê>Ê Ü>ÞÊ̅>ÌÊÀiÃ̜ÀiÃÊiµÕˆˆLÀˆÕ“°

O H 3C

EtO

+

[H ]

+ 2 EtOH

CH3

OEt

H 3C

CH3

+ H2O

Reactants are favored at equilibrium

In such cases, formation of the acetal can be accomplished by removing one of the products (water) via a special distillation technique. By removing water as it is formed, the reaction can be forced to completion. Notice that acetal formation requires two equivalents of the alcohol. That is, two molecules of ROH are required for every molecule of ketone. Alternatively, a compound containing two OH groups can be used, forming a cyclic acetal. This reaction proceeds via the regular seven-step mechanism for acetal formation: three steps for formation of the hemiacetal followed by four steps for formation of the cyclic acetal: OH OH

OH

O

O

3 steps

OH

O

4 steps

Hemiacetal

O

+

H2O

Cyclic acetal

The seven-step mechanism for acetal formation is very similar to other mechanisms that we will explore. It is therefore critical to master these seven steps. To help you draw the mechanism properly, remember to divide the entire mechanism into two parts, where each part begins and ends with proton transfers. Let’s get some practice.

SKILLBUILDER 20.2

DRAWING THE MECHANISM OF ACETAL FORMATION O

LEARN the skill

EtO

À>ÜÊ >Ê «>ÕÈLiÊ “iV…>˜ˆÃ“Ê vœÀÊ Ì…iÊ vœœÜˆ˜}Ê ÌÀ>˜ÃvœÀ“>̈œ˜\

OEt

[H2SO4] excess EtOH –H2O

SOLUTION

STEP 1 À>ÜÊ̅iÊ̅ÀiiÊÃÌi«ÃÊ ˜iViÃÃ>ÀÞÊvœÀʅi“ˆ>ViÌ>Ê vœÀ“>̈œ˜°

/…iÊ Ài>V̈œ˜Ê >LœÛiÊ ˆÃÊ >˜Ê iÝ>“«iÊ œvÊ >Vˆ`‡V>Ì>Þâi`Ê >ViÌ>Ê vœÀ“>̈œ˜]Ê ˆ˜Ê ܅ˆV…Ê ̅iÊ «Àœ`ÕVÌÊ ˆÃÊ v>ۜÀi`Ê LÞÊ Ì…iÊ Ài“œÛ>Ê œvÊ Ü>ÌiÀ°Ê /…iÊ “iV…>˜ˆÃ“Ê V>˜Ê LiÊ `ˆÛˆ`i`Ê ˆ˜ÌœÊ ÌÜœÊ «>ÀÌÃ\ÊÊ ­£®ÊvœÀ“>̈œ˜ÊœvÊ̅iʅi“ˆ>ViÌ>Ê>˜`Ê­Ó®ÊvœÀ“>̈œ˜ÊœvÊ̅iÊ>ViÌ>°ÊœÀ“>̈œ˜ÊœvÊ̅iʅi“ˆ>ViÌ>Ê ˆ˜ÛœÛiÃÊ̅ÀiiʓiV…>˜ˆÃ̈VÊÃÌi«Ã\ Proton transfer

Nucleophilic attack

Proton transfer

7…i˜Ê`À>܈˜}Ê̅iÃiÊ̅ÀiiÊÃÌi«Ã]ʓ>ŽiÊÃÕÀiÊ̜ÊvœVÕÃʜ˜Ê«Àœ«iÀÊ>ÀÀœÜÊ«>Vi“i˜ÌÊ­>ÃÊ`iÃVÀˆLi`Ê ˆ˜Ê …>«ÌiÀÊÈ®]Ê>˜`ʓ>ŽiÊÃÕÀiÊ̜ʫ>ViÊ>Ê«œÃˆÌˆÛiÊV…>À}iÃʈ˜Ê̅iˆÀÊ>««Àœ«Àˆ>ÌiʏœV>̈œ˜Ã°Ê œÌˆViÊ Ì…>ÌÊiÛiÀÞÊÃÌi«ÊÀiµÕˆÀiÃÊÌܜÊVÕÀÛi`Ê>ÀÀœÜð The tail of this arrow should be placed on a lone pair

Don’t forget the positive charges +

O

H H O

+

O

Et

Et

Don’t forget the second curved arrow that shows release of the proton

klein_c20_001-056v1.4.indd 13

H

H O

H

HO

+

O

Et

O

H

HO

OEt

Et

Hemiacetal

30/04/10 16.46

14

CHAPTER 20

Aldehydes and Ketones

œÜʏi̽ÃÊvœVÕÃʜ˜Ê̅iʏ>ÃÌÊvœÕÀÊÃÌi«ÃʜvÊ̅iʓiV…>˜ˆÃ“]ʈ˜Ê܅ˆV…Ê̅iʅi“ˆ>ViÌ>ÊˆÃÊVœ˜ÛiÀÌi`ʈ˜ÌœÊ>˜Ê>ViÌ>\

STEP 2 À>ÜÊ̅iÊvœÕÀÊÃÌi«ÃÊ ˜iViÃÃ>ÀÞÊ̜ÊVœ˜ÛiÀÌÊ Ì…iʅi“ˆ>ViÌ>Êˆ˜ÌœÊ>˜Ê >ViÌ>°

Proton transfer

Loss of a leaving group

Proton transfer

Nucleophilic attack

"˜ViÊ>}>ˆ˜]Ê̅ˆÃÊÃiµÕi˜ViʜvÊÃÌi«ÃÊLi}ˆ˜ÃÊ܈̅Ê>Ê«ÀœÌœ˜ÊÌÀ>˜ÃviÀÊ>˜`Êi˜`ÃÊ܈̅Ê>Ê«ÀœÌœ˜Ê ÌÀ>˜ÃviÀ°Ê7…i˜Ê`À>܈˜}Ê̅iÃiÊvœÕÀÊÃÌi«Ã]ʓ>ŽiÊÃÕÀiÊ̜Ê`À>ÜÊ̅iʓˆ``iÊÌܜÊÃÌi«ÃÊÃi«>À>ÌiÞ]Ê >ÃÊ`ˆÃVÕÃÃi`Êi>ÀˆiÀ°Ê˜Ê>``ˆÌˆœ˜]ʓ>ŽiÊÃÕÀiÊ̜ÊvœVÕÃʜ˜Ê«Àœ«iÀÊ>ÀÀœÜÊ«>Vi“i˜Ì]Ê>˜`ʓ>ŽiÊ ÃÕÀiÊ̜ʫ>ViÊ>Ê«œÃˆÌˆÛiÊV…>À}iÃʈ˜Ê̅iˆÀÊ>««Àœ«Àˆ>ÌiʏœV>̈œ˜Ã\ The tail of this arrow should be placed on a lone pair

H

+

HO

OEt

H H O+ Et

Don’t forget the positive charges +

OEt

H O

H

Et O O

–H2O

H

EtO

+

O

Et

Et

O

EtO

H

OEt

Et

Acetal

Don’t forget the second curved arrow that shows release of the proton

PRACTICE the skill 20.8 À>ÜÊ>Ê«>ÕÈLiʓiV…>˜ˆÃ“ÊvœÀÊi>V…ÊœvÊ̅iÊvœœÜˆ˜}ÊÌÀ>˜ÃvœÀ“>̈œ˜Ã\ O

MeO

OMe

O

[H2SO4] excess MeOH

OEt [TsOH] excess EtOH

–H2O

(b)

(a) EtO

O

O

OEt

MeO

OMe

[TsOH] excess MeOH

[H2SO4] excess EtOH

–H2O

–H2O

(c)

APPLY the skill

OEt

–H2O

(d)

20.9

À>ÜÊ>Ê«>ÕÈLiʓiV…>˜ˆÃ“ÊvœÀÊi>V…ÊœvÊ̅iÊvœœÜˆ˜}ÊÀi>V̈œ˜Ã\

HO

OH

O

[H2SO4]

O O

–H2O

(a)

O

HO OH [H2SO4]

–H2O

O

O

(b) 20.10

*Ài`ˆVÌÊ̅iÊ«Àœ`ÕVÌʜvÊi>V…ÊœvÊ̅iÊvœœÜˆ˜}ÊÀi>V̈œ˜Ã\ O

O [H2SO4] excess MeOH –H2O

?

(a)

HO

OH

[H2SO4] –H2O

?

(b)

need more PRACTICE? Try Problems 20.57, 20.62, 20.67

klein_c20_001-056v1.4.indd 14

30/04/10 16.46

20.5

Acetals as Protecting Groups

HO

Acetal formation is a reversible process that can be controlled by carefully choosing reagents and conditions:

15

Oxygen Nucleophiles

O

OH +

[H ] –H2O

O

O

As mentioned in the previous section, acetal forH2O mation is favored by removal of water. To con+ [H ] vert an acetal back into the corresponding aldehyde or ketone, it is simply treated with water in the presence of an acid catalyst. In this way, acetals can be used to protect ketones or aldehydes. For example, consider how the following transformation might be accomplished: O

O

O OR

OH

?

This transformation involves reduction of an ester to form an alcohol. Recall that lithium aluminum hydride (LAH) can be used to accomplish this type of reaction. However, under these conditions, the ketone moiety will also be reduced. The problem above requires reduction of the ester moiety without also reducing the ketone moiety. To accomplish this, a protecting group can be used. The first step is to convert the ketone into an acetal: O

O

O OR

HO

O

O OR

OH [H+] –H2O

Notice that the ketone moiety is converted into an acetal, but the ester moiety is not. The resulting acetal group is stable under strongly basic conditions and will not react with LAH. This makes it possible to reduce only the ester, after which the acetal can be removed to regenerate the ketone: The three steps are summarized below: O

O

O OR

1) [H+], HO

OH

OH , –H2O

2) LAH 3) H3O+

CONCEPTUAL CHECKPOINT 20.11 *Àœ«œÃiÊ>˜ÊivwVˆi˜ÌÊÃޘ̅iÈÃÊvœÀÊi>V…ÊœvÊ̅iÊvœœÜˆ˜}ÊÌÀ>˜ÃvœÀ“>̈œ˜Ã\ O

20.12

O

*Ài`ˆVÌÊ̅iÊ«Àœ`ÕV̭îÊvœÀÊi>V…ÊÀi>V̈œ˜ÊLiœÜ\

O

O H3O+

(a) (a) O

O

O

?

OMe OMe

H3O+

?

H3O+

?

(b)

OH O

(c)

Ph Ph

O O

(b)

klein_c20_001-056v1.4.indd 15

O

O O

O

(c)

OH

O

H3O+

?

O

(d)

H H

30/04/10 16.47

16

CHAPTER 20

Aldehydes and Ketones

MEDICALLYSPEAKING Acetals as Prodrugs ˜Ê …>«ÌiÀÊ£™ÊÜiÊiÝ«œÀi`Ê̅iÊVœ˜Vi«ÌÊ œvÊ «Àœ`ÀÕ}Ãp«…>À“>Vœœ}ˆV>ÞÊ ˆ˜>V̈ÛiÊ Vœ“«œÕ˜`ÃÊ Ì…>ÌÊ >ÀiÊ Vœ˜ÛiÀÌi`Ê LÞÊ Ì…iÊ Lœ`ÞÊ ˆ˜ÌœÊ >V̈ÛiÊ Vœ“«œÕ˜`Ã°Ê >˜ÞÊ ÃÌÀ>Ìi}ˆiÃÊ >ÀiÊ ÕÃi`Ê ˆ˜Ê ̅iÊ `iÈ}˜Ê œvÊ «Àœ`ÀÕ}Ã°Ê "˜iÊ ÃÕV…Ê ÃÌÀ>Ìi}ÞÊ ˆ˜ÛœÛiÃÊ >˜Ê>ViÌ>Ê“œˆiÌÞ° ÃÊ >˜Ê iÝ>“«i]Ê y՜Vˆ˜œ˜ˆ`iÊ ˆÃÊ >Ê «Àœ`ÀÕ}Ê Ì…>ÌÊ Vœ˜Ì>ˆ˜ÃÊ >˜Ê >ViÌ>Ê “œˆiÌÞ]Ê >˜`Ê ˆÃÊ Ãœ`ʈ˜Ê>ÊVÀi>“ÊÕÃi`ÊvœÀÊ̅iÊ̜«ˆV>ÊÌÀi>̓i˜ÌÊ œvÊiVâi“>Ê>˜`ʜ̅iÀÊΈ˜ÊVœ˜`ˆÌˆœ˜Ã°Ê

-Žˆ˜Ê …>ÃÊ ÃiÛiÀ>Ê ˆ“«œÀÌ>˜ÌÊ v՘V̈œ˜Ã]Ê ˆ˜VÕ`ˆ˜}Ê «ÀiÛi˜Ìˆ˜}Ê Ì…iÊ>LÜÀ«Ìˆœ˜ÊœvÊvœÀiˆ}˜ÊÃÕLÃÌ>˜ViÃʈ˜ÌœÊ̅iÊ}i˜iÀ>ÊVˆÀVՏ>̈œ˜°Ê /…ˆÃÊvi>ÌÕÀiÊ«ÀœÌiVÌÊÕÃÊvÀœ“Ê…>À“vՏÊÃÕLÃÌ>˜ViÃ]ÊLÕÌʈÌÊ>ÃœÊ«ÀiÛi˜ÌÃÊLi˜iwVˆ>Ê`ÀÕ}ÃÊvÀœ“Ê«i˜iÌÀ>̈˜}Ê`ii«Êˆ˜ÌœÊ̅iÊΈ˜°Ê/…ˆÃÊ ivviVÌʈÃʓœÃÌÊ«Àœ˜œÕ˜Vi`ÊvœÀÊ`ÀÕ}ÃÊVœ˜Ì>ˆ˜ˆ˜}Ê"Ê}ÀœÕ«ÃÊ̅>ÌÊ V>˜Êˆ˜ÌiÀ>VÌÊ܈̅ÊLˆ˜`ˆ˜}ÊÈÌiÃʜ˜Ê̅iÊΈ˜½ÃÊÃÕÀv>Vi°Ê/œÊVˆÀVՓÛi˜ÌÊ Ì…ˆÃÊ«ÀœLi“]ÊÌܜÊ"Ê}ÀœÕ«ÃÊV>˜ÊLiÊÌi“«œÀ>ÀˆÞÊVœ˜ÛiÀÌi`ʈ˜ÌœÊ >˜Ê>ViÌ>°Ê/…iÊ>ViÌ>Ê«Àœ`ÀÕ}ʈÃÊV>«>LiʜvÊ«i˜iÌÀ>̈˜}Ê̅iÊΈ˜Ê “œÀiÊ `ii«Þ]Ê LiV>ÕÃiÊ ˆÌÊ >VŽÃÊ Ì…iÊ "Ê }ÀœÕ«ÃÊ Ì…>ÌÊ Lˆ˜`Ê ÌœÊ Ì…iÊ ÃŽˆ˜°Ê "˜ViÊ Ì…iÊ «Àœ`ÀÕ}Ê Ài>V…iÃÊ ˆÌÃÊ Ì>À}iÌ]Ê Ì…iÊ >ViÌ>Ê “œˆiÌÞÊ ˆÃÊ ÃœÜÞʅÞ`ÀœÞâi`]Ê̅iÀiLÞÊÀii>Ș}Ê̅iÊ>V̈ÛiÊ`ÀÕ}\

O

O O O

O HO

O

H F

Acetal moiety is removed

O

HO OH

H

H Fluocinonide

F

O

+

O OH

O

H Active drug

O F

F /Ài>̓i˜ÌÊ ÜˆÌ…Ê y՜Vˆ˜œ˜ˆ`iÊ ˆÃÊ Ãˆ}˜ˆwV>˜ÌÞÊ “œÀiÊ ivviV̈ÛiÊ Ì…>˜Ê `ˆÀiVÌÊÌÀi>̓i˜ÌÊ܈̅Ê̅iÊ>V̈ÛiÊ`ÀÕ}]ÊLiV>ÕÃiÊ̅iʏ>ÌÌiÀÊV>˜˜œÌÊ Ài>V…Ê>ÊœvÊ̅iÊ>vviVÌi`Ê>Ài>ðÊ

Stable Hemiacetals In the previous section, we saw how to convert an aldehyde or ketone into an acetal. In most cases, it is very difficult to isolate the intermediate hemiacetal: O

RO + 2 ROH

Favored by the equilibrium

OH



RO + ROH

OR



+ H2O

Hemiacetal

Acetal

Difficult to isolate

Favored when water is removed

For ketones we saw that the equilibrium generally favors the reactants unless water is removed, which enables formation of the acetal. The hemiacetal is not favored under either set of conditions (with or without removal of water). However, when a compound contains both a carbonyl group and a hydroxyl group, the resulting cyclic hemiacetal can often be isolated; for example:

klein_c20_001-056v1.4.indd 16

30/04/10 16.47

20.6

O

17

Nitrogen Nucleophiles

OH

O

[H+]

HO Cyclic hemiacetal

This will be important when we learn about carbohydrate chemistry in Chapter 24. Glucose, the major source of energy for the body, exists primarily as a cyclic hemiacetal: OH

OH

HO

O



HO H

O

HO HO OH OH

OH

OH

Glucose (Open-chain)

Glucose (Cyclic hemiacetal)

CONCEPTUAL CHECKPOINT O

HO

20.13 À>ÜÊ>Ê«>ÕÈLiʓiV…>˜ˆÃ“ÊvœÀÊ̅iÊvœœÜˆ˜}ÊÌÀ>˜ÃvœÀ“>̈œ˜\

[H2SO4]

OH

20.14 œ“«œÕ˜`Ê Ê …>ÃÊ “œiVՏ>ÀÊ vœÀ“Տ>Ê nH£{"Ó°Ê 1«œ˜Ê ÌÀi>̓i˜ÌÊ܈̅ÊV>Ì>Þ̈VÊ>Vˆ`]ÊVœ“«œÕ˜`ÊʈÃÊVœ˜ÛiÀÌi`ʈ˜ÌœÊ̅iÊ VÞVˆVʅi“ˆ>ViÌ>°Ê`i˜ÌˆvÞÊ̅iÊÃÌÀÕVÌÕÀiʜvÊVœ“«œÕ˜`Ê°

HO

O

O

[H+]

Compound A

20.6 Nitrogen Nucleophiles Primary Amines In mildly acidic conditions, an aldehyde or ketone will react with a primary amine to form an imine: O

N H

[H+]

CH3

H

CH3NH2

Imines are compounds that possess a C5N double bond and are common in biological pathways. Imines are also called Schiff bases, named after Hugo Schiff, a German chemist who first described their formation. A six-step mechanism for imine formation is shown in Mechanism 20.6. It is best to divide the mechanism conceptually into two parts (just as we did to conceptualize the mechanism of acetal formation): (1) The first three steps produce an intermediate called a carbinolamine and (2) the last three steps convert the carbinolamine into an imine:

klein_c20_001-056v1.4.indd 17

30/04/10 16.47

18

CHAPTER 20

Aldehydes and Ketones

MECHANISM 20.6 IMINE FORMATION Nucleophilic attack

Proton transfer

O

+

O

+

H A

H

H

Proton transfer

H N

OH

R

OH

A

H

H

N

N

+H

The amine attacks the protonated carbonyl to generate a tetrahedral intermediate

The carbonyl group is protonated, rendering it more electrophilic

R

The tetrahedral intermediate is deprotonated to form a carbinolamine

R Carbinolamine

Proton transfer +

H A

Loss of a leaving group

Proton transfer H + R N

R N

A

–H2O

The OH group is protonated, thereby converting it into an excellent leaving group

H + H O H N

Imine

The intermediate is deprotonated, to generate an imine

Note: There is experimental evidence that the first two steps of this mechanism (protonation and nucleophilic attack) more likely occur either simultaneously or in the reverse order of what is shown above. Most nitrogen nucleophiles are sufficiently nucleophilic to attack a carbonyl group directly, before protonation occurs. Nevertheless, the first two steps of the mechanism above have been drawn in the order shown (which

FIGURE 20.5 /…iÊÃiµÕi˜ViʜvÊÃÌi«Ãʈ˜ÛœÛi`Ê ˆ˜ÊvœÀ“>̈œ˜ÊœvÊ>ÊV>ÀLˆ˜œ>“ˆ˜i°

R

Water leaves, forming a C“N double bond

only rarely occurs), because this sequence enables a more effective comparison of all acid-catalyzed mechanisms in this chapter and also unifies the rationale behind proton transfers, as we will discuss in Sections 20.6 and 20.7. Interested students can learn more from the following literature references: 1. J. Am. Chem. Soc., 1974, 96(26), 7998–09 2. J. Org. Chem., 2007, 72(22), 8202–8215

Nucleophilic attack

Proton transfer

Proton transfer

Let’s begin our analysis of this mechanism by focusing on the first part: formation of the carbinolamine, which involves the three steps in Figure 20.5. Notice that these three steps are identical to the first three steps of acetal formation. Specifically, this sequence of steps involves a nucleophilic attack that is sandwiched between proton transfer steps. The identity of the acid, HA+, is most likely a protonated amine, which received its extra proton from the acid source: N H

+

A

H

+

N

H

R

Once the carbinolamine has been formed, formation of the imine is accomplished with three steps (Figure 20.6). Notice again that this reaction sequence begins and ends with proton transfers. FIGURE 20.6 /…iÊÃiµÕi˜ViʜvÊÃÌi«ÃÊ̅>ÌÊ Vœ˜ÛiÀÌÊ>ÊV>ÀLˆ˜œ>“ˆ˜iʈ˜ÌœÊ>˜Ê ˆ“ˆ˜i°

klein_c20_001-056v1.4.indd 18

Proton transfer

Loss of a leaving group

Proton transfer

30/04/10 16.47

20.6

19

Nitrogen Nucleophiles

Rate

The pH of the solution is an important consideration during imine formation, with the rate of reaction being greatest when the pH is around 4.5 (Figure 20.7). If the pH is too high (i.e., if no acid catalyst is used), the carbonyl group is not protonated (step 1 of the mechanism) and the carbinolamine is also not protonated (step 4 of the mechanism); so the reaction occurs more slowly. If the pH is too low (too much acid is used), most of the amine molecules will be protonated: H

0

1

2

3

4

5

6

7

H

H

pH

H

H

N

R

FIGURE 20.7 /…iÊÀ>Ìiʜvʈ“ˆ˜iÊvœÀ“>̈œ˜Ê>ÃÊ>Ê v՘V̈œ˜ÊœvÊ«°

O+

+

N

R

H

H

H NOT a nucleophile

A nucleophile

Under these conditions, step 2 of the mechanism occurs too slowly. As a result, care must be taken to ensure optimal pH of the solution during imine formation.

SKILLBUILDER 20.3 DRAWING THE MECHANISM OF IMINE FORMATION LEARN the skill

À>ÜÊ>Ê«>ÕÈLiʓiV…>˜ˆÃ“ÊvœÀÊ̅iÊvœœÜˆ˜}ÊÌÀ>˜ÃvœÀ“>̈œ˜\ Et N

O [H2SO4] EtNH2 –H2O

SOLUTION /…iÊÀi>V̈œ˜Ê>LœÛiʈÃÊ>˜ÊiÝ>“«iʜvʈ“ˆ˜iÊvœÀ“>̈œ˜°Ê/…iʓiV…>˜ˆÃ“ÊV>˜ÊLiÊ`ˆÛˆ`i`ʈ˜ÌœÊ Ìܜʫ>ÀÌÃ\Ê­£®ÊvœÀ“>̈œ˜ÊœvÊ̅iÊV>ÀLˆ˜œ>“ˆ˜iÊ>˜`Ê­Ó®ÊvœÀ“>̈œ˜ÊœvÊ̅iʈ“ˆ˜i° œÀ“>̈œ˜ÊœvÊ̅iÊV>ÀLˆ˜œ>“ˆ˜iʈ˜ÛœÛiÃÊ̅ÀiiʓiV…>˜ˆÃ̈VÊÃÌi«Ã\ Proton transfer

Nucleophilic attack

Proton transfer

7…i˜Ê `À>܈˜}Ê Ì…iÃiÊ Ì…ÀiiÊ ÃÌi«Ã]Ê “>ŽiÊ ÃÕÀiÊ ÌœÊ «>ViÊ Ì…iÊ …i>`Ê >˜`Ê Ì>ˆÊ œvÊ iÛiÀÞÊ VÕÀÛi`Ê >ÀÀœÜʈ˜ÊˆÌÃÊ«ÀiVˆÃiʏœV>̈œ˜]Ê>˜`ʓ>ŽiÊÃÕÀiÊ̜ʫ>ViÊ>Ê«œÃˆÌˆÛiÊV…>À}iÃʈ˜Ê̅iˆÀÊ>««Àœ«Àˆ>ÌiÊÊ œV>̈œ˜Ã°Ê œÌˆViÊ̅>ÌÊiÛiÀÞÊÃÌi«ÊÀiµÕˆÀiÃÊÌܜÊVÕÀÛi`Ê>ÀÀœÜð The tail of this arrow should be placed on a lone pair STEP 1 À>ÜÊ̅iÊ̅ÀiiÊÃÌi«ÃÊ ˜iViÃÃ>ÀÞÊ̜ÊvœÀ“Ê>Ê V>ÀLˆ˜œ>“ˆ˜i°

O

H

+

Don’t forget the positive charges +

O

H N H Et

Don’t forget the second curved arrow that shows release of the proton

klein_c20_001-056v1.4.indd 19

H

H H

H H

N Et

H

HO

+

N

Et

H

N

H

HO

N

Et

Et

Carbinolamine

30/04/10 16.47

20

CHAPTER 20

Aldehydes and Ketones

œÜÊ i̽ÃÊ vœVÕÃÊ œ˜Ê ̅iÊ ÃiVœ˜`Ê «>ÀÌÊ œvÊ Ì…iÊ “iV…>˜ˆÃ“]Ê ˆ˜Ê ܅ˆV…Ê ̅iÊ V>ÀLˆ˜œ>“ˆ˜iÊ ˆÃÊ Vœ˜ÛiÀÌi`ʈ˜ÌœÊ>˜Êˆ“ˆ˜i°Ê/…ˆÃÊÀiµÕˆÀiÃÊ̅ÀiiÊÃÌi«Ã\ Proton transfer

Loss of a leaving group

Proton transfer

"˜ViÊ>}>ˆ˜]Ê̅ˆÃÊÃiµÕi˜ViʜvÊÃÌi«ÃÊLi}ˆ˜ÃÊ܈̅Ê>Ê«ÀœÌœ˜ÊÌÀ>˜ÃviÀÊ>˜`Êi˜`ÃÊ܈̅Ê>Ê«ÀœÌœ˜Ê ÌÀ>˜ÃviÀ°Ê>ŽiÊÃÕÀiÊ̜ʫ>ViÊ̅iʅi>`Ê>˜`ÊÌ>ˆÊœvÊiÛiÀÞÊVÕÀÛi`Ê>ÀÀœÜʈ˜ÊˆÌÃÊ«ÀiVˆÃiʏœV>̈œ˜]Ê >˜`ʓ>ŽiÊÃÕÀiÊ̜ʫ>ViÊ>Ê«œÃˆÌˆÛiÊV…>À}iÃʈ˜Ê̅iˆÀÊ>««Àœ«Àˆ>ÌiʏœV>̈œ˜Ã\ The tail of this arrow should be placed on a lone pair STEP 2 À>ÜÊ̅iÊ̅ÀiiÊÃÌi«ÃÊ ˜iViÃÃ>ÀÞÊ̜ÊVœ˜ÛiÀÌÊ̅iÊ V>ÀLˆ˜œ>“ˆ˜iʈ˜ÌœÊ>˜Ê ˆ“ˆ˜i°

Don’t forget the positive charges

H N

HO

H H

Et

+

+

H

H N H

O

H N

H + Et N Et

Et H

–H2O

Et

N

N

H

Et

Imine

Don’t forget the second curved arrow that shows release of the proton

PRACTICE the skill 20.15 À>ÜÊ>Ê«>ÕÈLiʓiV…>˜ˆÃ“ÊvœÀÊi>V…ÊœvÊ̅iÊvœœÜˆ˜}ÊÌÀ>˜ÃvœÀ“>̈œ˜Ã\ O

Et

N

O

N

[ TsOH] MeNH2

[ TsOH] EtNH2

–H2O

–H2O

(a)

APPLY the skill

(b)

20.16

*Ài`ˆVÌÊ̅iʓ>œÀÊ«Àœ`ÕVÌÊvœÀÊi>V…ÊœvÊ̅iÊvœœÜˆ˜}ÊÀi>V̈œ˜Ã\ O [H+] NH3

–H2O

(a) 20.17

? [H+]

NH2

H

20.18

O

[H+] –H2O

(b)

?

*Ài`ˆVÌÊ̅iʓ>œÀÊ«Àœ`ÕVÌÊvœÀÊi>V…ÊœvÊ̅iÊvœœÜˆ˜}ʈ˜ÌÀ>“œiVՏ>ÀÊÀi>V̈œ˜Ã\ O

(a)

NH2

–H2O

O

?

[H+]

NH2

(b)

–H2O

?

`i˜ÌˆvÞÊ̅iÊÀi>VÌ>˜ÌÃÊ̅>ÌÊޜÕÊܜՏ`ÊÕÃiÊ̜ʓ>ŽiÊi>V…ÊœvÊ̅iÊvœœÜˆ˜}ʈ“ˆ˜iÃ\ N

N

(a)

N

(b)

(c)

need more PRACTICE? Try Problems 20.72, 20.86

Many different compounds of the form RNH2 will react with aldehydes and ketones, including compounds in which R is not an alkyl group. In the following examples, the R group of the amine has been replaced with a group that has been highlighted in red:

R

OH

[H+] HO–NH2

O R

–H2O

R

R

An oxime

klein_c20_001-056v1.4.indd 20

[H+] H2N–NH2

O

N R

R

–H2O

N R

NH2

R

A hydrazone

30/04/10 16.47

20.6

LOOKING AHEAD

21

Nitrogen Nucleophiles

When hydroxylamine (NH2OH) is used as a nucleophile, an oxime is formed. When hydrazine (NH2NH2) is used as a nucleophile, a hydrazone is formed. The mechanism for each of these reactions is directly analogous to the mechanism of imine formation.

Þ`À>✘iÃÊ>ÀiÊÃޘ̅ïV>ÞÊ ÕÃivՏ]Ê>ÃÊÜiÊ܈ÊÃiiÊ ˆ˜Ê̅iÊ`ˆÃVÕÃȜ˜ÊœvÊ̅iÊ 7œvv‡ˆÃ…˜iÀÊÀi`ÕV̈œ˜Ê >ÌiÀʈ˜Ê̅ˆÃÊV…>«ÌiÀ°

PRACTICALLYSPEAKING Beta-Carotene and Vision iÌ>‡V>ÀœÌi˜iÊ ˆÃÊ >Ê ˜>ÌÕÀ>ÞÊ œVVÕÀÀˆ˜}Ê Vœ“«œÕ˜`Ê vœÕ˜`Ê ˆ˜Ê “>˜ÞÊ œÀ>˜}i‡VœœÀi`Ê vÀՈÌÃÊ >˜`Ê Ûi}iÌ>LiÃ]Ê ˆ˜VÕ`ˆ˜}Ê V>ÀÀœÌÃ]Ê ÃÜiiÌÊ «œÌ>̜iÃ]Ê «Õ“«Žˆ˜Ã]Ê “>˜}œiÃ]Ê V>˜Ì>œÕ«iÃ]Ê >˜`Ê >«ÀˆVœÌÃ°Ê ÃÊ “i˜Ìˆœ˜i`ʈ˜Ê̅iÊV…>«ÌiÀʜ«i˜iÀ]ÊLiÌ>‡V>ÀœÌi˜iʈÃʎ˜œÜ˜Ê̜ÊLiÊ }œœ`Ê vœÀÊ ÞœÕÀÊ iÞiÃ°Ê /œÊ ՘`iÀÃÌ>˜`Ê Ü…Þ]Ê ÜiÊ “ÕÃÌÊ iÝ«œÀiÊ Ü…>ÌÊ …>««i˜ÃÊ̜ÊLiÌ>‡V>ÀœÌi˜iʈ˜ÊޜÕÀÊLœ`Þ°Ê“ˆ˜iÊvœÀ“>̈œ˜Ê«>ÞÃÊ>˜Ê ˆ“«œÀÌ>˜ÌÊÀœiʈ˜Ê̅iÊ«ÀœViÃð

ı-carotene

iÌ>‡V>ÀœÌi˜iÊ ˆÃÊ “iÌ>Lœˆâi`Ê ˆ˜Ê ̅iÊ ˆÛiÀÊ ÌœÊ «Àœ`ÕViÊ ÛˆÌ>“ˆ˜ÊÊ­>ÃœÊV>i`ÊÀ®\

OH Vitamin A (Retinol)

6ˆÌ>“ˆ˜Ê Ê ˆÃÊ Ì…i˜Ê œÝˆ`ˆâi`]Ê >˜`ʜ˜iʜvÊ̅iÊ`œÕLiÊLœ˜`ÃÊ Õ˜`iÀ}œiÃÊ ˆÃœ“iÀˆâ>̈œ˜Ê ÌœÊ «Àœ`ÕViÊ££‡cis‡Àï˜>\

OH This bond undergoes isomerization

This group is oxidized 11-cis-retinal

/…iÊÀiÃՏ̈˜}Ê>`i…Þ`iÊ̅i˜Ê Ài>VÌÃÊ ÜˆÌ…Ê >˜Ê >“ˆ˜œÊ }ÀœÕ«Ê œvÊ>Ê«ÀœÌiˆ˜Ê­V>i`ʜ«Ãˆ˜®ÊÌœÊ «Àœ`ÕViÊ À…œ`œ«Ãˆ˜]Ê Ü…ˆV…Ê «œÃÃiÃÃiÃÊ>˜Êˆ“ˆ˜iʓœˆiÌÞ\

H

O

H2N–Protein

11-cis-retinal

H

O

H

N

Protein

Rhodopsin

ÃÊ`iÃVÀˆLi`ʈ˜Ê-iV̈œ˜Ê£Ç°£Î]ÊÀ…œ`œ«Ãˆ˜ÊV>˜Ê>LÜÀLÊ>Ê«…œÌœ˜Êœvʏˆ}…Ì]ʈ˜ˆÌˆ>̈˜}Ê>Ê«…œÌœˆÃœ“iÀˆâ>̈œ˜ÊœvÊ̅iÊcisÊ`œÕLiÊLœ˜`Ê ÌœÊ vœÀ“Ê >Ê transÊ `œÕLiÊ Lœ˜`°Ê /…iÊ ÀiÃՏ̈˜}Ê V…>˜}iÊ ˆ˜Ê }iœ“iÌÀÞÊ ÌÀˆ}}iÀÃÊ>ÊÈ}˜>Ê̅>ÌʈÃÊՏ̈“>ÌiÞÊ`iÌiVÌi`ÊLÞÊ̅iÊLÀ>ˆ˜Ê>˜`ʈ˜ÌiÀ«ÀiÌi`Ê>ÃÊۈȜ˜°

klein_c20_001-056v1.4.indd 21

Ê `iwVˆi˜VÞÊ œvÊ ÛˆÌ>“ˆ˜Ê Ê V>˜Ê i>`Ê ÌœÊ º˜ˆ}…ÌÊ Lˆ˜`˜iÃÃ]»Ê >Ê Vœ˜`ˆÌˆœ˜Ê ̅>ÌÊ «ÀiÛi˜ÌÃÊ Ì…iÊ iÞiÃÊ vÀœ“Ê >`ÕÃ̈˜}Ê ÌœÊ `ˆ“ÞÊ ˆÌÊÊ i˜ÛˆÀœ˜“i˜Ìð

30/04/10 16.47

22

CHAPTER 20

Aldehydes and Ketones

CONCEPTUAL CHECKPOINT 20.19 *Ài`ˆVÌÊ̅iÊ«Àœ`ÕVÌʜvÊi>V…ÊœvÊ̅iÊvœœÜˆ˜}ÊÀi>V̈œ˜Ã\

20.20 `i˜ÌˆvÞÊ̅iÊÀi>VÌ>˜ÌÃÊ̅>ÌÊޜÕÊܜՏ`ÊÕÃiÊ̜ʓ>ŽiÊi>V…ÊœvÊ Ì…iÊvœœÜˆ˜}ÊVœ“«œÕ˜`Ã\

[H+]

O

HO–NH2 –H2O

(a)

?

N

OH

N

(a) O

NH2

(b)

[H+]

?

H2N–NH2 –H2O

(b)

Secondary Amines

R

R N

O

In acidic conditions, an aldehyde or ketone will react with a secondary amine to form an enamine:

[H+]

Enamines are compounds in which the nitrogen lone pair is delocalized by the presence of an adjacent C5C double bond. A mechanism for enamine formation is shown in Mechanism 20.7

R N

H

R

An enamine

MECHANISM 20.7 ENAMINE FORMATION Proton transfer

Nucleophilic attack +

O

H

O

+

R

H N

OH

R

H¬A

Proton transfer

OH

A

H N

N

R The carbonyl group is protonated, rendering it more electrophilic

The amine attacks the protonated carbonyl to generate a tetrahedral intermediate

+

R

R

R

The tetrahedral intermediate is deprotonated to form a carbinolamine

Carbinolamine

Proton transfer

H¬A

Loss of a leaving group

Proton transfer R

R N

A

Enamine

The intermediate is deprotonated to generate an imine

R + R N

–H2O

Note: There is experimental evidence that the first two steps of this mechanism (protonation and nucleophilic attack) more likely occur either simultaneously or in the reverse order of what is shown above. Most nitrogen nucleophiles are sufficiently nucleophilic to attack a carbonyl group directly, before protonation occurs. Nevertheless, the first two steps of the mechanism above have been drawn in the order shown (which

Water leaves and a C“N double bond forms

The OH group is protonated thereby converting it into an excellent leaving group

H + H O

H

klein_c20_001-056v1.4.indd 22

+

N

R

R

only rarely occurs), because this sequence enables a more effective comparison of all acid-catalyzed mechanisms in this chapter and also unifies the rationale behind proton transfers, as we will discuss in Sections 20.6 and 20.7. Interested students can learn more from the following literature references: 1. J. Am. Chem. Soc., 1974, 96(26), 7998–09 2. J. Org. Chem., 2007, 72(22), 8202–8215

30/04/10 16.47

20.6

23

Nitrogen Nucleophiles

This mechanism of enamine formation is identical to the mechanism of imine formation except for the last step: R + H N

R N

[H+]

RNH2

RNH2

O

An imine

R

R + R N

R N

H

[H+]

R2NH

R2NH

An enamine

The difference in the iminium ions explains the different outcomes for the two reactions. During imine formation, the nitrogen atom of the iminium ion possesses a proton that can be removed as the final step of the mechanism. In contrast, during enamine formation, the nitrogen atom of the iminium ion does not possess a proton. As a result, elimination from the adjacent carbon is necessary in order to yield a neutral species.

SKILLBUILDER 20.4

DRAWING THE MECHANISM OF ENAMINE FORMATION Et

LEARN the skill

O

À>ÜÊ>Ê«>ÕÈLiʓiV…>˜ˆÃ“ÊvœÀÊ̅iÊvœœÜˆ˜}ÊÀi>V̈œ˜\

Et N

[H2SO4] Et2NH –H2O

SOLUTION /…iÊÀi>V̈œ˜Ê>LœÛiʈÃÊ>˜ÊiÝ>“«iʜvÊi˜>“ˆ˜iÊvœÀ“>̈œ˜°Ê/…iʓiV…>˜ˆÃ“ÊV>˜ÊLiÊ`ˆÛˆ`i`ʈ˜ÌœÊ Ìܜʫ>ÀÌÃ\Ê­£®ÊvœÀ“>̈œ˜ÊœvÊ̅iÊV>ÀLˆ˜œ>“ˆ˜iÊ>˜`Ê­Ó®ÊvœÀ“>̈œ˜ÊœvÊ̅iÊi˜>“ˆ˜i° œÀ“>̈œ˜ÊœvÊ̅iÊV>ÀLˆ˜œ>“ˆ˜iʈ˜ÛœÛiÃÊ̅ÀiiʓiV…>˜ˆÃ̈VÊÃÌi«Ã\ Proton transfer

Nucleophilic attack

Proton transfer

7…i˜Ê `À>܈˜}Ê Ì…iÃiÊ Ì…ÀiiÊ ÃÌi«Ã]Ê “>ŽiÊ ÃÕÀiÊ ÌœÊ «>ViÊ Ì…iÊ …i>`Ê >˜`Ê Ì>ˆÊ œvÊ iÛiÀÞÊ VÕÀÛi`Ê >ÀÀœÜʈ˜ÊˆÌÃÊ«ÀiVˆÃiʏœV>̈œ˜]Ê>˜`ʓ>ŽiÊÃÕÀiÊ̜ʫ>ViÊ>Ê«œÃˆÌˆÛiÊV…>À}iÃʈ˜Ê̅iˆÀÊ>««Àœ«Àˆ>ÌiÊÊ œV>̈œ˜Ã°Ê œÌˆViÊ̅>ÌÊiÛiÀÞÊÃÌi«ÊÀiµÕˆÀiÃÊÌܜÊVÕÀÛi`Ê>ÀÀœÜð Don’t forget the positive charges

The tail of this arrow should be placed on a lone pair STEP 1 À>ÜÊ̅iÊ̅ÀiiÊÃÌi«ÃÊ ˜iViÃÃ>ÀÞÊ̜ÊvœÀ“Ê>Ê V>ÀLˆ˜œ>“ˆ˜i°

O

N

+

+

H

O

H N Et Et

Don’t forget the second curved arrow that shows release of the proton

klein_c20_001-056v1.4.indd 23

H H

N Et

Et

HO

Et

Et N+ Et

H

N

Et

HO

N Et

Et

Carbinolamine

30/04/10 16.47

24

CHAPTER 20

Aldehydes and Ketones

˜Ê̅iÊÃiVœ˜`Ê«>ÀÌʜvÊ̅iʓiV…>˜ˆÃ“]Ê̅iÊV>ÀLˆ˜œ>“ˆ˜iʈÃÊVœ˜ÛiÀÌi`ʈ˜ÌœÊ>˜Êi˜>“ˆ˜iÊ Ûˆ>Ê>Ê̅Àii‡ÃÌi«Ê«ÀœViÃðÊ"˜ViÊ>}>ˆ˜]Ê̅ˆÃÊÃiµÕi˜ViÊLi}ˆ˜ÃÊ>˜`Êi˜`ÃÊ܈̅ʫÀœÌœ˜ÊÌÀ>˜ÃviÀÃ\ Proton transfer

Loss of a leaving group

The tail of this arrow should be placed on a lone pair

Don’t forget the positive charges

H

Et STEP 2 À>ÜÊ̅iÊ̅ÀiiÊÃÌi«ÃÊ ˜iViÃÃ>ÀÞÊ̜ÊVœ˜ÛiÀÌÊ̅iÊ V>ÀLˆ˜œ>“ˆ˜iʈ˜ÌœÊ>˜Ê i˜>“ˆ˜i°

HO

N Et

Et

+

H H N Et

Et + N

N Et

H O

+

Proton transfer

Et H

H

–H2O

Et

Et N

Et N

Et

Et

Enamine

Don’t forget the second curved arrow that shows release of the proton

PRACTICE the skill 20.21 À>ÜÊ>Ê«>ÕÈLiʓiV…>˜ˆÃ“ÊvœÀÊi>V…ÊœvÊ̅iÊvœœÜˆ˜}ÊÀi>V̈œ˜Ã\ Et

Et N

O [TsOH]

–H2O

(a)

[H2SO4]

O

Et2NH

N

Et2NH –H2O

(b) N

O [H2SO4]

Me2NH –H2O

(c)

APPLY the skill

20.22

*Ài`ˆVÌÊ̅iʓ>œÀÊ«Àœ`ÕVÌÊvœÀÊi>V…ÊœvÊ̅iÊvœœÜˆ˜}ÊÀi>V̈œ˜Ã\ O

[H+] N –H2O

(a)

20.23

H

H N

?

O [H+] –H2O

(b)

*Ài`ˆVÌÊ̅iʓ>œÀÊ«Àœ`ÕVÌÊvœÀÊi>V…ÊœvÊ̅iÊvœœÜˆ˜}ʈ˜ÌÀ>“œiVՏ>ÀÊÀi>V̈œ˜Ã\ NH O

[H+] –H2O

H

O

?

(a) 20.24

? [H+]

N

–H2O

?

(b) `i˜ÌˆvÞÊ̅iÊÀi>VÌ>˜ÌÃÊ̅>ÌÊޜÕÊܜՏ`ÊÕÃiÊ̜ʓ>ŽiÊi>V…ÊœvÊ̅iÊvœœÜˆ˜}Êi˜>“ˆ˜iÃ\ N N

(a)

N

(b)

(c)

need more PRACTICE? Try Problem 20.72

klein_c20_001-056v1.4.indd 24

30/04/10 16.47

20.6

25

Nitrogen Nucleophiles

Wolff-Kishner Reduction At the end of the previous section, we noted that ketones can be converted into hydrazones. This transformation has practical utility, because hydrazones are readily reduced under strongly basic conditions: NNH2

H H KOH/H2O heat

A hydrazone

(82%)

This transformation is called the Wolff-Kishner reduction, named after the German chemist Ludwig Wolff (University of Jena) and the Russian chemist N. M. Kishner (University of Moscow). This provides a two-step procedure for reducing a ketone to an alkane: N

O

NH2 H H

[H+] H2N¬NH2

KOH / H2 O

–H2O

heat

+ N2 (80%)

The second part of the Wolff-Kishner reduction is believed to proceed via Mechanism 20.8.

MECHANISM 20.8 THE WOLFF-KISHNER REDUCTION Proton transfer

H

Proton transfer H

N N

H

H

N-

-

OH

N

N H

N

H

H N

N

Proton transfer

-

OH

Proton transfer H H

O

The intermediate is protonated

-

One of the protons is removed, forming a resonance-stabilized intermediate

H

H

O

H

The carbanion is protonated, generating the product

H -

Loss of a leaving group

Another proton is removed

H N

-

N

Nitrogen gas is expelled, generating a carbanion

Notice that four of the five steps in the mechanism are proton transfers, the exception being the loss of N2 gas to generate a carbanion. This step warrants special attention, because formation of a carbanion in a solution of aqueous hydroxide is thermodynamically unfavorable (significantly uphill in energy). Why, then, does this step occur? It is true that the equilibrium for this step greatly disfavors formation of the carbanion, and therefore, only a very small number of molecules will initially lose N2 to form the carbanion. However, the resulting N2 gas then

klein_c20_001-056v1.4.indd 25

30/04/10 16.47

26

CHAPTER 20

Aldehydes and Ketones

bubbles out of the reaction mixture, and the equilibrium is adjusted to form more nitrogen gas, which again leaves the reaction mixture. The evolution of nitrogen gas ultimately renders this step irreversible and forces the reaction to completion. As a result, the yields for this process are generally very good.

CONCEPTUAL CHECKPOINT 20.25 *Ài`ˆVÌÊ̅iÊ«Àœ`ÕVÌʜvÊ̅iÊÌܜ‡ÃÌi«Ê«ÀœVi`ÕÀiÊLiœÜ]Ê>˜`Ê`À>ÜÊ>Ê “iV…>˜ˆÃ“ÊvœÀʈÌÃÊvœÀ“>̈œ˜\

O 1) [H+], H2N–NH2, –H2O 2) KOH / H2O, heat

?

20.7 Mechanism Strategies Compare the mechanistic steps for the formation of acetals, imines, and enamines (Figure 20.8). Each of the mechanisms has been divided into two parts, and in all cases, the first part consists of the same three steps. In addition, even the second part of each mechanism begins with the same first two steps (proton transfer followed by loss of a leaving group). In other words, these three mechanisms are identical until the fifth step, in which water is lost (loss of a leaving group), shown in red in Figure 20.8. Rather than viewing these reactions as three separate, unrelated reactions, it is best to view them as nearly identical with different endings. Acetal formation has one additional nucleophile attack, giving a total of seven steps. In contrast, imine formation and enamine formation do not exhibit a nucleophilic attack during the second part of the mechanism, giving a total of only six steps. FORMATION OF HEMIACETAL Proton transfer

Nucleophilic attack

Proton transfer

FORMATION OF ACETAL

FORMATION OF CARBINOLAMINE Proton transfer

Nucleophilic attack

Proton transfer

Nucleophilic attack

Proton transfer

Nucleophilic attack

Proton transfer

FORMATION OF IMINE Loss of a leaving group

Proton transfer

FORMATION OF CARBINOLAMINE Proton transfer

Loss of a leaving group

Proton transfer

Proton transfer

FORMATION OF ENAMINE Proton transfer

Loss of a leaving group

Proton transfer

FIGURE 20.8 ÊVœ“«>ÀˆÃœ˜ÊœvÊ̅iÊÃiµÕi˜ViʜvÊÃÌi«ÃÊvœÀÊ>ViÌ>]ʈ“ˆ˜i]Ê>˜`Êi˜>“ˆ˜iÊvœÀ“>̈œ˜°

In each of these mechanisms, there are four proton transfer steps. In order to draw the mechanism correctly, it is critical to draw these proton transfers properly. To do so, it will be helpful to remember the following rules that dictate when and why proton transfers occur in acid-catalyzed conditions: s 4HECARBONYLGROUPSHOULDBEPROTONATEDBEFOREITISATTACKED4HISGENERATESAMORE powerful electrophile, and it avoids formation of a negative charge that would occur if the nucleophile attacked the carbonyl directly.

klein_c20_001-056v1.4.indd 26

30/04/10 16.47

20.7

Mechanism Strategies

27

s !VOIDFORMATIONOFTWOPOSITIVECHARGESONASINGLEINTERMEDIATE4HISTYPEOFINTERMEDIate will generally be too high in energy to form. s 4HELEAVINGGROUPSHOULDBECOMENEUTRALWHENITLEAVES$ONOTEXPELHYDROXIDEASA leaving group; rather, it should first be protonated so that it can leave as water. s !TTHEENDOFTHEMECHANISM APROTONTRANSFERISUSEDTOFORMANEUTRALPRODUCT The four rules above correspond with each of the four proton transfers, respectively. These four rules can be consolidated into one master rule that defines when and why proton transfer steps are utilized: In acidic conditions, all reagents, intermediates, and leaving groups should either be neutral (no charge) or bear one positive charge. All of the proton transfers in the mechanism occur in order to fulfill this requirement. Acetals, imines, and enamines can be converted back into ketones by treatment with excess water under acid-catalyzed conditions: RO

OR

H3O+

R N

R

O

H3O+

R N H3O+

Each of the reactions above is called a hydrolysis reaction, because in each case bonds are cleaved by treatment with water. These three reactions are essentially the reverse of the reactions we have seen. The following procedure should be used in drawing the mechanisms for the above hydrolysis reactions: 1.

Begin by drawing all of the intermediates without any curved arrows. For example, suppose that we want to draw the mechanism for hydrolysis of an acetal to form a ketone: RO

OR

H3O+

O

We did not learn the mechanism for this reaction; however, we did learn the mechanism for acetal formation. Think about the first intermediate in acetal formation (a protonated carbonyl), and then draw that intermediate as the last intermediate of the hydrolysis reaction: OR

+

H

O

O

OR The last intermediate should be a protonated carbonyl

Continue working backward until you have drawn all of the intermediates. 2.

Then, working forward, draw the curved arrows that are necessary to transform each intermediate into the next intermediate. At each stage, make sure you are following the master rule for proton transfers in acid-catalyzed conditions.

The skill of being able to draw the reverse of a known mechanism is incredibly important and will be used again for other reactions in the remaining chapters of this book. The following example illustrates this procedure.

klein_c20_001-056v1.4.indd 27

30/04/10 16.47

28

CHAPTER 20

Aldehydes and Ketones

SKILLBUILDER 20.5

DRAWING THE MECHANISM OF A HYDROLYSIS REACTION

LEARN the skill

*Àœ«œÃiÊ>ʓiV…>˜ˆÃ“ÊvœÀÊ̅iÊvœœÜˆ˜}ÊÌÀ>˜ÃvœÀ“>̈œ˜\

O

O

O H3O+

+ HO

OH

SOLUTION /…ˆÃʈÃÊ>ʅÞ`ÀœÞÈÃÊÀi>V̈œ˜Êˆ˜Ê܅ˆV…Ê>ÊVÞVˆVÊ>ViÌ>ÊˆÃʜ«i˜i`Ê̜ÊvœÀ“Ê>ʎi̜˜i°Ê7iÊ̅iÀivœÀiÊ iÝ«iVÌÊ̅iʓiV…>˜ˆÃ“Ê̜ÊLiÊ̅iÊÀiÛiÀÃiʜvÊ>ViÌ>ÊvœÀ“>̈œ˜°Ê i}ˆ˜ÊLÞÊVœ˜Ãˆ`iÀˆ˜}Ê>ÊœvÊ̅iÊ ˆ˜ÌiÀ“i`ˆ>ÌiÃʈ˜ÛœÛi`ʈ˜Ê>ViÌ>ÊvœÀ“>̈œ˜\

OH +

H +

HO

O

HO

H

First intermediate

Ketone

O

Acetal

LOOKING BACK vÊޜÕʘii`ÊÌœÊ «œˆÃ…ÊޜÕÀÊ>ÀÀœÜ‡ «Õň˜}ÊΈÃ]Ê}œÊ ̜Ê-iV̈œ˜ÊÈ°n°

klein_c20_001-056v1.4.indd 28

O

O

+

O

O

H

O

O

Acetal

7iÊ Ãˆ“«ÞÊ `À>ÜÊ >Ê œvÊ Ì…iÃiÊ ˆ˜ÌiÀ“i`ˆ>ÌiÃÊ ˆ˜Ê ÀiÛiÀÃiÊ œÀ`iÀÊ ÃœÊ Ì…>ÌÊ Ì…iÊ wÀÃÌÊ ˆ˜ÌiÀ“i`ˆ>ÌiÊ >LœÛiÊ­…ˆ}…ˆ}…Ìi`®ÊLiVœ“iÃÊ̅iʏ>ÃÌʈ˜ÌiÀ“i`ˆ>ÌiʜvÊ̅iʅÞ`ÀœÞÈÃʓiV…>˜ˆÃ“\

OH

O

+

O

+

H

OH

OH

OH

H

+

O

H

O

Hemiacetal

STEP 1 À>ÜÊ>Êˆ˜ÌiÀ“i`ˆ>ÌiÃÊ vœÀÊ>ViÌ>ÊvœÀ“>̈œ˜Êˆ˜Ê ÀiÛiÀÃiʜÀ`iÀ°

O

+

+

O

OH

OH

H

O

O

OH

H

O

+

O

HO

O

Hemiacetal

HO

+

O

H

O

O

Last intermediate

Ketone

H

Þ`ÀœÞÈÃʜvÊ̅iÊ>ViÌ>Ê“ÕÃÌʈ˜ÛœÛiÊ̅iÃiʈ˜ÌiÀ“i`ˆ>ÌiÃ]ʈ˜Ê̅iʜÀ`iÀÊŜܘÊ>LœÛi°ÊvÊ>˜ÞÊ œvÊ̅iʈ˜ÌiÀ“i`ˆ>ÌiÃʅ>ÃÊ>ʘi}>̈ÛiÊV…>À}i]Ê̅i˜ÊޜÕʅ>Ûiʓ>`iÊ>ʓˆÃÌ>Ži° 7ˆÌ…Ê̅iʈ˜ÌiÀ“i`ˆ>ÌiÃÊ«>Vi`ʈ˜Ê̅iÊVœÀÀiVÌʜÀ`iÀ]Ê̅iÊw˜>ÊÃÌi«ÊˆÃÊ̜Ê`À>ÜÊ̅iÊÀi>}i˜ÌÃÊ >˜`ÊVÕÀÛi`Ê>ÀÀœÜÃÊ̅>ÌÊŜÜʅœÜÊi>V…ʈ˜ÌiÀ“i`ˆ>ÌiʈÃÊÌÀ>˜ÃvœÀ“i`ʈ˜ÌœÊ̅iʘiÝÌʈ˜ÌiÀ“i`ˆ>Ìi°Ê i}ˆ˜Ê܈̅Ê̅iÊ>ViÌ>]Ê>˜`ÊܜÀŽÊvœÀÜ>À`Ê՘̈ÊÀi>V…ˆ˜}Ê̅iʎi̜˜i°Ê/…ˆÃÊÀiµÕˆÀiÃÊ̅>ÌÊ ÞœÕÀÊ>ÀÀœÜ‡«Õň˜}ÊΈÃÊ>Àiʈ˜Ê}œœ`ÊÅ>«i° >ŽiÊ ÃÕÀiÊ ÌœÊ ÕÃiÊ œ˜ÞÊ Ì…iÊ Ài>}i˜ÌÃÊ Ì…>ÌÊ >ÀiÊ «ÀœÛˆ`i`]Ê >˜`Ê œLiÞÊ Ì…iÊ “>ÃÌiÀÊ ÀՏiÊ vœÀÊ «ÀœÌœ˜ÊÌÀ>˜ÃviÀðÊœÀÊiÝ>“«i]Ê̅ˆÃÊ«ÀœLi“ʈ˜`ˆV>ÌiÃÊ̅>ÌÊÎ"+ʈÃÊ>Û>ˆ>Li°Ê/…ˆÃʓi>˜ÃÊ̅>ÌÊ HÎ"+ÊŜՏ`ÊLiÊÕÃi`ÊvœÀÊ«ÀœÌœ˜>̈˜}]Ê>˜`ÊÓ"ÊŜՏ`ÊLiÊÕÃi`ÊvœÀÊ`i«ÀœÌœ˜>̈˜}°Ê œÊ˜œÌÊ ÕÃiʅÞ`ÀœÝˆ`iʈœ˜Ã]Ê>ÃÊ̅iÞÊ>ÀiʘœÌÊ«ÀiÃi˜Ìʈ˜ÊÃÕvwVˆi˜ÌʵÕ>˜ÌˆÌÞÊ՘`iÀÊ>Vˆ`‡V>Ì>Þâi`ÊVœ˜`ˆÌˆœ˜Ã°Ê««ˆV>̈œ˜ÊœvÊ̅iÃiÊÀՏiÃÊ}ˆÛiÃÊ̅iÊvœœÜˆ˜}Ê>˜ÃÜiÀ\

30/04/10 16.47

20.7

OH

OH

O

O

O

O

+

O H

+

O

+

OH

H

+

H

29

Mechanism Strategies

H H

H

O

H

O

HO

O

O

O

H

H

H

Acetal

Hemiacetal

H

STEP 2 À>ÜÊ̅iÊVÕÀÛi`Ê>ÀÀœÜÃÊ >˜`ʘiViÃÃ>ÀÞÊÀi>}i˜ÌÃÊ vœÀÊi>V…ÊÃÌi«ÊœvÊ̅iÊ “iV…>˜ˆÃ“°

+

O H

H

OH H O

+

O

HO

+

O

O

HO

H

+

OH

H

H

Ketone

œÌˆViÊ̅iÊÕÃiʜvÊiµÕˆˆLÀˆÕ“Ê>ÀÀœÜÃ]ÊLiV>ÕÃiÊ̅iÊ«ÀœViÃÃʈÃÊ}œÛiÀ˜i`ÊLÞÊ>˜ÊiµÕˆˆLÀˆÕ“]Ê>ÃÊ ˜œÌi`ʈ˜Ê«ÀiۈœÕÃÊÃiV̈œ˜Ã°

PRACTICE the skill 20.26 *Àœ«œÃiÊ>Ê«>ÕÈLiʓiV…>˜ˆÃ“ÊvœÀÊi>V…ÊœvÊ̅iÊvœœÜˆ˜}ʅÞ`ÀœÞÈÃÊÀi>V̈œ˜Ã\ EtO

O

OEt H3

O+

O

H3O+

20.27

+

N

+ MeNH2 O

O

(d)

APPLY the skill

(b)

H

O

H3O+

(c)

O H3O+

+ 2 EtOH

(a)

N

N

H

HO

OH

*Àœ«œÃiÊ>Ê«>ÕÈLiʓiV…>˜ˆÃ“ÊvœÀÊ̅iÊÀi>V̈œ˜ÊLiœÜ\ O

O

N

[H2SO4]

OH H2N

HO

Hint: ÌʈÃʘœÌʘiViÃÃ>ÀÞÊ̜ʜ«i˜Ê̅iÊ>ViÌ>Êˆ˜ÌœÊ>ʎi̜˜iÊ­ÇÊÃÌi«Ã®ÊvœœÜi`ÊLÞÊVœÃˆ˜}Ê̅iÊ ˆ“ˆ˜iÊ­ÈÊÃÌi«Ã®°ÊÊ«>ÕÈLiʓiV…>˜ˆÃ“ÊV>˜ÊLiÊ`À>ܘÊ܈̅ÊviÜiÀÊ̅>˜Ê£ÎÊÃÌi«Ã°Ê-Ì>ÀÌÊܜÀŽˆ˜}Ê܈̅Ê̅iÊ>ViÌ>Ê­ÕȘ}Ê̅iÊÃÌi«ÃʘiViÃÃ>ÀÞÊ̜ʜ«i˜Ê>˜Ê>ViÌ>®]Ê>˜`ʏœœŽÊvœÀÊ>˜Êˆ˜ÌiÀ“i`ˆ>ÌiÊ̅>ÌÊV>˜ÊLiÊ>ÌÌ>VŽi`ÊLÞÊ̅iÊ>“ˆ˜œÊ}ÀœÕ«°Ê>ŽiÊÃÕÀiÊ̜Ê>ۜˆ`Ê`À>܈˜}Ê>˜Ê-NÓÊ«ÀœViÃÃÊ >ÌÊ>ÊÃÌiÀˆV>Þʅˆ˜`iÀi`ÊÃÕLÃÌÀ>Ìit need more PRACTICE?

klein_c20_001-056v1.4.indd 29

Try Problem 20.65

30/04/10 16.47

30

CHAPTER 20

Aldehydes and Ketones

The Medically Speaking box below provides two examples of hydrolysis reactions that are exploited in drug design.

MEDICALLYSPEAKING Prodrugs Methenamine as a Prodrug of Formaldehyde œÀ“>`i…Þ`iʅ>ÃÊ>˜ÌˆÃi«ÌˆVÊ«Àœ«iÀ̈iÃÊ>˜`ÊV>˜ÊLiÊi“«œÞi`ʈ˜Ê ̅iÊÌÀi>̓i˜ÌʜvÊÕÀˆ˜>ÀÞÊÌÀ>VÌʈ˜viV̈œ˜ÃÊ`ÕiÊ̜ʈÌÃÊ>LˆˆÌÞÊ̜ÊÀi>VÌÊ ÜˆÌ…Ê ˜ÕViœ«…ˆiÃÊ «ÀiÃi˜ÌÊ ˆ˜Ê ÕÀˆ˜i°Ê œÜiÛiÀ]Ê vœÀ“>`i…Þ`iÊ V>˜Ê LiÊ̜݈VÊ܅i˜ÊiÝ«œÃi`Ê̜ʜ̅iÀÊÀi}ˆœ˜ÃʜvÊ̅iÊLœ`Þ°Ê/…iÀivœÀi]Ê Ì…iÊÕÃiʜvÊvœÀ“>`i…Þ`iÊ>ÃÊ>˜Ê>˜ÌˆÃi«ÌˆVÊ>}i˜ÌÊÀiµÕˆÀiÃÊ>ʓi̅œ`Ê vœÀÊ ÃiiV̈ÛiÊ `iˆÛiÀÞÊ ÌœÊ Ì…iÊ ÕÀˆ˜>ÀÞÊ ÌÀ>VÌ°Ê /…ˆÃÊ V>˜Ê LiÊ >VVœ“«ˆÃ…i`ÊLÞÊÕȘ}Ê>Ê«Àœ`ÀÕ}ÊV>i`ʓi̅i˜>“ˆ˜i\ N N

N

i̅i˜>“ˆ˜iʈÃÊ«>Vi`ʈ˜ÊëiVˆ>ÊÌ>LiÌÃÊ̅>ÌÊ`œÊ˜œÌÊ`ˆÃ܏ÛiÊ >ÃÊ̅iÞÊÌÀ>ÛiÊ̅ÀœÕ}…Ê̅iÊ>Vˆ`ˆVÊi˜ÛˆÀœ˜“i˜ÌʜvÊ̅iÊÃ̜“>V…ÊLÕÌÊ `œÊ`ˆÃ܏Ûiʜ˜ViÊ̅iÞÊÀi>V…Ê̅iÊL>ÈVÊi˜ÛˆÀœ˜“i˜ÌʜvÊ̅iʈ˜ÌiÃ̈˜>ÊÌÀ>VÌ°Êi̅i˜>“ˆ˜iʈÃÊ̅iÀiLÞÊÀii>Ãi`ʈ˜Ê̅iʈ˜ÌiÃ̈˜>ÊÌÀ>VÌ]Ê Ü…iÀiÊ ˆÌÊ ˆÃÊ ÃÌ>LiÊ Õ˜`iÀÊ L>ÈVÊ Vœ˜`ˆÌˆœ˜Ã°Ê "˜ViÊ ˆÌÊ Ài>V…iÃÊ Ì…iÊ >Vˆ`ˆVÊ i˜ÛˆÀœ˜“i˜ÌÊ œvÊ Ì…iÊ ÕÀˆ˜>ÀÞÊ ÌÀ>VÌ]Ê “i̅i˜>“ˆ˜iÊ ˆÃÊ …Þ`ÀœÞâi`]Ê Àii>Ș}Ê vœÀ“>`i…Þ`i]Ê >ÃÊ Ã…œÜ˜Ê >LœÛi°Ê ˜Ê ̅ˆÃÊ Ü>Þ]Ê “i̅i˜>“ˆ˜iʈÃÊÕÃi`Ê>ÃÊ>Ê«Àœ`ÀÕ}Ê̅>ÌÊi˜>LiÃÊ`iˆÛiÀÞʜvÊvœÀ“>`i…Þ`iÊ Ã«iVˆwV>ÞÊ ÌœÊ Ì…iÊ ÕÀˆ˜>ÀÞÊ ÌÀ>VÌ°Ê /…ˆÃÊ “i̅œ`Ê «ÀiÛi˜ÌÃÊ Ì…iÊÃÞÃÌi“ˆVÊÀii>ÃiʜvÊvœÀ“>`i…Þ`iʈ˜ÊœÌ…iÀʜÀ}>˜ÃʜvÊ̅iÊLœ`ÞÊ Ü…iÀiʈÌÊܜՏ`ÊLiÊ̜݈V°

N Methenamine

CONCEPTUAL CHECKPOINT

/…ˆÃÊVœ“«œÕ˜`ʈÃÊ>ʘˆÌÀœ}i˜Ê>˜>œ}ÕiʜvÊ>˜Ê>ViÌ>°Ê/…>ÌʈÃ]Êi>V…Ê V>ÀLœ˜Ê>̜“ʈÃÊVœ˜˜iVÌi`Ê̜ÊÌܜʘˆÌÀœ}i˜Ê>̜“Ã]ÊÛiÀÞʓÕV…ʏˆŽiÊ >˜Ê >ViÌ>Ê ˆ˜Ê ܅ˆV…Ê >Ê V>ÀLœ˜Ê >̜“Ê ˆÃÊ Vœ˜˜iVÌi`Ê ÌœÊ ÌÜœÊ œÝÞ}i˜Ê >̜“ðÊÊV>ÀLœ˜Ê>̜“Ê̅>ÌʈÃÊVœ˜˜iVÌi`Ê̜ÊÌܜʅiÌiÀœ>̜“ÃÊ­"Ê œÀÊ ®ÊV>˜Ê՘`iÀ}œÊ>Vˆ`‡V>Ì>Þâi`ʅÞ`ÀœÞÈÃ\ Z

O

Z H3O+

R

R

R

20.28 ÃÊŜܘÊ>LœÛi]ʓi̅i˜>“ˆ˜iʈÃʅÞ`ÀœÞâi`ʈ˜Ê>µÕiœÕÃÊ >Vˆ`Ê̜ʫÀœ`ÕViÊvœÀ“>`i…Þ`iÊ>˜`Ê>““œ˜ˆ>°Ê À>ÜÊ>ʓiV…>˜ˆÃ“Ê Ŝ܈˜}ÊvœÀ“>̈œ˜Êœvʜ˜iʓœiVՏiʜvÊvœÀ“>`i…Þ`iʭ̅iÊÀi“>ˆ˜ˆ˜}ÊwÛiʓœiVՏiÃʜvÊvœÀ“>`i…Þ`iÊ>ÀiÊi>V…ÊÀii>Ãi`Êۈ>Ê>Êȓˆ>ÀÊ ÃiµÕi˜ViÊ œvÊ ÃÌi«Ã®°Ê /…iÊ Àii>ÃiÊ œvÊ i>V…Ê “œiVՏiÊ œvÊ vœÀ“>`i…Þ`iʈÃÊ`ˆÀiV̏ÞÊ>˜>œ}œÕÃÊ̜Ê̅iʅÞ`ÀœÞÈÃʜvÊ>˜Ê>ViÌ>°Ê/œÊ}iÌÊ ÞœÕÊÃÌ>ÀÌi`]Ê̅iÊwÀÃÌÊÌܜÊÃÌi«ÃÊ>ÀiÊ«ÀœÛˆ`i`ÊLiœÜ\

R

H

N

Z“ O or N

>V…Ê œvÊ Ì…iÊ V>ÀLœ˜Ê >̜“ÃÊ ˆ˜Ê “i̅i˜>“ˆ˜iÊ V>˜Ê LiÊ …Þ`ÀœÞâi`]Ê Àii>Ș}ÊvœÀ“>`i…Þ`i\ N H3O+

N

N

O

+

H

N

N N

N

H

N

/…iʈ“ˆ˜iʓœˆiÌÞʈÃÊÕÃi`ʈ˜Ê̅iÊ`iÛiœ«“i˜Ìʜvʓ>˜ÞÊ«Àœ`ÀÕ}Ã°Ê iÀiÊÜiÊ܈ÊiÝ«œÀiʜ˜iÊÃÕV…ÊiÝ>“«i° /…iÊVœ“«œÕ˜`ÊLiœÜ]ÊL‡>“ˆ˜œLÕÌÞÀˆVÊ>Vˆ`]ʈÃÊ>˜Êˆ“«œÀÌ>˜ÌÊ ˜iÕÀœÌÀ>˜Ã“ˆÌÌiÀ\ O

OH

H

O N

OH

a

H2O

/…iÊ Vœ“«œÕ˜`Ê i݈ÃÌÃÊ «Àˆ“>ÀˆÞÊ ˆ˜Ê ̅ˆÃÊ ˆœ˜ˆVÊ vœÀ“]Ê Ü…ˆV…Ê V>˜˜œÌÊ VÀœÃÃÊ Ì…iÊ ˜œ˜«œ>ÀÊ i˜ÛˆÀœ˜“i˜ÌÊ œvÊ Ì…iÊ Lœœ`‡LÀ>ˆ˜Ê L>ÀÀˆiÀ°Ê *Àœ}>Lˆ`iÊ ˆÃÊ >Ê «Àœ`ÀÕ}Ê `iÀˆÛ>̈ÛiÊ ÕÃi`Ê ÌœÊ ÌÀi>ÌÊ «>̈i˜ÌÃÊ Ü…œÊ i݅ˆLˆÌÊ̅iÊÃޓ«Ìœ“ÃʜvÊ>Ê`iwVˆi˜VÞʜvÊL‡>“ˆ˜œLÕÌÞÀˆVÊ>Vˆ`\

b g

N N

Formaldehyde

Imines as Prodrugs

H

+

+ 4 NH4 H

H2N

N

+

N

+

6

N

H

N

N

O

NH2

F

g-aminobutyric acid

Ê `iwVˆi˜VÞÊ œvÊ Ì…ˆÃÊ Vœ“«œÕ˜`Ê V>˜Ê V>ÕÃiÊ Vœ˜ÛՏȜ˜Ã°Ê `“ˆ˜ˆÃ̇ iÀˆ˜}Ê L‡>“ˆ˜œLÕÌÞÀˆVÊ >Vˆ`Ê `ˆÀiV̏ÞÊ ÌœÊ >Ê «>̈i˜ÌÊ ˆÃÊ ˜œÌÊ >˜Ê ivviV̈ÛiÊ ÌÀi>̓i˜Ì]Ê LiV>ÕÃiÊ Ì…iÊ Vœ“«œÕ˜`Ê `œiÃÊ ˜œÌÊ Ài>`ˆÞÊ VÀœÃÃÊ Ì…iÊ Lœœ`‡LÀ>ˆ˜ÊL>ÀÀˆiÀ°Ê7…ÞʘœÌ¶ÊÌÊ«…ÞȜœ}ˆV>Ê«]Ê̅iÊ>“ˆ˜œÊ}ÀœÕ«Ê ˆÃÊ«ÀœÌœ˜>Ìi`Ê>˜`Ê̅iÊV>ÀLœÝޏˆVÊ>Vˆ`ʓœˆiÌÞʈÃÊ`i«ÀœÌœ˜>Ìi`\ H N+

H H

klein_c20_001-056v1.4.indd 30

O -

O

Progabide

Cl /…iÊ V>ÀLœÝޏˆVÊ >Vˆ`Ê …>ÃÊ Lii˜Ê Vœ˜ÛiÀÌi`Ê ÌœÊ >˜Ê >“ˆ`i]Ê >˜`Ê Ì…iÊ >“ˆ˜œÊ}ÀœÕ«Ê…>ÃÊLii˜ÊVœ˜ÛiÀÌi`ʈ˜ÌœÊ>˜Êˆ“ˆ˜iÊ­…ˆ}…ˆ}…Ìi`®°ÊÌÊ «…ÞȜœ}ˆV>Ê«]Ê̅ˆÃÊVœ“«œÕ˜`Êi݈ÃÌÃÊ«Àˆ“>ÀˆÞÊ>ÃÊ>ʘiÕÌÀ>ÊVœ“«œÕ˜`ʭ՘V…>À}i`®]Ê>˜`ʈÌÊV>˜Ê̅iÀivœÀiÊVÀœÃÃÊ̅iÊLœœ`‡LÀ>ˆ˜ÊL>À-

30/04/10 16.47

20.8

ÀˆiÀ°Ê"˜Viʈ˜Ê̅iÊLÀ>ˆ˜]ʈÌʈÃÊVœ˜ÛiÀÌi`Ê̜ÊL‡>“ˆ˜œLÕÌÞÀˆVÊ>Vˆ`Êۈ>Ê …Þ`ÀœÞÈÃʜvÊ̅iʈ“ˆ˜iÊ>˜`Ê>“ˆ`iʓœˆïiÃ\ OH

31

*Àœ}>Lˆ`iʈÃʍÕÃÌʜ˜iÊiÝ>“«iʈ˜Ê܅ˆV…Ê̅iʈ“ˆ˜iʓœˆiÌÞʅ>ÃÊLii˜Ê ÕÃi`ʈ˜Ê̅iÊ`iÛiœ«“i˜ÌʜvÊ>Ê«Àœ`ÀÕ}°

O N

F

Sulfur Nucleophiles

NH2

hydrolysis

O H2N

OH

Cl

20.8 Sulfur Nucleophiles In acidic conditions, an aldehyde or ketone will react with two equivalents of a thiol to form a thioacetal: O

[H+]

+ 2 RSH

RS

SR + H2O

Thioacetal

The mechanism of this transformation is directly analogous to acetal formation, with sulfur atoms taking the place of oxygen atoms. If a compound with two SH groups is used, a cyclic thioacetal is formed: O +

S

[H+]

HS

S

SH

+ H2O

Cyclic thioacetal

When treated with Raney nickel, thioacetals undergo desulfurization, yielding an alkane:

S

S

R

R

H

Raney Ni

H

R

R

Raney Ni is a spongy form of nickel that has adsorbed hydrogen atoms. It is these hydrogen atoms that ultimately replace the sulfur atoms, although a discussion of the mechanism for desulfurization is beyond the scope of this text. The reactions above provide us with another two-step method for the reduction of a ketone: O

H 1) [H+],HS

H

SH

2) Raney Ni

This method involves formation of the thioacetal followed by desulfurization with Raney nickel. It is the third method we have encountered for achieving this type of transformation. The other two methods are the Clemmensen reduction (Section 19.6) and the Wolff-Kishner reduction (Section 20.6).

klein_c20_001-056v1.4.indd 31

30/04/10 16.47

32

CHAPTER 20

Aldehydes and Ketones

CONCEPTUAL CHECKPOINT 20.29 *Ài`ˆVÌÊ̅iʓ>œÀÊ«Àœ`ÕVÌÊvœÀÊi>V…ÊÀi>V̈œ˜ÊLiœÜ\

20.30 À>ÜÊ̅iÊÃÌÀÕVÌÕÀiʜvÊ̅iÊVÞVˆVÊVœ“«œÕ˜`Ê̅>ÌʈÃÊ«Àœ`ÕVi`Ê܅i˜Ê>Vi̜˜iʈÃÊÌÀi>Ìi`Ê܈̅ʣ]·«Àœ«>˜i`ˆÌ…ˆœÊˆ˜Ê̅iÊ «ÀiÃi˜ViʜvÊ>˜Ê>Vˆ`ÊV>Ì>ÞÃÌ°

O 1) [H+], HS

SH

2) Raney Ni

(a)

?

O HS Acetone

O 1) [H+], HS

H

2) Raney Ni

(b)

SH

SH

1,3-propanedithiol

?

20.9 Hydrogen Nucleophiles When treated with a hydride reducing agent, such as LAH or sodium borohydride (NaBH4), aldehydes and ketones are reduced to alcohols: 1) LAH 2) H2O

O R

OH

R

R

R

NaBH4, MeOH

These reactions were discussed in Section 13.4, and we saw that LAH and NaBH4 both function as delivery agents of hydride (H−). The precise mechanism of action for these reagents has been heavily investigated and is somewhat complex. Nevertheless, the simplified version shown in Mechanism 20.9 will be sufficient for our purposes.

MECHANISM 20.9 THE REDUCTION OF KETONES OR ALDEHYDES WITH HYDRIDE AGENTS Nucleophilic attack

Proton transfer -

O

O

H

R

R H

H

-

Al

H

R Lithium aluminium hydride (LAH ) functions as a delivery agent of hydride ions (H –)

H

O

O

H

H

R

R The resulting tetrahedral intermediate is protonated to form an alcohol

H

R

H

In the first step of the mechanism, the reducing agent delivers a hydride ion, which attacks the carbonyl group, producing a tetrahedral intermediate. This intermediate is then treated with a proton source to yield the product. This simplified mechanism does not take into account many important observations, such as the role of the lithium cation (Li+). For example, when

klein_c20_001-056v1.4.indd 32

30/04/10 16.47

20.10

LOOKING BACK

33

Carbon Nucleophiles

12-crown-4 is added to the reaction mixture, the lithium ions are solvated (as described in Section 14.4), and reduction does not occur. Clearly, the lithium cation plays a pivotal role in the mechanism. However, a full treatment of the mechanism of hydride reducing agents is beyond the scope of this text, and the simplified version above will suffice. The reduction of a carbonyl group with LAH or NaBH4 is not a reversible process, because hydride does not function as a leaving group. Notice that the mechanism above employs oneway arrows (rather than equilibrium arrows) to signify that the reverse process is insignificant.

Þ`Àˆ`iÊV>˜˜œÌÊv՘V̈œ˜Ê>ÃÊ >ʏi>ۈ˜}Ê}ÀœÕ«ÊLiV>ÕÃiÊ ˆÌʈÃÊ̜œÊÃÌÀœ˜}ÞÊL>ÈV°Ê ­-iiÊ-iV̈œ˜ÊÇ°n°®

CONCEPTUAL CHECKPOINT 20.31 *Ài`ˆVÌÊ Ì…iÊ “>œÀÊ «Àœ`ÕVÌÊ vœÀÊ i>V…Ê œvÊ Ì…iÊ vœœÜˆ˜}Ê Ài>V̈œ˜Ã\ O

1) LAH

2) H2O

(a)

?

20.32 7…i˜Ê ÓÊ “œiÃÊ œvÊ Li˜â>`i…Þ`iÊ >ÀiÊ ÌÀi>Ìi`Ê ÜˆÌ…Ê Ãœ`ˆÕ“Ê…Þ`ÀœÝˆ`i]Ê>ÊÀi>V̈œ˜ÊœVVÕÀÃʈ˜Ê܅ˆV…ʣʓœiʜvÊLi˜â>`i…Þ`iʈÃʜ݈`ˆâi`Ê­}ˆÛˆ˜}ÊLi˜âœˆVÊ>Vˆ`®Ê܅ˆiÊ̅iʜ̅iÀʓœiʜvÊ Li˜â>`i…Þ`iʈÃÊÀi`ÕVi`Ê­}ˆÛˆ˜}ÊLi˜âޏÊ>Vœ…œ®\ O

O H

O

OH H

OH

1) NaOH

H

2) H3O+

H

NaBH4,

MeOH

(b)

?

/…ˆÃÊÀi>V̈œ˜]ÊV>i`Ê̅iÊ >˜˜ˆââ>ÀœÊÀi>V̈œ˜]ʈÃÊLiˆiÛi`ÊÌœÊ œVVÕÀÊۈ>Ê̅iÊvœœÜˆ˜}ʓiV…>˜ˆÃ“\ÊʅÞ`ÀœÝˆ`iʈœ˜ÊÃiÀÛiÃÊ>ÃÊ >ʘÕViœ«…ˆiÊ̜Ê>ÌÌ>VŽÊ̅iÊV>ÀLœ˜ÞÊ}ÀœÕ«ÊœvÊLi˜â>`i…Þ`i]Ê }i˜iÀ>̈˜}Ê>ÊÌiÌÀ>…i`À>Êˆ˜ÌiÀ“i`ˆ>Ìi°Ê/…ˆÃÊÌiÌÀ>…i`À>Êˆ˜ÌiÀ“i`ˆ>ÌiÊ̅i˜Êv՘V̈œ˜ÃÊ>ÃÊ>ʅÞ`Àˆ`iÊÀi`ÕVˆ˜}Ê>}i˜ÌÊLÞÊ`iˆÛiÀˆ˜}Ê>ʅÞ`Àˆ`iʈœ˜Ê̜Ê>˜œÌ…iÀʓœiVՏiʜvÊLi˜â>`i…Þ`i°Ê˜Ê̅ˆÃÊ Ü>Þ]ʜ˜iʓœiVՏiʈÃÊÀi`ÕVi`Ê܅ˆiÊ̅iʜ̅iÀʈÃʜ݈`ˆâi`°

O 1) LAH

2) H2O

(c)

?

­>® 1Ș}Ê̅iÊiÝ«>˜>̈œ˜Ê>LœÛi]Ê`À>ÜÊ̅iʓiV…>˜ˆÃ“ÊœvÊ̅iÊ

>˜˜ˆââ>ÀœÊÀi>V̈œ˜°

O 1) NaBH4

2) MeOH

(d)

­L® 7…>ÌʈÃÊ̅iÊv՘V̈œ˜ÊœvÊÎ"+ʈ˜Ê̅iÊÃiVœ˜`ÊÃÌi«¶

?

­V® 7>ÌiÀÊ>œ˜iʈÃʘœÌÊÃÕvwVˆi˜ÌÊ̜Ê>VVœ“«ˆÃ…Ê̅iÊv՘V̈œ˜ÊœvÊ Ì…iÊÃiVœ˜`ÊÃÌi«°Ê Ý«>ˆ˜°

20.10 Carbon Nucleophiles Grignard Reagents When treated with a Grignard reagent, aldehydes and ketones are converted into alcohols, accompanied by the formation of a new C–C bond: O

H3C

OH O

OH 1) CH3MgBr

1) CH3MgBr 2) H2O

H

2) H2O

H

CH3

Grignard reactions were discussed in more detail in Section 13.6. The precise mechanism of action for these reagents has been heavily investigated and is fairly complex. The simplified version shown in Mechanism 20.10 will be sufficient for our purposes.

klein_c20_001-056v1.4.indd 33

30/04/10 16.47

34

CHAPTER 20

Aldehydes and Ketones

MECHANISM 20.10 THE REACTION BETWEEN A GRIGNARD REAGENT AND A KETONE OR ALDEHYDE Nucleophilic attack

Proton transfer -

O

O

H

R

R

R -

R

R

O

H

H

R

R

The Grignard reagent functions as a nucleophile and attacks the carbonyl group

LOOKING BACK

O

R

R

The resulting tetrahedral intermediate is protonated to form an alcohol

Grignard reactions are not reversible because carbanions generally do not function as leaving groups. Notice that the mechanism above employs one-way arrows (rather than equilibrium arrows) to signify that the reverse process is insignificant.

>ÀL>˜ˆœ˜ÃÊÀ>ÀiÞÊv՘V̈œ˜Ê >Ãʏi>ۈ˜}Ê}ÀœÕ«ÃÊLiV>ÕÃiÊ Ì…iÞÊ>ÀiÊ}i˜iÀ>ÞÊÃÌÀœ˜}ÞÊ L>ÈV°Ê­-iiÊ-iV̈œ˜ÊÇ°n°®

CONCEPTUAL CHECKPOINT 20.33 *Ài`ˆVÌÊ̅iʓ>œÀÊ«Àœ`ÕVÌʜvÊi>V…ÊÀi>V̈œ˜ÊLiœÜ\ O

20.34 `i˜ÌˆvÞÊ̅iÊÀi>}i˜ÌÃʘiViÃÃ>ÀÞÊ̜Ê>VVœ“«ˆÃ…Êi>V…ÊœvÊ Ì…iÊÌÀ>˜ÃvœÀ“>̈œ˜ÃÊLiœÜ\ OH

2) H2O

(a)

Me

?

1) EtMgBr

OH

(a) O H

1) PhMgBr 2) H2O

(b)

OH

?

OH

(b)

O

O

O

(c)

1) PhMgBr 2) H3O+

? Cyannohydrin Formation When treated with hydrogen cyanide (HCN), aldehydes and ketones are converted into cyanohydrins, which are characterized by the presence of a cyano group and a hydroxyl group connected to the same carbon atom: O

HCN

HO CN

A cyanohydrin

This reaction was studied extensively by Arthur Lapworth (University of Manchester) and was found to occur more rapidly in mildly basic conditions. In the presence of a catalytic amount of base, a small amount of hydrogen cyanide is deprotonated to give cyanide ions, which catalyze the reaction (Mechanism 20.11).

klein_c20_001-056v1.4.indd 34

30/04/10 16.47

20.10

Carbon Nucleophiles

35

MECHANISM 20.11 CYANOHYDRIN FORMATION Nucleophilic attack

Proton transfer -

O

C

O

OH

N

H C

N

CN The cyanide ion functions as a nucleophile and attacks the carbonyl group, forming a tetrahedral intermediate

CN

The tetrahedral intermediate is protonated, generating a cyanohydrin

In the first step, a cyanide ion attacks the carbonyl to produce a tetrahedral intermediate. This intermediate then abstracts a proton from HCN, regenerating a cyanide ion. In this way, cyanide functions as a catalyst for the addition of HCN to the carbonyl group. Rather than using a catalytic amount of base to form cyanide ions, the reaction can simply be performed in a mixture of HCN and cyanide ions (from KCN). The process is reversible, and the yield of products is therefore determined by equilibrium concentrations. For most aldehydes and unhindered ketones, the equilibrium favors formation of the cyanohydrin: O

HO CN

KCN, HCN

H3C

CH3

R

R 78%

O

HO CN H

H

KCN, HCN

88%

HCN is a liquid at room temperature and is extremely hazardous to handle because it is highly toxic and volatile (b.p. = 26°C). To avoid the dangers associated with handling HCN, cyanohydrins can also be prepared by treating a ketone or aldehyde with potassium cyanide and an alternate source of protons, such as HCl: O

KCN, HCl

HO CN

Cyanohydrins are useful in syntheses, because the cyano group can be further treated to yield a range of products. Two examples are shown below: N HO

1) LAH

C

2) H2O

R

H

HO R

NH2 H O

H3O

+

HO

C OH

heat

R

H

In the first example, the cyano group is reduced to an amino group. In the second example, the cyano group is hydrolyzed to give a carboxylic acid. Both of these reactions and their mechanisms will be explored in more detail in the next chapter.

klein_c20_001-056v1.4.indd 35

30/04/10 16.47

36

CHAPTER 20

Aldehydes and Ketones

CONCEPTUAL CHECKPOINT 20.35 *Ài`ˆVÌÊ̅iʓ>œÀÊ«Àœ`ÕVÌÊvœÀÊi>V…ÊÀi>V̈œ˜ÊLiœÜ\ O

1) KCN, HCN 2) LAH

3) H2O

(a)

20.36 `i˜ÌˆvÞÊ̅iÊÀi>}i˜ÌÃʘiViÃÃ>ÀÞÊ̜Ê>VVœ“«ˆÃ…Êi>V…ÊœvÊ Ì…iÊÌÀ>˜ÃvœÀ“>̈œ˜ÃÊLiœÜ\ OH

?

OH

O OH

(a) O

OH H

1) KCN, HCl

2) H3O+, heat

OH

?

NH2

(b)

(b)

PRACTICALLYSPEAKING Cyanohydrin Derivatives in Nature “Þ}`>ˆ˜ÊˆÃÊ>ʘ>ÌÕÀ>ÞʜVVÕÀÀˆ˜}ÊVœ“«œÕ˜`ÊvœÕ˜`ʈ˜Ê̅iÊ«ˆÌÃʜvÊ >«ÀˆVœÌÃ]Ê܈`ÊV…iÀÀˆiÃ]Ê>˜`Ê«i>V…ið vʈ˜}iÃÌi`]Ê̅ˆÃÊVœ“«œÕ˜`ʈÃʓiÌ>Lœˆâi`Ê̜ʫÀœ`ÕViʓ>˜`iœ˜ˆÌÀˆi]Ê >Ê VÞ>˜œ…Þ`Àˆ˜]Ê Ü…ˆV…Ê ˆÃÊ Vœ˜ÛiÀÌi`Ê LÞÊ i˜âޓiÃÊ ˆ˜ÌœÊ Li˜â>`i…Þ`iÊ>˜`Ê Ê}>Ã]Ê>Ê̜݈VÊVœ“«œÕ˜`\ OH HO OH

O

HO

O

HO OH

/…ˆÃʏ>ÃÌÊÃÌi«Ê­}i˜iÀ>̈œ˜ÊœvÊ Ê}>îʈÃÊÕÃi`Ê>ÃÊ>Ê`ivi˜ÃiʓiV…>˜ˆÃ“ÊLÞʓ>˜ÞÊëiVˆiÃʜvʓˆˆ«i`iðÊ/…iʓˆˆ«i`iÃʓ>˜Õv>VÌÕÀiÊ >˜`Ê Ã̜ÀiÊ “>˜`iœ˜ˆÌÀˆi]Ê >˜`Ê ˆ˜Ê >Ê Ãi«>À>ÌiÊ Vœ“«>À̓i˜Ì]Ê Ì…iÞÊ Ã̜ÀiÊ i˜âޓiÃÊ Ì…>ÌÊ >ÀiÊ V>«>LiÊ œvÊ V>Ì>Þ∘}Ê Ì…iÊ Vœ˜ÛiÀȜ˜Ê œvÊ “>˜`iœ˜ˆÌÀˆiʈ˜ÌœÊLi˜â>`i…Þ`iÊ>˜`Ê °Ê/œÊÜ>À`ʜvvÊ«Ài`>̜ÀÃ]Ê>ʓˆˆ«i`iÊ܈Ê“ˆÝÊ̅iÊVœ˜Ìi˜ÌÃʜvÊ̅iÊÌܜÊVœ“«>À̓i˜ÌÃÊ >˜`ÊÃiVÀiÌiÊ Ê}>ð

OH

O O

OH C

O C

H

N

N

+

HCN Toxic

Amygdalin

Mandelonitrile

Benzaldehyde

Wittig Reaction Georg Wittig, a German chemist, was awarded the 1979 Nobel Prize in Chemistry for his work with phosphorous compounds and his discovery of a reaction with enormous synthetic utility. Below is an example of this reaction, called the Wittig reaction (pronounced Vittig): O

H Ph -+ C P Ph H Ph H

C

H

This reaction can be used to convert a ketone into an alkene by forming a new C–C bond at the location of the carbonyl moiety. The phosphorus-containing reagent that accomplishes this transformation is called a phosphorane, and it belongs to a larger class of compounds called ylides. An ylide is a compound with two oppositely charged atoms adjacent to each

klein_c20_001-056v1.4.indd 36

30/04/10 16.47

20.10

37

Carbon Nucleophiles

other. The phosphorane above exhibits a negative charge on the carbon atom and a positive charge on the phosphorous atom. This ylide does, in fact, have a resonance structure that is free of any charges: H -C

+

P

H

Ph

H

Ph Ph

H

Ph C

P

Ph Ph

However, this resonance structure (with a C5P double bond) does not contribute much character to the overall resonance hybrid, because the p orbitals on C and P are vastly different in size and do not effectively overlap. A similar argument was used in describing S5O bonds in the previous chapter (Section 19.3). Despite this fact, the phosphorus ylide above, also called a Wittig reagent, is often drawn using either of the resonance structures shown above. A mechanism for the Wittig reaction is shown in Mechanism 20.12.

MECHANISM 20.12 THE WITTIG REACTION Nucleophilic attack H

O

-+

Ph

C P Ph H Ph

Nucleophilic attack

-

Rearrangement

Ph

O

Ph C + Ph P

BY THE WAY

Ý«iÀˆ“i˜Ì>Êiۈ`i˜ViÊ ÃÕ}}iÃÌÃÊ̅>ÌÊ̅iÊ ˆ˜ÌiÀ“i`ˆ>ÌiÊLiÌ>ˆ˜iʈÃʜ˜ÞÊ vœÀ“i`ʈ˜Êˆ“ˆÌi`ÊV>ÃiðÊ˜Ê œÌ…iÀÊV>ÃiÃ]ʈÌÊ>««i>ÀÃÊ̅>ÌÊ Ì…iÊ7ˆÌ̈}ÊÀi>}i˜Ìʓ>ÞÊÀi>VÌÊ ÜˆÌ…Ê̅iÊV>ÀLœ˜ÞÊVœ“«œÕ˜`Ê ˆ˜Ê>ÊQÓ³ÓRÊVÞVœ>``ˆÌˆœ˜Ê «ÀœViÃÃ]Ê`ˆÀiV̏ÞÊ}i˜iÀ>̈˜}Ê Ì…iʜÝ>«…œÃ«…iÌ>˜i°Ê/…iÊ “iV…>˜ˆÃ“ÊvœÀÊ̅ˆÃÊÀi>V̈œ˜Ê ˆÃÊÃ̈Ê՘`iÀʈ˜ÛiÃ̈}>̈œ˜°

P

Ph Ph

C

H

O

+ Ph

P



+

Ph

Ph

H H An Oxaphosphetane

A lone pair on the oxygen atom functions as a nucleophile and attacks the phosphorus atom in an intramolecular attack

The oxaphosphetane decomposes to produce an alkene and triphenylphosphine oxide

The Wittig reagent is a carbanion and can attack the carbonyl group in the first step of the mechanism, generating an intermediate called a betaine (pronounced “bay-tuh-een”). A betaine is a neutral compound with two oppositely charged atoms that are not adjacent to each other. The negatively charged oxygen atom then attacks the positively charged phosphorous atom in an intramolecular nucleophilic attack, generating an oxaphosphetane. This compound then rearranges to give the alkene product. Wittig reagents are easily prepared by treating triphenylphosphine with an alkyl halide followed by a strong base: Ph Ph

P Ph

Triphenylphosphine

klein_c20_001-056v1.4.indd 37

H

C

H H A Betaine The Wittig reagent functions as a nucleophile and attacks the carbonyl group, forming a tetrahedral intermediate

Ph

O

1) CH3I 2) BuLi

Ph Ph Ph

+-

P

H

C

H

Wittig reagent

30/04/10 16.47

38

CHAPTER 20

Aldehydes and Ketones

The mechanism of formation for Wittig reagents involves an SN2 reaction followed by deprotonation: Ph Ph

Ph

H3C I

P

Ph

+

P

Ph

Ph

H C

-

H

+

Ph

CH3CH2CH2CH2 Li

Ph

+-

P

C

Ph

H

H H

Triphenylphosphine

Since the first step is an SN2 process, the regular restrictions of SN2 processes apply. Specifically, primary alkyl halides will react more readily than secondary alkyl halides, and tertiary alkyl halides cannot be used. The Wittig reaction is useful for preparing mono-, di-, or trisubstituted alkenes. Tetrasubstituted alkenes are more difficult to prepare due to steric hindrance in the transition states. The following exercise illustrates how to choose the reagents for a Wittig reaction.

SKILLBUILDER 20.6

PLANNING AN ALKENE SYNTHESIS WITH A WITTIG REACTION

LEARN the skill

`i˜ÌˆvÞÊ̅iÊÀi>}i˜ÌÃʘiViÃÃ>ÀÞÊ̜ʫÀi«>ÀiÊ̅iÊVœ“«œÕ˜`ÊLiœÜÊÕȘ}Ê>Ê7ˆÌ̈}ÊÀi>V̈œ˜\

SOLUTION i}ˆ˜ÊLÞÊvœVÕȘ}ʜ˜Ê̅iÊÌܜÊV>ÀLœ˜Ê>̜“ÃʜvÊ̅iÊ`œÕLiÊLœ˜`°Ê"˜iÊV>ÀLœ˜Ê>̜“Ê“ÕÃÌÊ …>ÛiÊLii˜Ê>ÊV>ÀLœ˜ÞÊ}ÀœÕ«]Ê܅ˆiÊ̅iʜ̅iÀʓÕÃÌʅ>ÛiÊLii˜Ê>Ê7ˆÌ̈}ÊÀi>}i˜Ì°Ê/…ˆÃÊ}ˆÛiÃÊ ÌܜʫœÌi˜Ìˆ>ÊÀœÕÌiÃÊ̜ÊiÝ«œÀi\ STEP 1 1Ș}Ê>ÊÀiÌÀœÃޘ̅ïVÊ >˜>ÞÈÃ]Ê`iÌiÀ“ˆ˜iÊ̅iÊ ÌܜʫœÃÈLiÊÃiÌÃʜvÊ Ài>VÌ>˜ÌÃÊ̅>ÌÊVœÕ`ÊLiÊ ÕÃi`Ê̜ÊvœÀ“Ê̅iÊ 5 Ê Lœ˜`°

H

H O

O

Ph3P

PPh3

+

Method 1

+

Method 2

i̽ÃÊVœ“«>ÀiÊ̅iÃiÊÌܜʓi̅œ`ÃÊLÞÊvœVÕȘ}ʜ˜Ê̅iÊ7ˆÌ̈}ÊÀi>}i˜Ìʈ˜Êi>V…ÊV>Ãi°Ê,iV>Ê̅>ÌÊ Ì…iÊ7ˆÌ̈}ÊÀi>}i˜ÌʈÃÊ«Ài«>Ài`ÊLÞÊ>˜Ê-NÓÊ«ÀœViÃÃ]Ê>˜`ÊÜiÊ̅iÀivœÀiʓÕÃÌÊVœ˜Ãˆ`iÀÊÃÌiÀˆVÊv>V̜ÀÃÊ`ÕÀˆ˜}ʈÌÃÊ«Ài«>À>̈œ˜°Êi̅œ`Ê£ÊÀiµÕˆÀiÃÊ̅iÊÕÃiʜvÊ>ÊÃiVœ˜`>ÀÞÊ>ŽÞÊ…>ˆ`i\ X 1) PPh3

Ph3P

2) BuLi

2° Alkyl halide

klein_c20_001-056v1.4.indd 38

30/04/10 16.47

20.10

Carbon Nucleophiles

39

LÕÌʓi̅œ`ÊÓÊÀiµÕˆÀiÃÊ̅iÊÕÃiʜvÊ>Ê«Àˆ“>ÀÞÊ>ŽÞÊ…>ˆ`i\ H

STEP 2

œ˜Ãˆ`iÀʅœÜÊޜÕÊ ÜœÕ`ʓ>ŽiÊi>V…Ê «œÃÈLiÊ7ˆÌ̈}ÊÀi>}i˜Ì]Ê >˜`Ê`iÌiÀ“ˆ˜iÊ܅ˆV…Ê “i̅œ`ʈ˜ÛœÛiÃÊ̅iʏiÃÃÊ ÃÕLÃ̈ÌÕÌi`Ê>ŽÞÊ…>ˆ`i°

PPh3

1) PPh3

X

2) BuLi

1° Alkyl halide

i̅œ`Ê ÓÊ ˆÃÊ ˆŽiÞÊ ÌœÊ LiÊ “œÀiÊ ivwVˆi˜Ì]Ê LiV>ÕÃiÊ >Ê «Àˆ“>ÀÞÊ >ŽÞÊ …>ˆ`iÊ ÜˆÊ ՘`iÀ}œÊ -NÓÊ “œÀiÊÀ>«ˆ`ÞÊ̅>˜Ê>ÊÃiVœ˜`>ÀÞÊ>ŽÞÊ…>ˆ`i°Ê/…iÀivœÀi]Ê̅iÊvœœÜˆ˜}ÊܜՏ`ÊLiÊ̅iÊ«ÀiviÀÀi`Ê Ãޘ̅iÈÃ\ O

-

+

P Ph Ph Ph

PRACTICE the skill 20.37 `i˜ÌˆvÞÊ̅iÊÀi>}i˜ÌÃʘiViÃÃ>ÀÞÊ̜ʫÀi«>ÀiÊi>V…ÊœvÊ̅iÊvœœÜˆ˜}ÊVœ“«œÕ˜`ÃÊÕȘ}Ê >Ê7ˆÌ̈}ÊÀi>V̈œ˜\

APPLY the skill

(a)

(b)

(d)

(e)

20.38

(c)

œ˜Ãˆ`iÀÊ̅iÊÃÌÀÕVÌÕÀiʜvÊLiÌ>‡V>ÀœÌi˜i]ʓi˜Ìˆœ˜i`Êi>ÀˆiÀʈ˜Ê̅ˆÃÊV…>«ÌiÀ\

b-carotene

iÈ}˜Ê>ÊÃޘ̅iÈÃʜvÊLiÌ>ÊV>ÀœÌi˜iÊÕȘ}Ê̅iÊVœ“«œÕ˜`ÊLiœÜÊ>ÃÊޜÕÀʜ˜ÞÊÜÕÀViʜvÊV>ÀLœ˜Ê>̜“Ã\

Br

20.39

`i˜ÌˆvÞÊ̅iÊÀi>}i˜ÌÃʘiViÃÃ>ÀÞÊ̜Ê>VVœ“«ˆÃ…Êi>V…ÊœvÊ̅iÊÌÀ>˜ÃvœÀ“>̈œ˜ÃÊLiœÜ\ OH

(a)

(b)

need more PRACTICE? Try Problems 20.51–20.53

klein_c20_001-056v1.4.indd 39

30/04/10 16.47

40

CHAPTER 20

Aldehydes and Ketones

20.11 Baeyer-Villiger Oxidation of Aldehydes and Ketones O

When treated with a peroxy acid, ketones can be converted into esters via the insertion of an oxygen atom:

O RCO3H

R

R

R

O

R

This reaction, discovered by Adolf von Baeyer and Victor Villiger in 1899, is called the BaeyerVilliger oxidation. This process is believed to proceed via Mechanism 20.13.

MECHANISM 20.13 THE BAEYER-VILLIGER OXIDATION Nucleophilic attack H

O

O R

-

O O

R

-

O R R

The peroxyacid functions as a nucleophile and attacks the carbonyl group, forming a tetrahedral intermediate

O

Proton transfer +

O H

Rearrangement

R

O

O

R

O

A proton is transferred from one location to another. This step can occur intramolecularly, because it would involve a five-membered transition state

O

O

O O

+

H

The carbonyl group is reformed, with simultaneous migration of an alkyl group

R

O

+ OH

R

The peroxy acid attacks the carbonyl group of the ketone, giving a tetrahedral intermediate that then undergoes an intramolecular proton transfer (or two successive intermolecular proton transfers). Finally, the C5O double bond is re-formed by migration of an R group. This rearrangement produces the ester. In much the same way, treatment of a cyclic ketone with a peroxy acid yields a cyclic ester, or lactone. O

O RCO3H

O

A lactone

When an unsymmetrical ketone is treated with a peroxy acid, formation of the ester is regioselective; for example: O O RCO3H

O

In this case, the oxygen atom is inserted on the left side of the carbonyl group, rather than the right side. This occurs because the isopropyl group migrates more rapidly than the methyl group during the rearrangement step of the mechanism. The migration rates of different groups, or migratory aptitude, can be summarized as follows: H > 3° > 2°, Ph > 1° > methyl

klein_c20_001-056v1.4.indd 40

30/04/10 16.47

20.12

Synthesis Strategies

41

A hydrogen atom will migrate more rapidly than a tertiary alkyl group, which will migrate more rapidly than a secondary alkyl group or phenyl group. Below is one more example that illustrates this concept: O

O H

O

RCO3H

H

In this example, the oxygen atom is inserted on the right side of the carbonyl, because the hydrogen atom exhibits a greater migratory aptitude than the phenyl group.

CONCEPTUAL CHECKPOINT 20.40 *Ài`ˆVÌÊ̅iʓ>œÀÊ«Àœ`ÕVÌʜvÊi>V…ÊÀi>V̈œ˜ÊLiœÜ\ O

O RCO3H

?

(a)

O H

(b)

RCO3H

?

RCO3H

?

(c)

20.12 Synthesis Strategies Recall from Chapter 12 that there are two main questions to ask when approaching a synthesis problem: 1. 2.

Is there any change in the carbon skeleton? Is there any change in the functional group?

Let’s focus on these issues separately, beginning with functional groups.

Functional Group Interconversion In previous chapters, we learned how to interconvert many different functional groups (Figure 20.9). The reactions in this chapter expand the playing field by opening up the frontier of aldehydes and ketones. You should be able to fill in the reagents for each transformation in Figure 20.9. If you are having trouble, refer to Figure 13.13 for help. Then, you should be able to make a list of the various products than can be made from aldehydes and ketones and identify the required reagents in each case.

Reactions covered in this chapter

O

OH

X

FIGURE 20.9 ՘V̈œ˜>Ê}ÀœÕ«ÃÊ̅>ÌÊV>˜ÊLiÊ ˆ˜ÌiÀVœ˜ÛiÀÌi`ÊÕȘ}ÊÀi>V̈œ˜ÃÊ Ì…>ÌÊÜiʅ>Ûiʏi>À˜i`Ê̅ÕÃÊv>À°

klein_c20_001-056v1.4.indd 41

30/04/10 16.47

42

CHAPTER 20

Aldehydes and Ketones

Reactions Involving a Change in Carbon Skeleton In this chapter, we have seen three C–C bond-forming reactions: (1) a Grignard reaction, (2) cyanohydrin formation, and (3) a Wittig reaction: O C

H

1) CH3MgBr 2) H2O

+

N

H C

C

Ph

Ph

H

OH

H3C



C P Ph

KCN, HCN

H C

OH C

C

We have only seen one C–C bond-breaking reaction: the Baeyer-Villiger oxidation: O C

O RCO3H

C

C

C O

These four reactions should be added to your list of reactions that can change a carbon skeleton. Let’s get some practice using these reactions.

SKILLBUILDER 20.7

PROPOSING A SYNTHESIS

LEARN the skill

*Àœ«œÃiÊ>˜ÊivwVˆi˜ÌÊÃޘ̅iÈÃÊvœÀÊ̅iÊvœœÜˆ˜}ÊÊ ÌÀ>˜ÃvœÀ“>̈œ˜\

SOLUTION STEP 1 ˜Ã«iVÌÊ܅i̅iÀÊ̅iÀiÊ ˆÃÊ>ÊV…>˜}iʈ˜Ê̅iÊ V>ÀLœ˜ÊÎii̜˜Ê>˜`ɜÀÊ >ÊV…>˜}iʈ˜Ê̅iʈ`i˜ÌˆÌÞÊ œÀʏœV>̈œ˜ÊœvÊ̅iÊ v՘V̈œ˜>Ê}ÀœÕ«Ã°

Ü>ÞÃÊLi}ˆ˜Ê>ÊÃޘ̅iÈÃÊ«ÀœLi“ÊLÞÊ>Έ˜}Ê̅iÊvœœÜˆ˜}ÊÌܜʵÕiÃ̈œ˜Ã\ 1. Is there any change in the carbon skeleton?Ê9iðÊ/…iÊ«Àœ`ÕVÌʅ>ÃÊÌܜÊ>``ˆÌˆœ˜>ÊV>ÀLœ˜Ê >̜“ð 2. Is there any change in the functional groups?Ê œ°Ê œÌ…Ê̅iÊÃÌ>À̈˜}ʓ>ÌiÀˆ>Ê>˜`Ê̅iÊ «Àœ`ÕVÌʅ>ÛiÊ>Ê`œÕLiÊLœ˜`ʈ˜Ê̅iÊiÝ>VÌÊÃ>“iʏœV>̈œ˜°ÊvÊÜiÊ`iÃÌÀœÞÊ̅iÊ`œÕLiÊLœ˜`ʈ˜Ê ̅iÊ«ÀœViÃÃʜvÊ>``ˆ˜}Ê̅iÊÌܜÊV>ÀLœ˜Ê>̜“Ã]ÊÜiÊ܈Ê˜ii`Ê̜ʓ>ŽiÊÃÕÀiÊ̅>ÌÊÜiÊ`œÊÜʈ˜Ê ÃÕV…Ê>ÊÜ>ÞÊ̅>ÌÊÜiÊV>˜ÊÀiÃ̜ÀiÊ̅iÊ`œÕLiÊLœ˜`° œÜʏi̽ÃÊVœ˜Ãˆ`iÀʅœÜÊÜiʓˆ}…Ìʈ˜ÃÌ>Ê̅iÊ>``ˆÌˆœ˜>ÊÌܜ‡V>ÀLœ˜Ê>̜“Ã°Ê /…iÊvœœÜˆ˜}Ê q ÊLœ˜`ʈÃÊ̅iʜ˜iÊ̅>Ìʘii`ÃÊ̜ÊLiʓ>`i\ ˜Ê̅ˆÃÊV…>«ÌiÀ]ÊÜiʅ>ÛiÊÃii˜Ê̅ÀiiÊ q ÊLœ˜`‡vœÀ“ˆ˜}ÊÀi>V̈œ˜Ã°Êi̽ÃÊVœ˜Ãˆ`iÀÊi>V…Êœ˜iÊ>ÃÊ>Ê«œÃÈLˆˆÌÞ°

klein_c20_001-056v1.4.indd 42

30/04/10 16.47

20.12

STEP 2 7…i˜Ê̅iÀiʈÃÊ>ÊV…>˜}iÊ ˆ˜Ê̅iÊV>ÀLœ˜ÊÎii̜˜]Ê Vœ˜Ãˆ`iÀÊ>ÊœvÊ̅iÊ q Ê Lœ˜`‡vœÀ“ˆ˜}ÊÀi>V̈œ˜ÃÊ >˜`Ê>ÊœvÊ̅iÊ q ÊLœ˜`‡ LÀi>Žˆ˜}ÊÀi>V̈œ˜ÃÊ̅>ÌÊ ÞœÕʅ>Ûiʏi>À˜i`ÊÜÊv>À°

Synthesis Strategies

43

7iÊV>˜Êˆ““i`ˆ>ÌiÞÊÀՏiʜÕÌÊVÞ>˜œ…Þ`Àˆ˜ÊvœÀ“>̈œ˜]Ê>ÃÊ̅>ÌÊ«ÀœViÃÃʈ˜ÃÌ>Ãʜ˜Þʜ˜iÊ V>ÀLœ˜Ê>̜“]ʘœÌÊÌܜ°Ê-œÊi̽ÃÊVœ˜Ãˆ`iÀÊvœÀ“ˆ˜}Ê̅iÊ q ÊLœ˜`Ê܈̅ÊiˆÌ…iÀÊ>ÊÀˆ}˜>À`ÊÀi>V̈œ˜ÊœÀÊ>Ê7ˆÌ̈}ÊÀi>V̈œ˜° Ê Àˆ}˜>À`Ê Ài>}i˜ÌÊ Üœ˜½ÌÊ >ÌÌ>VŽÊ >Ê 5 Ê `œÕLiÊ Lœ˜`]Ê ÃœÊ ÕȘ}Ê >Ê Àˆ}˜>À`Ê Ài>V̈œ˜Ê ܜՏ`ÊÀiµÕˆÀiÊwÀÃÌÊVœ˜ÛiÀ̈˜}Ê̅iÊ 5 Ê`œÕLiÊLœ˜`ʈ˜ÌœÊ>Êv՘V̈œ˜>Ê}ÀœÕ«Ê̅>ÌÊV>˜ÊLiÊ >ÌÌ>VŽi`ÊLÞÊ>ÊÀˆ}˜>À`ÊÀi>}i˜Ì]ÊÃÕV…Ê>ÃÊ>ÊV>ÀLœ˜ÞÊ}ÀœÕ«\ HO H

O H 1) EtMgBr 2) H2O

/…ˆÃÊ Ài>V̈œ˜Ê V>˜Ê ˆ˜`ii`Ê LiÊ ÕÃi`Ê ÌœÊ vœÀ“Ê ̅iÊ VÀÕVˆ>Ê q Ê Lœ˜`°Ê /œÊ ÕÃiÊ Ì…ˆÃÊ “i̅œ`Ê œvÊ

q ÊLœ˜`ÊvœÀ“>̈œ˜]ÊÜiʓÕÃÌÊwÀÃÌÊvœÀ“Ê̅iʘiViÃÃ>ÀÞÊ>`i…Þ`i]Ê̅i˜Ê«iÀvœÀ“Ê̅iÊÀˆ}˜>À`Ê Ài>V̈œ˜]Ê>˜`Ê̅i˜Êw˜>ÞÊÀiÃ̜ÀiÊ̅iÊ`œÕLiÊLœ˜`ʈ˜ÊˆÌÃÊ«Àœ«iÀʏœV>̈œ˜°Ê/…ˆÃÊV>˜ÊLiÊ>VVœ“«ˆÃ…i`Ê܈̅Ê̅iÊvœœÜˆ˜}ÊÀi>}i˜ÌÃ\

1) BH3 # THF 2) H2O2 , NaOH

HO

OH

H

O H

PCC

1) EtMgBr

1) TsCl, py

2) H2O

2) NaOEt, heat

C¬C bond-forming reaction

/…ˆÃÊ«ÀœÛˆ`iÃÊÕÃÊ܈̅Ê>ÊvœÕÀ‡ÃÌi«Ê«ÀœVi`ÕÀi]Ê>˜`Ê̅ˆÃÊ>˜ÃÜiÀʈÃÊViÀÌ>ˆ˜ÞÊÀi>ܘ>Li° i̽ÃʘœÜÊiÝ«œÀiÊ̅iÊ«œÃÈLˆˆÌÞʜvÊ«Àœ«œÃˆ˜}Ê>ÊÃޘ̅iÈÃÊ܈̅Ê>Ê7ˆÌ̈}Ê Ài>V̈œ˜°Ê,iV>Ê̅>ÌÊ>Ê7ˆÌ̈}ÊÀi>V̈œ˜ÊV>˜ÊLiÊÕÃi`Ê̜ÊvœÀ“Ê>Ê 5 ÊLœ˜`]ÊÃœÊ ÜiÊvœVÕÃʜ˜ÊvœÀ“>̈œ˜ÊœvÊ̅ˆÃÊLœ˜`\ /…ˆÃÊLœ˜`ÊV>˜ÊLiÊvœÀ“i`ʈvÊÜiÊÃÌ>ÀÌÊ܈̅Ê>ʎi̜˜iÊ>˜`ÊÕÃiÊ̅iÊvœœÜˆ˜}Ê7ˆÌ̈}Ê Ài>}i˜Ì\ O PPh3

/œÊÕÃiÊ̅ˆÃÊÀi>V̈œ˜]ÊÜiʓÕÃÌÊwÀÃÌÊvœÀ“Ê̅iʘiViÃÃ>ÀÞʎi̜˜iÊvÀœ“Ê̅iÊÃÌ>À̈˜}Ê>Ži˜i\ O

/…ˆÃÊV>˜ÊLiÊ>VVœ“«ˆÃ…i`Ê܈̅ʜ✘œÞÈðÊ/…ˆÃÊ}ˆÛiÃÊ>ÊÌܜ‡ÃÌi«Ê«ÀœVi`ÕÀiÊvœÀÊ>VVœ“«ˆÃ…ˆ˜}Ê̅iÊ`iÈÀi`ÊÌÀ>˜ÃvœÀ“>̈œ˜\ʜ✘œÞÈÃÊvœœÜi`ÊLÞÊ>Ê7ˆÌ̈}ÊÀi>V̈œ˜°Ê/…ˆÃÊ>««Àœ>V…ʈÃÊ `ˆvviÀi˜ÌÊ̅>˜ÊœÕÀÊwÀÃÌÊ>˜ÃÜiÀ°Ê˜Ê̅ˆÃÊ>««Àœ>V…]ÊÜiÊ>ÀiʘœÌÊ>ÌÌ>V…ˆ˜}Ê>ÊÌܜ‡V>ÀLœ˜ÊV…>ˆ˜]ÊLÕÌÊ À>̅iÀ]ÊÜiÊ>ÀiÊwÀÃÌÊiÝ«iˆ˜}Ê>ÊV>ÀLœ˜Ê>̜“Ê>˜`Ê̅i˜Ê>ÌÌ>V…ˆ˜}Ê>Ê̅Àii‡V>ÀLœ˜ÊV…>ˆ˜°

klein_c20_001-056v1.4.indd 43

30/04/10 16.47

44

CHAPTER 20

Aldehydes and Ketones

˜Ê ÃՓ“>ÀÞ]Ê ÜiÊ …>ÛiÊ `ˆÃVœÛiÀi`Ê ÌÜœÊ «>ÕÈLiÊ “i̅œ`Ã°Ê œÌ…Ê “i̅œ`ÃÊ >ÀiÊ VœÀÀiVÌÊ >˜ÃÜiÀÃÊ̜Ê̅ˆÃÊ«ÀœLi“]ÊLÕÌÊ̅iʓi̅œ`Êi“«œÞˆ˜}Ê̅iÊ7ˆÌ̈}ÊÀi>V̈œ˜ÊˆÃʏˆŽiÞÊ̜ÊLiʓœÀiÊ ivwVˆi˜Ì]ÊLiV>ÕÃiʈÌÊÀiµÕˆÀiÃÊviÜiÀÊÃÌi«Ã° 1) BH3 # THF 2) H2O2 , NaOH 3) PCC 4) EtMgBr 5) H2O 6) TsCl, py 7) NaOEt, heat

1) O3 2) DMS 3) Ph3P

PRACTICE the skill 20.41 *Àœ«œÃiÊ>˜ÊivwVˆi˜ÌÊÃޘ̅iÈÃÊvœÀÊi>V…ÊœvÊ̅iÊvœœÜˆ˜}ÊÌÀ>˜ÃvœÀ“>̈œ˜Ã\ OH

(a)

(b) OH OH

(c)

(d)

O

Br

N

(e)

(f) O O

O EtO

OEt

(g) O

O

APPLY the skill

20.42 1Ș}Ê>˜ÞÊVœ“«œÕ˜`ÃʜvÊޜÕÀÊV…œœÃˆ˜}]ʈ`i˜ÌˆvÞÊ>ʓi̅œ`ÊvœÀÊ«Ài«>Àˆ˜}Êi>V…ÊœvÊ Ì…iÊvœœÜˆ˜}ÊVœ“«œÕ˜`ðÊYour only limitation is that the compounds you use can have no more than two carbon atoms°ÊœÀÊ«ÕÀ«œÃiÃʜvÊVœÕ˜Ìˆ˜}ÊV>ÀLœ˜Ê>̜“Ã]ÊޜÕʓ>Þʈ}˜œÀiÊ̅iÊ «…i˜ÞÊ}ÀœÕ«ÃʜvÊ>Ê7ˆÌ̈}ÊÀi>}i˜Ì°Ê/…>ÌʈÃ]ÊޜÕÊ>ÀiÊ«iÀ“ˆÌÌi`Ê̜ÊÕÃiÊ7ˆÌ̈}ÊÀi>}i˜Ìð N

NH

(a)

(b)

O

O

O

(c)

O

(d)

HO N

OH

OH H2 N

HO

(e)

(f)

(g)

O

(h)

need more PRACTICE? Try Problems 20.55, 20.58, 20.67–20.69, 20.71, 20.75

klein_c20_001-056v1.4.indd 44

30/04/10 16.47

20.13

45

Spectroscopic Analysis of Aldehydes and Ketones

20.13 Spectroscopic Analysis of Aldehydes and Ketones Aldehydes and ketones exhibit several characteristic signals in their infrared (IR) and nuclear magnetic resonance (NMR) spectra. We will now summarize these characteristic signals.

IR Signals The carbonyl group produces a strong signal in an IR spectrum, generally around 1715 or 1720 cm−1. However, a conjugated carbonyl will produce a signal at a lower wavenumber as a result of electron delocalization via resonance effects: O

O

LOOKING BACK œÀÊ>˜ÊiÝ«>˜>̈œ˜ÊœvÊ̅ˆÃÊ ivviVÌ]ÊÃiiÊ-iV̈œ˜Ê£x°Î°

1715 cm –1

1680 cm –1

Ring strain has the opposite effect on a carbonyl group. That is, increasing ring strain tends to increase the wavenumber of absorption: O

1715 cm –1

O

O

1745 cm –1

1780 cm –1

Aldehydes generally exhibit one or two signals (C–H stretching) between 2700 and 2850 cm−1 (Figure 20.10).

100

% Transmittance

80

2719 O

60 H 40

1726

20

FIGURE 20.10 ˜Ê,ÊëiVÌÀՓʜvÊ>˜Ê>`i…Þ`i°

klein_c20_001-056v1.4.indd 45

0 4000

3500

3000

2500

2000

1500

1000

Wavenumber (cm⫺1)

30/04/10 16.47

46

CHAPTER 20

Aldehydes and Ketones 1

H NMR Signals

In a 1H NMR spectrum, the carbonyl group itself does not produce a signal. However, it has a pronounced effect on the chemical shift of neighboring protons. We saw in Section 16.5 that a carbonyl group adds +1 ppm to the chemical shift of its neighbors: O R

R

R

R H

H

H

~1.2 ppm

H

~2.2 ppm

Aldehydic protons generally produce signals around 10 ppm. These signals can usually be identified with relative ease, because very few signals appear that far downfield in a 1H NMR spectrum (Figure 20.11).

O H

5

1

FIGURE 20.11 Ê£Ê ,ÊëiVÌÀՓʜvÊ>˜Ê >`i…Þ`i°

2 2

10

9

8

7

6

5

4

3

2

Chemical Shift (ppm)

13

C NMR Signals

In a 13C NMR spectrum, the carbon atom of a carbonyl group will generally produce a weak signal near 200 ppm. This signal can often be identified with relative ease, because very few signals appear that far downfield in a 13C NMR spectrum (Figure 20.12).

O 209.1 FIGURE 20.12 Ê£Î Ê ,ÊëiVÌÀՓʜvÊ>ʎi̜˜i°

220

200

180

160

140

120

100

80

60

40

20

0

Chemical Shift (ppm)

CONCEPTUAL CHECKPOINT 20.43 œ“«œÕ˜`Ê Ê …>ÃÊ “œiVՏ>ÀÊ vœÀ“Տ>Ê £äH£ä"Ê >˜`Ê i݅ˆLˆÌÃÊ >Ê ÃÌÀœ˜}ÊÈ}˜>Ê>ÌÊ£ÇÓäÊV“•£Êˆ˜ÊˆÌÃÊ,ÊëiVÌÀՓ°Ê/Ài>̓i˜ÌÊ܈̅ʣ]Ӈi̅>˜i`ˆÌ…ˆœÊvœœÜi`ÊLÞÊ,>˜iÞʘˆVŽiÊ>vvœÀ`ÃÊ̅iÊ«Àœ`ÕVÌÊŜܘÊLiœÜ°Ê `i˜ÌˆvÞÊ̅iÊÃÌÀÕVÌÕÀiʜvÊVœ“«œÕ˜`Ê°

klein_c20_001-056v1.4.indd 46

Compound A

1) [H+], HS

SH

2) Raney Ni

30/04/10 16.47

47

Review of Concepts and Vocabulary

REVIEW OF REACTIONS

SYNTHETICALLY USEFUL REACTIONS

1. Þ`À>ÌiÊœÀ“>̈œ˜ 2. ViÌ>ÊœÀ“>̈œ˜ 3. ÞVˆVÊViÌ>Ê œÀ“>̈œ˜

HO

O

[H+], H2O

OH

RCO3H

O

15

1

4. ÞVˆVÊ/…ˆœ>Vi̜Ê œÀ“>̈œ˜

H2C

ROH, –H2O

RO

OR

OH

HO

2

O

–H2O

3 [H+] RNH2 –H2O

8. "݈“iÊœÀ“>̈œ˜ S

9. Þ`À>✘iÊœÀ“>̈œ˜ 10. 7œvv‡ˆÃ…˜iÀÊ,i`ÕV̈œ˜

S

4 N

11. ,i`ÕV̈œ˜ÊœvÊ>Êi̜˜i 12. Àˆ}˜>À`Ê,i>V̈œ˜ 14. 7ˆÌ̈}Ê,i>V̈œ˜ 15. >iÞiÀ‡6ˆˆ}iÀÊ"݈`>̈œ˜

[H+] R2NH –H2O

[H+] NH2OH –H2O

[H+] NH2NH2 –H2O

R

N

R N

OH

7

5

OH R

N

NH2

9 8

H

13

12

OH

6

H

CN

1) LAH 2) H2O

R

Raney Ni

13. Þ>˜œ…Þ`Àˆ˜ÊœÀ“>̈œ˜

OH

1) RMgBr 2) H2O

SH

HS

O

14

HCN, KCN [H+]

–H2O

6. “ˆ˜iÊœÀ“>̈œ˜

PPh3

[H+]

5. iÃՏvÕÀˆâ>̈œ˜ 7. ˜>“ˆ˜iÊœÀ“>̈œ˜

O

[H+]

11

85%

NaOH, H2O, heat

10

REVIEW OF CONCEPTS AND VOCABULARY SECTION 20.1 UÊ œÌ…Ê>`i…Þ`iÃÊ>˜`ʎi̜˜iÃÊVœ˜Ì>ˆ˜Ê>Êcarbonyl group]Ê>˜`Ê LœÌ…Ê>ÀiÊVœ““œ˜Êˆ˜Ê˜>ÌÕÀiÊ>˜`ʈ˜`ÕÃÌÀÞÊ>˜`ʜVVÕ«ÞÊ>ÊVi˜ÌÀ>Ê Àœiʈ˜ÊœÀ}>˜ˆVÊV…i“ˆÃÌÀÞ°

SECTION 20.2 UÊ /…iÊÃÕvwÝʺ‡>»Êˆ˜`ˆV>ÌiÃÊ>˜Ê>`i…Þ`ˆVÊ}ÀœÕ«]Ê>˜`Ê̅iÊÃÕvwÝÊ º‡œ˜i»ÊˆÃÊÕÃi`ÊvœÀʎi̜˜ið UÊ ˜Ê˜>“ˆ˜}Ê>`i…Þ`iÃÊ>˜`ʎi̜˜iÃ]ʏœV>˜ÌÃÊŜՏ`ÊLiÊ>ÃÈ}˜i`Ê ÃœÊ>ÃÊ̜Ê}ˆÛiÊ̅iÊV>ÀLœ˜ÞÊ}ÀœÕ«Ê̅iʏœÜiÃÌʘՓLiÀÊ«œÃÈLi°

SECTION 20.3 UÊ `i…Þ`iÃÊV>˜ÊLiÊ«Ài«>Ài`Êۈ>ʜ݈`>̈œ˜ÊœvÊ«Àˆ“>ÀÞÊ>Vœ…œÃ]Ê œâœ˜œÞÈÃʜvÊ>Ži˜iÃ]ʜÀʅÞ`ÀœLœÀ>̈œ˜‡œÝˆ`>̈œ˜ÊœvÊÌiÀ“ˆ˜>Ê >ŽÞ˜ið UÊ i̜˜iÃÊ V>˜Ê LiÊ «Ài«>Ài`Ê Ûˆ>Ê œÝˆ`>̈œ˜Ê œvÊ ÃiVœ˜`>ÀÞÊ >Vœ…œÃ]ʜ✘œÞÈÃʜvÊ>Ži˜iÃ]Ê>Vˆ`‡V>Ì>Þâi`ʅÞ`À>̈œ˜ÊœvÊÌiÀ“ˆ˜>Ê>ŽÞ˜iÃ]ʜÀÊÀˆi`i‡ À>vÌÃÊ>Vޏ>̈œ˜°

SECTION 20.4 UÊ /…iÊ iiVÌÀœ«…ˆˆVˆÌÞÊ œvÊ >Ê V>ÀLœ˜ÞÊ }ÀœÕ«Ê `iÀˆÛiÃÊ vÀœ“Ê Àiܘ>˜ViÊivviVÌÃÊ>ÃÊÜiÊ>Ãʈ˜`ÕV̈ÛiÊivviVÌð UÊ `i…Þ`iÃÊ>ÀiʓœÀiÊÀi>V̈ÛiÊ̅>˜ÊŽi̜˜iÃÊ>ÃÊ>ÊÀiÃՏÌʜvÊÃÌiÀˆVÊ ivviVÌÃÊ>˜`ÊiiVÌÀœ˜ˆVÊivviVÌð UÊ Ê }i˜iÀ>Ê “iV…>˜ˆÃ“Ê vœÀÊ ˜ÕViœ«…ˆˆVÊ >``ˆÌˆœ˜Ê ՘`iÀÊ L>ÈVÊ Vœ˜`ˆÌˆœ˜Ãʈ˜ÛœÛiÃÊÌܜÊÃÌi«Ã\ 1.Ê ÕViœ«…ˆˆVÊ>ÌÌ>VŽÊ̜Ê}i˜iÀ>ÌiÊ>Êtetrahedral intermediate° 2.Ê *ÀœÌœ˜ÊÌÀ>˜ÃviÀ

klein_c20_001-056v1.4.indd 47

UÊ /…iÊ«œÃˆÌˆœ˜ÊœvÊiµÕˆˆLÀˆÕ“ʈÃÊ`i«i˜`i˜Ìʜ˜Ê̅iÊ>LˆˆÌÞʜvÊ̅iÊ ˜ÕViœ«…ˆiÊ̜Êv՘V̈œ˜Ê>ÃÊ>ʏi>ۈ˜}Ê}ÀœÕ«°

SECTION 20.5 UÊ 7…i˜Ê>˜Ê>`i…Þ`iʜÀʎi̜˜iʈÃÊÌÀi>Ìi`Ê܈̅ÊÜ>ÌiÀ]Ê̅iÊV>ÀLœ˜ÞÊ}ÀœÕ«ÊV>˜ÊLiÊVœ˜ÛiÀÌi`ʈ˜ÌœÊ>Êhydrate°Ê/…iÊiµÕˆˆLÀˆÕ“Ê}i˜iÀ>ÞÊv>ۜÀÃÊ̅iÊV>ÀLœ˜ÞÊ}ÀœÕ«]ÊiÝVi«Ìʈ˜Ê̅iÊV>ÃiÊ œvÊ ÛiÀÞÊ Ãˆ“«iÊ >`i…Þ`iÃ]Ê œÀÊ Ži̜˜iÃÊ ÜˆÌ…Ê ÃÌÀœ˜}Ê iiVÌÀœ˜‡ ܈̅`À>܈˜}ÊÃÕLÃ̈ÌÕi˜Ìð UÊ ˜Ê>Vˆ`ˆVÊVœ˜`ˆÌˆœ˜Ã]Ê>˜Ê>`i…Þ`iʜÀʎi̜˜iÊ܈ÊÀi>VÌÊ܈̅ÊÌÜœÊ “œiVՏiÃʜvÊ>Vœ…œÊ̜ÊvœÀ“Ê>˜Êacetal° UÊ ˜Ê̅iÊ«ÀiÃi˜ViʜvÊ>˜Ê>Vˆ`]Ê̅iÊV>ÀLœ˜ÞÊ}ÀœÕ«ÊˆÃÊ«ÀœÌœ˜>Ìi`Ê ÌœÊvœÀ“Ê>ÊÛiÀÞÊ«œÜiÀvՏÊiiVÌÀœ«…ˆi° UÊ /…iʓiV…>˜ˆÃ“ÊvœÀÊ>ViÌ>ÊvœÀ“>̈œ˜ÊV>˜ÊLiÊ`ˆÛˆ`i`ʈ˜ÌœÊÌÜœÊ «>ÀÌÃ\ 1.Ê /…iÊwÀÃÌÊ̅ÀiiÊÃÌi«ÃÊ«Àœ`ÕViÊ>Êhemiacetal° 2.Ê /…iʏ>ÃÌÊvœÕÀÊÃÌi«ÃÊVœ˜ÛiÀÌÊ̅iʅi“ˆ>ViÌ>Ê̜Ê>˜Ê>ViÌ>° UÊ œÀÊ “>˜ÞÊ Ãˆ“«iÊ >`i…Þ`iÃ]Ê Ì…iÊ iµÕˆˆLÀˆÕ“Ê v>ۜÀÃÊ vœÀ“>̈œ˜ÊœvÊ̅iÊ>ViÌ>ÆʅœÜiÛiÀ]ÊvœÀʓœÃÌʎi̜˜iÃ]Ê̅iÊiµÕˆˆLÀˆÕ“Ê v>ۜÀÃÊÀi>VÌ>˜ÌÃÊÀ>̅iÀÊ̅>˜Ê«Àœ`ÕVÌð UÊ ˜Ê>`i…Þ`iʜÀʎi̜˜iÊ܈ÊÀi>VÌÊ܈̅ʜ˜iʓœiVՏiʜvÊ>Ê`ˆœÊ ̜ÊvœÀ“Ê>ÊVÞVˆVÊ>ViÌ>° UÊ /…iÊÀiÛiÀÈLˆˆÌÞʜvÊ>ViÌ>ÊvœÀ“>̈œ˜Êi˜>LiÃÊ>ViÌ>ÃÊ̜Êv՘V̈œ˜Ê>ÃÊ«ÀœÌiV̈˜}Ê}ÀœÕ«ÃÊvœÀʎi̜˜iÃʜÀÊ>`i…Þ`iðÊViÌ>ÃÊ >ÀiÊÃÌ>LiÊ՘`iÀÊÃÌÀœ˜}ÞÊL>ÈVÊVœ˜`ˆÌˆœ˜Ã° UÊ i“ˆ>ViÌ>ÃÊ>ÀiÊ}i˜iÀ>ÞÊ`ˆvwVՏÌÊ̜ʈ܏>ÌiÊ՘iÃÃÊ̅iÞÊ>ÀiÊ VÞVˆV°

30/04/10 16.47

48

CHAPTER 20

Aldehydes and Ketones

SECTION 20.6 UÊ ˜Ê>Vˆ`ˆVÊVœ˜`ˆÌˆœ˜Ã]Ê>˜Ê>`i…Þ`iʜÀʎi̜˜iÊ܈ÊÀi>VÌÊ܈̅Ê>Ê «Àˆ“>ÀÞÊ>“ˆ˜iÊ̜ÊvœÀ“Ê>˜Êimine° UÊ /…iÊwÀÃÌÊ̅ÀiiÊÃÌi«Ãʈ˜Êˆ“ˆ˜iÊvœÀ“>̈œ˜Ê«Àœ`ÕViÊ>Êcarbinolamine]Ê >˜`Ê Ì…iÊ >ÃÌÊ Ì…ÀiiÊ ÃÌi«ÃÊ Vœ˜ÛiÀÌÊ Ì…iÊ V>ÀLˆ˜œ>“ˆ˜iÊ ˆ˜ÌœÊ>˜Êˆ“ˆ˜i° UÊ >˜ÞÊ`ˆvviÀi˜ÌÊVœ“«œÕ˜`ÃʜvÊ̅iÊvœÀ“Ê, ÓÊ܈ÊÀi>VÌÊÜˆÌ…Ê >`i…Þ`iÃÊ>˜`ʎi̜˜iÃÆÊvœÀÊiÝ>“«i\ 1.Ê 7…i˜Ê …Þ`À>∘iÊ ˆÃÊ ÕÃi`Ê >ÃÊ >Ê ˜ÕViœ«…ˆiÊ ­ ÓNHÓ®]Ê >Ê hydrazoneʈÃÊvœÀ“i`° 2.Ê 7…i˜Ê …Þ`ÀœÝޏ>“ˆ˜iÊ ˆÃÊ ÕÃi`Ê >ÃÊ >Ê ˜ÕViœ«…ˆiÊ ­ Ó"®]Ê >˜ÊoximeʈÃÊvœÀ“i`° UÊ ˜Ê >Vˆ`ˆVÊ Vœ˜`ˆÌˆœ˜Ã]Ê >˜Ê >`i…Þ`iÊ œÀÊ Ži̜˜iÊ ÜˆÊ Ài>VÌÊ ÜˆÌ…Ê >ÊÃiVœ˜`>ÀÞÊ>“ˆ˜iÊ̜ÊvœÀ“Ê>˜Êenamine°Ê/…iʓiV…>˜ˆÃ“ÊœvÊ i˜>“ˆ˜iÊvœÀ“>̈œ˜ÊˆÃʈ`i˜ÌˆV>Ê̜Ê̅iʓiV…>˜ˆÃ“Êœvʈ“ˆ˜iÊvœÀ“>̈œ˜ÊiÝVi«ÌÊvœÀÊ̅iʏ>ÃÌÊÃÌi«° UÊ ˜Ê̅iÊWolff-Kishner reduction]Ê>ʅÞ`À>✘iʈÃÊÀi`ÕVi`Ê̜Ê>˜Ê >Ž>˜iÊ՘`iÀÊÃÌÀœ˜}ÞÊL>ÈVÊVœ˜`ˆÌˆœ˜Ã°

SECTION 20.7 UÊ ˜Ê>Vˆ`ˆVÊVœ˜`ˆÌˆœ˜Ã]Ê>ÊÀi>}i˜ÌÃ]ʈ˜ÌiÀ“i`ˆ>ÌiÃ]Ê>˜`ʏi>ۈ˜}Ê }ÀœÕ«ÃÊ iˆÌ…iÀÊ Ã…œÕ`Ê LiÊ ˜iÕÌÀ>Ê ­˜œÊ V…>À}i®Ê œÀÊ Ã…œÕ`Ê Li>ÀÊ œ˜iÊ«œÃˆÌˆÛiÊV…>À}i° UÊ HydrolysisÊ œvÊ >ViÌ>Ã]Ê ˆ“ˆ˜iÃ]Ê >˜`Ê i˜>“ˆ˜iÃÊ Õ˜`iÀÊ >Vˆ`ˆVÊ Vœ˜`ˆÌˆœ˜ÃÊ«Àœ`ÕViÃʎi̜˜iÃʜÀÊ>`i…Þ`ið

UÊ Àˆ}˜>À`ÊÀi>V̈œ˜ÃÊ>ÀiʘœÌÊÀiÛiÀÈLi]ÊLiV>ÕÃiÊV>ÀL>˜ˆœ˜ÃÊ`œÊ ˜œÌÊv՘V̈œ˜Ê>Ãʏi>ۈ˜}Ê}ÀœÕ«Ã° UÊ 7…i˜ÊÌÀi>Ìi`Ê܈̅ʅÞ`Àœ}i˜ÊVÞ>˜ˆ`iÊ­ ®]Ê>`i…Þ`iÃÊ>˜`Ê Ži̜˜iÃÊ >ÀiÊ Vœ˜ÛiÀÌi`Ê ˆ˜ÌœÊ cyanohydrins°Ê œÀÊ “œÃÌÊ >`i…Þ`iÃÊ>˜`Ê՘…ˆ˜`iÀi`ʎi̜˜iÃ]Ê̅iÊiµÕˆˆLÀˆÕ“Êv>ۜÀÃÊvœÀ“>̈œ˜ÊœvÊ̅iÊVÞ>˜œ…Þ`Àˆ˜° UÊ /…iÊWittig reactionÊV>˜ÊLiÊÕÃi`Ê̜ÊVœ˜ÛiÀÌÊ>ʎi̜˜iÊ̜Ê>˜Ê >Ži˜i°Ê/…iÊWittig reagentÊ̅>ÌÊ>VVœ“«ˆÃ…iÃÊ̅ˆÃÊÌÀ>˜ÃvœÀ“>̈œ˜Ê ˆÃÊ V>i`Ê >Ê phosphorane]Ê Ü…ˆV…Ê Liœ˜}ÃÊ ÌœÊ >Ê >À}iÀÊ V>ÃÃʜvÊVœ“«œÕ˜`ÃÊV>i`Êylides° UÊ /…iʓiV…>˜ˆÃ“ÊœvÊ>Ê7ˆÌ̈}ÊÀi>V̈œ˜Êˆ˜ÛœÛiÃʈ˜ˆÌˆ>ÊvœÀ“>̈œ˜Ê œvÊ>Êbetaine]Ê܅ˆV…Ê՘`iÀ}œiÃÊ>˜Êˆ˜ÌÀ>“œiVՏ>ÀʘÕViœ«…ˆˆVÊ >ÌÌ>VŽ]Ê }i˜iÀ>̈˜}Ê >˜Ê oxaphosphetane°Ê ,i>ÀÀ>˜}i“i˜ÌÊ }ˆÛiÃÊ̅iÊ«Àœ`ÕVÌ° UÊ *Ài«>À>̈œ˜ÊœvÊ7ˆÌ̈}ÊÀi>}i˜ÌÃʈ˜ÛœÛiÃÊ>˜Ê-NÓÊÀi>V̈œ˜]Ê>˜`Ê Ì…iÊÀi}Տ>ÀÊÀiÃÌÀˆV̈œ˜ÃʜvÊ-NÓÊ«ÀœViÃÃiÃÊ>««Þ°

SECTION 20.11 UÊ ÊBaeyer-Villiger oxidationÊVœ˜ÛiÀÌÃÊ>ʎi̜˜iÊ̜Ê>˜ÊiÃÌiÀÊLÞÊ ˆ˜ÃiÀ̈˜}Ê>˜ÊœÝÞ}i˜Ê>̜“ʘiÝÌÊ̜Ê̅iÊV>ÀLœ˜ÞÊ}ÀœÕ«°Ê ÞVˆVÊ Ži̜˜iÃÊ«Àœ`ÕViÊVÞVˆVÊiÃÌiÀÃÊV>i`Êlactones° UÊ 7…i˜Ê>˜Ê՘Ãޓ“iÌÀˆV>ÊŽi̜˜iʈÃÊÌÀi>Ìi`Ê܈̅Ê>Ê«iÀœÝÞÊ>Vˆ`]Ê vœÀ“>̈œ˜Ê œvÊ Ì…iÊ iÃÌiÀÊ ˆÃÊ Ài}ˆœÃiiV̈Ûi]Ê >˜`Ê Ì…iÊ «Àœ`ÕVÌÊ ˆÃÊ `iÌiÀ“ˆ˜i`Ê LÞÊ Ì…iÊ migratory aptitudeÊ œvÊ i>V…Ê }ÀœÕ«Ê ˜iÝÌÊ ÌœÊ̅iÊV>ÀLœ˜Þ°

SECTION 20.12

SECTION 20.8 UÊ ˜Ê>Vˆ`ˆVÊVœ˜`ˆÌˆœ˜Ã]Ê>˜Ê>`i…Þ`iʜÀʎi̜˜iÊ܈ÊÀi>VÌÊ܈̅ÊÌÜœÊ iµÕˆÛ>i˜ÌÃÊ œvÊ >Ê Ì…ˆœÊ ÌœÊ vœÀ“Ê >Ê thioacetal°Ê vÊ >Ê Vœ“«œÕ˜`Ê ÜˆÌ…ÊÌܜÊ-Ê}ÀœÕ«ÃʈÃÊÕÃi`]Ê>ÊVÞVˆVÊ̅ˆœ>ViÌ>ÊˆÃÊvœÀ“i`° UÊ 7…i˜ÊÌÀi>Ìi`Ê܈̅Ê,>˜iÞʘˆVŽi]Ê̅ˆœ>ViÌ>ÃÊ՘`iÀ}œÊdesulfurizationÊ̜Êވi`Ê>ʓi̅ޏi˜iÊ}ÀœÕ«°

UÊ /…ˆÃÊ V…>«ÌiÀÊ iÝ«œÀi`Ê Ì…ÀiiÊ q Ê Lœ˜`‡vœÀ“ˆ˜}Ê Ài>V̈œ˜Ã\Ê ­£®Ê>ÊÀˆ}˜>À`ÊÀi>V̈œ˜]Ê­Ó®ÊVÞ>˜œ…Þ`Àˆ˜ÊvœÀ“>̈œ˜]Ê>˜`ʭήÊ>Ê 7ˆÌ̈}ÊÀi>V̈œ˜° UÊ /…ˆÃÊV…>«ÌiÀÊiÝ«œÀi`ʜ˜Þʜ˜iÊ q ÊLœ˜`‡LÀi>Žˆ˜}ÊÀi>V̈œ˜\Ê Ì…iÊ >iÞiÀ‡6ˆˆ}iÀʜ݈`>̈œ˜°

SECTION 20.13

SECTION 20.9 UÊ 7…i˜ÊÌÀi>Ìi`Ê܈̅Ê>ʅÞ`Àˆ`iÊÀi`ÕVˆ˜}Ê>}i˜Ì]ÊÃÕV…Ê>ÃʏˆÌ…ˆÕ“Ê >Õ“ˆ˜Õ“Ê …Þ`Àˆ`iÊ ­®Ê œÀÊ Ãœ`ˆÕ“Ê LœÀœ…Þ`Àˆ`iÊ ­ > {®]Ê >`i…Þ`iÃÊ>˜`ʎi̜˜iÃÊ>ÀiÊÀi`ÕVi`Ê̜Ê>Vœ…œÃ° UÊ /…iÊÀi`ÕV̈œ˜ÊœvÊ>ÊV>ÀLœ˜ÞÊ}ÀœÕ«Ê܈̅ÊʜÀÊ > {ʈÃʘœÌÊ >ÊÀiÛiÀÈLiÊ«ÀœViÃÃ]ÊLiV>ÕÃiʅÞ`Àˆ`iÊ`œiÃʘœÌÊv՘V̈œ˜Ê>ÃÊ>Ê i>ۈ˜}Ê}ÀœÕ«°

SECTION 20.10 UÊ 7…i˜ÊÌÀi>Ìi`Ê܈̅Ê>ÊÀˆ}˜>À`Ê>}i˜Ì]Ê>`i…Þ`iÃÊ>˜`ʎi̜˜iÃÊ >ÀiÊVœ˜ÛiÀÌi`ʈ˜ÌœÊ>Vœ…œÃ]Ê>VVœ“«>˜ˆi`ÊLÞÊ̅iÊvœÀ“>̈œ˜Ê œvÊ>ʘiÜÊ q ÊLœ˜`°

UÊ >ÀLœ˜ÞÊ }ÀœÕ«ÃÊ «Àœ`ÕViÊ >Ê ÃÌÀœ˜}Ê ,Ê Ãˆ}˜>Ê >ÀœÕ˜`Ê £Ç£xÊ V“•£°Ê Ê Vœ˜Õ}>Ìi`Ê V>ÀLœ˜ÞÊ «Àœ`ÕViÃÊ >Ê Ãˆ}˜>Ê >ÌÊ >Ê œÜiÀÊÜ>Ûi˜Õ“LiÀ]Ê܅ˆiÊÀˆ˜}ÊÃÌÀ>ˆ˜Êˆ˜VÀi>ÃiÃÊ̅iÊÜ>Ûi˜Õ“LiÀʜvÊ>LÜÀ«Ìˆœ˜° UÊ `i…Þ`ˆVÊ qÊ Lœ˜`ÃÊ i݅ˆLˆÌÊ œ˜iÊ œÀÊ ÌÜœÊ Ãˆ}˜>ÃÊ LiÌÜii˜Ê ÓÇääÊ>˜`ÊÓnxäÊV“•£° £

UÊ ˜Ê >Ê Ê ,Ê Ã«iVÌÀՓ]Ê >Ê V>ÀLœ˜ÞÊ }ÀœÕ«Ê >``ÃÊ ³£Ê ««“Ê ÌœÊ Ì…iÊV…i“ˆV>ÊňvÌʜvʈÌÃʘiˆ}…LœÀÃ]Ê>˜`Ê>˜Ê>`i…Þ`ˆVÊ«ÀœÌœ˜Ê «Àœ`ÕViÃÊ>ÊÈ}˜>Ê>ÀœÕ˜`Ê£äÊ««“° £Î

UÊ ˜Ê>Ê Ê ,ÊëiVÌÀՓ]Ê>ÊV>ÀLœ˜ÞÊ}ÀœÕ«Ê«Àœ`ÕViÃÊ>ÊÜi>ŽÊ È}˜>Ê˜i>ÀÊÓääÊ««“°

KEY TERMINOLOGY acetalÊ UUU >iÞiÀ‡6ˆˆ}iÀʜ݈`>̈œ˜Ê UUU LiÌ>ˆ˜iÊ UUU V>ÀLˆ˜œ>“ˆ˜iÊ UUU V>ÀLœ˜ÞÊ}ÀœÕ«Ê UUU VÞ>˜œ…Þ`Àˆ˜ÃÊ UUU

klein_c20_001-056v1.4.indd 48

`iÃՏvÕÀˆâ>̈œ˜Ê UUU i˜>“ˆ˜iÊ UUU …i“ˆ>ViÌ>Ê UUU …Þ`À>ÌiÊ UUU …Þ`À>✘iÊ UUU …Þ`ÀœÞÈÃÊ UUU

ˆ“ˆ˜iÊ UUU >V̜˜iÊ UUU “ˆ}À>̜ÀÞÊ>«ÌˆÌÕ`iÊ UUU œÝ>«…œÃ«…iÌ>˜iÊ UUU œÝˆ“iÊ UUU «…œÃ«…œÀ>˜iÊ UUU

ÌiÌÀ>…i`À>Êˆ˜ÌiÀ“i`ˆ>ÌiÊ UUU ̅ˆœ>ViÌ>Ê UUU 7ˆÌ̈}ÊÀi>V̈œ˜Ê UUU 7ˆÌ̈}ÊÀi>}i˜ÌÊ UUU 7œvv‡ˆÃ…˜iÀÊÀi`ÕV̈œ˜Ê UUU ޏˆ`iÊ UUU

30/04/10 16.47

49

Skillbuilder Review

SKILLBUILDER REVIEW 20.1 NAMING ALDEHYDES AND KETONES STEP 1 …œœÃiÊ̅iʏœ˜}iÃÌÊ V…>ˆ˜ÊVœ˜Ì>ˆ˜ˆ˜}Ê̅iÊV>ÀLœ˜ÞÊ }ÀœÕ«]Ê>˜`ʘՓLiÀÊ̅iÊV…>ˆ˜Ê ÃÌ>À̈˜}ÊvÀœ“Ê̅iÊi˜`ÊVœÃiÃÌÊ ÌœÊ̅iÊV>ÀLœ˜ÞÊ}ÀœÕ«°

STEP 2 AND 3 `i˜ÌˆvÞÊ Ì…iÊÃÕLÃ̈ÌÕi˜ÌÃ]Ê>˜`Ê>ÃÈ}˜Ê œV>˜Ìð

4,4-dimethyl

8 7 2

4

1 3

9 2

4

1 3

5

O

9

R

6 5

O

O

4,4-dimethyl-6-ethyl

6-ethyl

O

3-nonanone

STEP 5 ÃÈ}˜Ê̅iÊ Vœ˜w}ÕÀ>̈œ˜ÊœvÊ>˜ÞÊV…ˆÀ>ˆÌÞÊ Vi˜ÌiÀ°

8

7

6

STEP 4 ÃÃi“LiÊ̅iÊ ÃÕLÃ̈ÌÕi˜ÌÃÊ>«…>LïV>Þ°

(R)-4,4-dimethyl-6-ethyl -3-nonanone

Try problems 20.1–20.4, 20.44–20.49

20.2 DRAWING THE MECHANISM OF ACETAL FORMATION STEP 1 À>ÜÊ̅iÊ̅ÀiiÊÃÌi«ÃʘiViÃÃ>ÀÞÊvœÀʅi“ˆ>ViÌ>ÊvœÀ“>̈œ˜° Proton transfer

Nucleophilic attack

H¬A

O

+

Proton transfer

H

+

O

COMMENTS UÊ ÛiÀÞÊÃÌi«Ê…>ÃÊÌܜÊVÕÀÛi`Ê>ÀÀœÜÃ°Ê À>ÜÊ̅i“Ê«ÀiVˆÃiÞ° UÊ œÊ˜œÌÊvœÀ}iÌÊ̅iÊV…>À}ið UÊ À>ÜÊi>V…ÊÃÌi«ÊÃi«>À>ÌiÞ°

OH

R¬O¬H

OH

A

H O+

OR

R

STEP 2 À>ÜÊ̅iÊvœÕÀÊÃÌi«ÃÊ̅>ÌÊVœ˜ÛiÀÌÊ̅iʅi“ˆ>ViÌ>Êˆ˜ÌœÊ>˜Ê>ViÌ>° Proton transfer

OH

H¬A

H + H O

+

Nucleophilic attack

Loss of a leaving group

–H2O

+

R

R + H O

R¬ O ¬H

O

Proton transfer

OR

OR

A

OR

OR OR

Try Problems 20.8–20.10, 20.57, 20.62, 20.67

20.3 DRAWING THE MECHANISM OF IMINE FORMATION STEP 1 À>ÜÊ̅iÊ̅ÀiiÊÃÌi«ÃʘiViÃÃ>ÀÞÊvœÀÊV>ÀLˆ˜œ>“ˆ˜iÊvœÀ“>̈œ˜° Nucleophilic attack

Proton transfer +

O H¬A

+

O

H R¬NH2

COMMENTS UÊ >V…Ê«>ÀÌʜvÊ̅iʓiV…>˜ˆÃ“ÊLi}ˆ˜ÃÊ ÜˆÌ…Ê>Ê«ÀœÌœ˜ÊÌÀ>˜ÃviÀÊ>˜`Êi˜`ÃÊ ÜˆÌ…Ê>Ê«ÀœÌœ˜ÊÌÀ>˜ÃviÀ° UÊ ÛiÀÞÊÃÌi«Ê…>ÃÊÌܜÊVÕÀÛi`Ê>ÀÀœÜÃ°Ê >ŽiÊÃÕÀiÊ̜Ê`À>ÜÊ̅i“Ê«ÀiVˆÃiÞ° UÊ œÊ˜œÌÊvœÀ}iÌÊ̅iÊ«œÃˆÌˆÛiÊV…>À}iÃ°Ê /…iÀiÊŜՏ`ÊLiʘœÊ˜i}>̈ÛiÊ V…>À}ið UÊ À>ÜÊi>V…ÊÃÌi«ÊÃi«>À>ÌiÞ]Ê vœœÜˆ˜}Ê̅iÊ«ÀiVˆÃiʜÀ`iÀʜvÊ ÃÌi«Ã°

Proton transfer

OH

OH A

H

N

N

R

+

H

H

R

STEP 2 À>ÜÊ̅iÊ̅ÀiiÊÃÌi«ÃÊ̅>ÌÊVœ˜ÛiÀÌÊ̅iÊV>ÀLˆ˜œ>“ˆ˜iʈ˜ÌœÊ>˜Êˆ“ˆ˜i° Loss of a leaving group

Proton transfer

OH

H¬A

N R

H

+

H + H O N

–H2O

R + H N

Proton transfer R R–NH2

N

H

R

Try Problems 20.15–20.18, 20.72, 20.86

klein_c20_001-056v1.4.indd 49

30/04/10 16.47

50

CHAPTER 20

Aldehydes and Ketones

20.4 DRAWING THE MECHANISM OF ENAMINE FORMATION COMMENTS UÊ >V…Ê«>ÀÌʜvÊ̅iʓiV…>˜ˆÃ“Ê Li}ˆ˜ÃÊ܈̅Ê>Ê«ÀœÌœ˜ÊÌÀ>˜ÃviÀÊ >˜`Êi˜`ÃÊ܈̅Ê>Ê«ÀœÌœ˜Ê ÌÀ>˜ÃviÀ° UÊ ÛiÀÞÊÃÌi«Ê…>ÃÊÌܜÊVÕÀÛi`Ê >ÀÀœÜðÊ>ŽiÊÃÕÀiÊ̜Ê`À>ÜÊ Ì…i“Ê«ÀiVˆÃiÞ° UÊ œÊ˜œÌÊvœÀ}iÌÊ̅iÊ«œÃˆÌˆÛiÊ V…>À}iðÊ/…iÀiÊŜՏ`ÊLiʘœÊ ˜i}>̈ÛiÊV…>À}ið UÊ À>ÜÊi>V…ÊÃÌi«ÊÃi«>À>ÌiÞÊ vœœÜˆ˜}Ê̅iÊ«ÀiVˆÃiʜÀ`iÀʜvÊ ÃÌi«Ã°

STEP 1 À>ÜÊ̅iÊ̅ÀiiÊÃÌi«ÃʘiViÃÃ>ÀÞÊvœÀÊV>ÀLˆ˜œ>“ˆ˜iÊvœÀ“>̈œ˜° Nucleophilic attack

Proton transfer H

+

O H¬A

O

+

Proton transfer OH

OH

R2NH

A

H

N

N

R +R

R

R

STEP 2 À>ÜÊ̅iÊ̅ÀiiÊÃÌi«ÃÊ̅>ÌÊVœ˜ÛiÀÌÊ̅iÊV>ÀLˆ˜œ>“ˆ˜iʈ˜ÌœÊ>˜Êi˜>“ˆ˜i° Proton transfer

OH

H¬A

N

H + H O

+

Nucleophilic attack

Loss of a leaving group

R

N

R

–H2O

R + R N

R

R2NH

R

R N

H

R

Try Problems 20.21–20.24, 20.72

20.5 DRAWING THE MECHANISM OF A HYDROLYSIS REACTION STEP 1 7œÀŽˆ˜}ÊL>VŽÜ>À`Ã]Ê`À>ÜÊ>Êˆ˜ÌiÀ“i`ˆ>Ìið

STEP 2 UÊvÌiÀÊ`À>܈˜}Ê>Êˆ˜ÌiÀ“i`ˆ>ÌiÃ]Ê̅i˜Ê`À>ÜÊ>Ê Ài>}i˜ÌÃÊ>˜`ÊVÕÀÛi`Ê>ÀÀœÜÃÊÕȘ}Ê̅iÊvœœÜˆ˜}ÊÀՏiÃ\ UʘÊ>Vˆ`ˆVÊVœ˜`ˆÌˆœ˜Ã]Ê>ÊÀi>}i˜ÌÃ]ʈ˜ÌiÀ“i`ˆ>ÌiÃ]Ê>˜`Ê i>ۈ˜}Ê}ÀœÕ«ÃÊiˆÌ…iÀÊŜՏ`ÊLiʘiÕÌÀ>ÊœÀÊŜՏ`Ê Li>Àʜ˜iÊ«œÃˆÌˆÛiÊV…>À}i° UÊ1Ãiʜ˜ÞÊ̅œÃiÊÀi>}i˜ÌÃÊ̅>ÌÊ>ÀiÊ>Ài>`ÞÊ«ÀiÃi˜Ì°

first draw this intermediate +

H

O

O

then draw the other intermediates, working backward

Try Problems 20.26, 20.27, 20.65

20.6 PLANNING AN ALKENE SYNTHESIS WITH A WITTIG REACTION ÊEXAMPLE `i˜ÌˆvÞÊ Ê̅iÊÀi>VÌ>˜ÌÃÊޜÕÊ ÊܜՏ`ÊÕÃiÊ̜ʫÀi«>ÀiÊ Ê̅ˆÃÊVœ“«œÕ˜`Êۈ>Ê>Ê 7ˆÌ̈}ÊÀi>V̈œ˜°

STEP 1 1Ș}Ê>ÊÀiÌÀœÃޘ̅ïVÊ>˜>ÞÈÃ]Ê`iÌiÀ“ˆ˜iÊ̅iÊÌÜœÊ «œÃÈLiÊÃiÌÃʜvÊÀi>VÌ>˜ÌÃÊ̅>ÌÊVœÕ`ÊLiÊÕÃi`Ê̜ÊvœÀ“Ê̅iÊ

5 ÊLœ˜`°

H

O

H PPh3

PPh3

+

+

Method 1

Method 2

STEP 2 œ˜Ãˆ`iÀʅœÜÊޜÕÊ ÜœÕ`ʓ>ŽiÊi>V…Ê«œÃÈLiÊ 7ˆÌ̈}ÊÀi>}i˜Ì]Ê>˜`Ê`iÌiÀ“ˆ˜iÊ Ü…ˆV…Ê“i̅œ`ʈ˜ÛœÛiÃÊ̅iʏiÃÃÊ ÃÕLÃ̈ÌÕÌi`Ê>ŽÞÊ…>ˆ`i° x

O

X

Will undergo SN2 more readily

2˚ Alkyl halide

1˚ Alkyl halide

Try Problems 20.37–20.39, 20.51–20.53

20.7 PROPOSING A SYNTHESIS STEP 1 i}ˆ˜ÊLÞÊ >Έ˜}Ê̅iÊvœœÜˆ˜}ÊÌÜœÊ µÕiÃ̈œ˜Ã\ £°ÊÃÊ̅iÀiÊ>ÊV…>˜}iʈ˜Ê̅iÊ V>ÀLœ˜ÊÎii̜˜¶ Ó°ÊÃÊ̅iÀiÊ>ÊV…>˜}iʈ˜Ê̅iÊ v՘V̈œ˜>Ê}ÀœÕ«Ã¶

AU/ED: long page klein_c20_001-056v1.4.indd 50

STEP 2 vÊ̅iÀiʈÃÊ>ÊV…>˜}iʈ˜Ê̅iÊV>ÀLœ˜ÊÎii̜˜]Ê Vœ˜Ãˆ`iÀÊ>ÊœvÊ̅iÊ q ÊLœ˜`‡vœÀ“ˆ˜}ÊÀi>V̈œ˜ÃÊ>˜`Ê >ÊœvÊ̅iÊ q ÊLœ˜`‡LÀi>Žˆ˜}ÊÀi>V̈œ˜ÃÊ̅>ÌÊޜÕʅ>ÛiÊ i>À˜i`ÊÜÊv>À

q ÊLœ˜`‡vœÀ“ˆ˜}ÊÀi>V̈œ˜Ãʈ˜Ê̅ˆÃÊV…>«ÌiÀ UÊÀˆ}˜>À`ÊÀi>V̈œ˜ UÊ Þ>˜œ…Þ`Àˆ˜ÊvœÀ“>̈œ˜ UÊ7ˆÌ̈˜}ÊÀi>V̈œ˜

q ÊLœ˜`‡LÀi>Žˆ˜}ÊÀi>V̈œ˜Ãʈ˜Ê̅ˆÃÊV…>«ÌiÀ UÊ >iÞiÀ‡6ˆˆ}iÀʜ݈`>̈œ˜

CONSIDERATIONS ,i“i“LiÀÊ̅>ÌÊ̅iÊ`iÈÀi`Ê«Àœ`ÕVÌÊŜՏ`ÊLiÊ̅iÊ “>œÀÊ«Àœ`ÕVÌʜvÊޜÕÀÊ«Àœ«œÃi`ÊÃޘ̅iÈð >ŽiÊÃÕÀiÊ̅>ÌÊ̅iÊÀi}ˆœV…i“ˆV>ÊœÕÌVœ“iʜvÊi>V…Ê ÃÌi«ÊˆÃÊVœÀÀiVÌ° Ü>ÞÃÊ̅ˆ˜ŽÊL>VŽÜ>À`Ê­ÀiÌÀœÃޘ̅ïVÊ>˜>ÞÈîÊ>ÃÊÜiÊ >ÃÊvœÀÜ>À`]Ê>˜`Ê̅i˜ÊÌÀÞÊ̜ÊLÀˆ`}iÊ̅iÊ}>«° œÃÌÊÃޘ̅iÈÃÊ«ÀœLi“ÃÊ܈Ê…>ÛiʓՏ̈«iÊVœÀÀiVÌÊ >˜ÃÜiÀÃ°Ê œÊ˜œÌÊviiÊ̅>ÌÊޜÕʅ>ÛiÊ̜Êw˜`Ê̅iʺœ˜i»Ê VœÀÀiVÌÊ>˜ÃÜiÀ°

Try Problems 20.41, 20.42, 20.55, 20.58, 20.67–20.69, 20.71, 20.75 30/04/10 16.47

51

Practice Problems

Note:ÊœÃÌʜvÊ̅iÊ*ÀœLi“ÃÊ>ÀiÊ>Û>ˆ>LiÊ܈̅ˆ˜ WileyPLUS]Ê>˜Êœ˜ˆ˜iÊÌi>V…ˆ˜}Ê>˜`ʏi>À˜ˆ˜}Ê܏Ṏœ˜°

PRACTICE PROBLEMS 20.44

*ÀœÛˆ`iÊ>ÊÃÞÃÌi“>̈VÊ­1* ®Ê˜>“iÊvœÀÊi>V…ÊœvÊ̅iÊvœœÜˆ˜}ÊVœ“«œÕ˜`Ã\

O

O H

O

O

(a)

H

O

O

(a)

(b)

O H

(c)

20.51

À>ÜÊ Ì…iÊ «Àœ`ÕVÌÃÊ œvÊ i>V…Ê 7ˆÌ̈}Ê Ài>V̈œ˜Ê LiœÜ°Ê vÊ ÌܜÊÃÌiÀiœˆÃœ“iÀÃÊ>ÀiÊ«œÃÈLi]Ê`À>ÜÊLœÌ…ÊÃÌiÀiœˆÃœ“iÀÃ\Ê

(d) O H

HO

(e)

(f)

Br

?

H

(a) O

À>ÜÊ̅iÊÃÌÀÕVÌÕÀiÊvœÀÊi>V…ÊVœ“«œÕ˜`ÊLiœÜ\

Ph

­>®Ê «Àœ«>˜i`ˆ> ­L®Ê {‡«…i˜ÞLÕÌ>˜>

Ph

Ph Ph P Ph

O

O

20.45

CH3

H3C

CF3

F3C

(b) O

(b)

Ph Ph P Ph

H

?

Ph Ph P Ph

H

?

Ph

­V®Ê (S)‡Î‡«…i˜ÞLÕÌ>˜> ­`®Ê Î]Î]x]x‡ÌiÌÀ>“i̅ޏ‡{‡…i«Ì>˜œ˜i O

­i®Ê (R)‡Î‡…Þ`ÀœÝÞ«i˜Ì>˜> ­v®Ê “iÌ>‡…Þ`ÀœÝÞ>Vi̜«…i˜œ˜i ­}®Ê Ó]{]ȇÌÀˆ˜ˆÌÀœLi˜â>`i…Þ`i

H

(c)

­…®Ê ÌÀˆLÀœ“œ>ViÌ>`i…Þ`i ­ˆ®Ê (3R,4R)‡Î]{‡`ˆ…Þ`ÀœÝއӇ«i˜Ì>˜œ˜i

Ph Ph P Ph

O

20.46

À>ÜÊ >Ê Vœ˜Ã̈ÌṎœ˜>ÞÊ ˆÃœ“iÀˆVÊ >`i…Þ`iÃÊ ÜˆÌ…Ê “œiVՏ>ÀÊ vœÀ“Տ>Ê {Hn"]Ê >˜`Ê «ÀœÛˆ`iÊ >Ê ÃÞÃÌi“>̈VÊ ­1* ®Ê ˜>“iÊvœÀÊi>V…ʈÜ“iÀ°

20.47

À>ÜÊ >Ê Vœ˜Ã̈ÌṎœ˜>ÞÊ ˆÃœ“iÀˆVÊ >`i…Þ`iÃÊ ÜˆÌ…Ê “œiVՏ>ÀÊ vœÀ“Տ>Ê xH£ä"]Ê >˜`Ê «ÀœÛˆ`iÊ >Ê ÃÞÃÌi“>̈VÊ ­1* ®Ê ˜>“iÊvœÀÊi>V…ʈÜ“iÀ°Ê7…ˆV…ÊœvÊ̅iÃiʈܓiÀÃÊ«œÃÃiÃÃiÃÊ>ÊV…ˆÀ>ˆÌÞÊVi˜ÌiÀ¶Ê

H

?

(d)

20.52 À>ÜÊ̅iÊÃÌÀÕVÌÕÀiʜvÊ̅iÊ>ŽÞÊ…>ˆ`iʘii`i`Ê̜ʫÀi«>ÀiÊ i>V…Ê œvÊ Ì…iÊ vœœÜˆ˜}Ê 7ˆÌ̈}Ê Ài>}i˜ÌÃ]Ê >˜`Ê Ì…i˜Ê `iÌiÀ“ˆ˜iÊ܅ˆV…Ê7ˆÌ̈}ÊÀi>}i˜ÌÊ܈ÊLiÊ̅iʓœÃÌÊ`ˆvwVՏÌÊ̜ʫÀi«>Ài°Ê

Ý«>ˆ˜ÊޜÕÀÊV…œˆVi\Ê

20.48

À>ÜÊ>ÊVœ˜Ã̈ÌṎœ˜>ÞʈܓiÀˆVʎi̜˜iÃÊ܈̅ʓœiVՏ>ÀÊ vœÀ“Տ>Ê ÈH£Ó"]Ê >˜`Ê «ÀœÛˆ`iÊ >Ê ÃÞÃÌi“>̈VÊ ­1* ®Ê ˜>“iÊ vœÀÊi>V…ʈÜ“iÀ°Ê

CH3

Ph

(a)

Ph P Ph

(c)

Ph Ph P Ph

Ph H

(b)

Ph Ph P Ph

H

20.49

Ý«>ˆ˜Ê ܅ÞÊ Ì…iÊ 1* Ê ˜>“iÊ œvÊ >Ê Vœ“«œÕ˜`Ê ÜˆÊ ˜iÛiÀÊi˜`Ê܈̅Ê̅iÊÃÕvwÝʺ‡£‡œ˜i°»Ê

20.50

œÀÊ i>V…Ê «>ˆÀÊ œvÊ Ì…iÊ vœœÜˆ˜}Ê Vœ“«œÕ˜`Ã]Ê ˆ`i˜ÌˆvÞÊ Ü…ˆV…ÊVœ“«œÕ˜`ÊܜՏ`ÊLiÊiÝ«iVÌi`Ê̜ÊÀi>VÌʓœÀiÊÀ>«ˆ`ÞÊÜˆÌ…Ê >ʘÕViœ«…ˆi\Ê

klein_c20_001-056v1.4.indd 51

30/04/10 16.47

52

CHAPTER 20

Aldehydes and Ketones

20.53

-…œÜÊ …œÜÊ >Ê 7ˆÌ̈}Ê Ài>V̈œ˜Ê V>˜Ê LiÊ ÕÃi`Ê ÌœÊ «Ài«>ÀiÊ i>V…ÊœvÊ̅iÊvœœÜˆ˜}ÊVœ“«œÕ˜`ðʘÊi>V…ÊV>Ãi]Ê>ÃœÊŜÜʅœÜÊ Ì…iÊ7ˆÌ̈}ÊÀi>}i˜ÌÊܜՏ`ÊLiÊ«Ài«>Ài`\Ê

20.60

*Ài`ˆVÌÊ̅iʓ>œÀÊ«Àœ`ÕV̭îÊvœÀÊi>V…ÊÀi>V̈œ˜ÊLiœÜ°Ê O 1) LAH 2) H2O

(a) (a)

O

(b)

1) PhMgBr 2) H2O

(b) O

(c)

(C6H5)3P=CH2

(c)

20.54

…œœÃiÊ>ÊÀˆ}˜>À`ÊÀi>}i˜ÌÊ>˜`Ê>ʎi̜˜iÊ̅>ÌÊV>˜ÊLiÊ ÕÃi`Ê̜ʫÀœ`ÕViÊi>V…ÊœvÊ̅iÊvœœÜˆ˜}ÊVœ“«œÕ˜`Ã\Ê ­>®Ê ·“i̅ޏ‡Î‡«i˜Ì>˜œÊ ­L®Ê £‡i̅ޏVÞVœ…iÝ>˜œ ­V®Ê ÌÀˆ«…i˜Þ“i̅>˜œÊ ­`®Ê x‡«…i˜Þ‡x‡˜œ˜>˜œ

O HCN, KCN

(d)

20.55

9œÕÊ >ÀiÊ ÜœÀŽˆ˜}Ê ˆ˜Ê >Ê >LœÀ>̜ÀÞ]Ê >˜`Ê ÞœÕÊ >ÀiÊ }ˆÛi˜Ê ̅iÊ Ì>ÃŽÊ œvÊ Vœ˜ÛiÀ̈˜}Ê VÞVœ«i˜Ìi˜iÊ ˆ˜ÌœÊ £]x‡«i˜Ì>˜i`ˆœ°Ê 9œÕÀÊ wÀÃÌÊ Ì…œÕ}…ÌÊ ˆÃÊ Ãˆ“«ÞÊ ÌœÊ «iÀvœÀ“Ê >˜Ê œâœ˜œÞÈÃÊ vœœÜi`Ê LÞÊ Ài`ÕV̈œ˜Ê ÜˆÌ…Ê ]Ê LÕÌÊ ÞœÕÀÊ >LÊ ˆÃÊ ˜œÌÊ iµÕˆ««i`Ê vœÀÊ >˜Ê œâœ˜œÞÈÃÊ Ài>V̈œ˜°Ê -Õ}}iÃÌÊ >˜Ê >ÌiÀ˜>̈ÛiÊ “i̅œ`Ê vœÀÊ Vœ˜ÛiÀ̈˜}Ê VÞVœ«i˜Ìi˜iÊ ˆ˜ÌœÊ £]x‡«i˜Ì>˜i`ˆœ°ÊÊ œÀʅi«]ÊÃiiÊ-iV̈œ˜Ê£Î°{Ê­Ài`ÕV̈œ˜ÊœvÊiÃÌiÀÃÊ̜Ê}ˆÛiÊ>Vœ…œÃ®°Ê

20.61

-Ì>À̈˜}Ê ÜˆÌ…Ê VÞVœ«i˜Ì>˜œ˜iÊ >˜`Ê ÕȘ}Ê >˜ÞÊ œÌ…iÀÊ Ài>}i˜ÌÃÊ œvÊ ÞœÕÀÊ V…œœÃˆ˜}]Ê ˆ`i˜ÌˆvÞÊ …œÜÊ ÞœÕÊ ÜœÕ`Ê «Ài«>ÀiÊ i>V…ÊœvÊ̅iÊvœœÜˆ˜}ÊVœ“«œÕ˜`Ã\Ê HO COOH

(a)

20.56

(b)

(c)

*Ài`ˆVÌÊ Ì…iÊ “>œÀÊ «Àœ`ÕVÌ­Ã®Ê vÀœ“Ê ̅iÊ ÌÀi>̓i˜ÌÊ œvÊ >Vi̜˜iÊ܈̅Ê̅iÊvœœÜˆ˜}ÊVœ“«œÕ˜`Ã\Ê ­>®Ê Q+RÊ]Ê ÎÊ]Ê­•Ó"®Ê

­L®Ê Q+RÊ]Ê ÎNHÓÊ]Ê­•Ó"®

­V®Ê Q+RÊ]ÊiÝViÃÃÊ Ì"]Ê­•Ó"®Ê

­`®Ê Q+RÊ]Ê­ Î®Ó Ê]Ê­•Ó"®

+

O

+

­i®Ê Q RÊ]Ê ÓNHÓÊ]Ê­•Ó"®Ê

­v®Ê Q RÊ]Ê Ó"Ê]Ê­•Ó"®

­}®Ê > {]Êi"Ê

­…®Ê  * 

­ˆ®Ê  ]Ê Ê

­®Ê Ì} ÀÊvœœÜi`ÊLÞÊÓ"

­Ž®Ê ­ ÈHx®ÎP5  Ó ÎÊ

­®Ê ÊvœœÜi`ÊLÞÊÓ"

(d)

20.57 *Àœ«œÃiÊ >Ê «>ÕÈLiÊ “iV…>˜ˆÃ“Ê vœÀÊ Ì…iÊ vœœÜˆ˜}Ê ÌÀ>˜ÃvœÀ“>̈œ˜\Ê

(e)

20.62

ÕÌ>À>`i…Þ`iÊ ˆÃÊ >Ê }iÀ“ˆVˆ`>Ê >}i˜ÌÊ Ì…>ÌÊ ˆÃÊ Ãœ“ï“iÃÊÕÃi`Ê̜ÊÃÌiÀˆˆâiʓi`ˆV>ÊiµÕˆ«“i˜ÌÊ̜œÊÃi˜ÃˆÌˆÛiÊ̜ÊLiÊ …i>Ìi`Ê ˆ˜Ê >˜Ê >Õ̜V>Ûi°Ê ˜Ê “ˆ`ÞÊ >Vˆ`ˆVÊ Vœ˜`ˆÌˆœ˜Ã]Ê }ÕÌ>À>`i…Þ`iÊ i݈ÃÌÃÊ ˆ˜Ê >Ê VÞVˆVÊ vœÀ“Ê ­LiœÜÊ Àˆ}…Ì®°Ê À>ÜÊ >Ê «>ÕÈLiÊ “iV…>˜ˆÃ“ÊvœÀÊ̅ˆÃÊÌÀ>˜ÃvœÀ“>̈œ˜\Ê O

O

O HO

O NH2

HO

O

[H+]

H

O

HO

[H3O+]

H

OH

O

H

EtOH

Glutaraldehyde

20.58 iۈÃiÊ>˜ÊivwVˆi˜ÌÊÃޘ̅iÈÃÊvœÀÊ̅iÊvœœÜˆ˜}ÊÌÀ>˜ÃvœÀ“>̈œ˜Ê­ÀiV>Ê̅>ÌÊÊ>`i…Þ`iÃÊ>ÀiʓœÀiÊÀi>V̈ÛiÊ̅>˜ÊŽi̜˜iî\Ê O

O O

H

HO

20.63 *Ài`ˆVÌÊ Ì…iÊ “>œÀÊ «Àœ`ÕVÌ­Ã®Ê œLÌ>ˆ˜i`Ê Ü…i˜Ê i>V…Ê œvÊ Ì…iÊvœœÜˆ˜}ÊVœ“«œÕ˜`ÃÊ՘`iÀ}œiÃʅÞ`ÀœÞÈÃʈ˜Ê̅iÊ«ÀiÃi˜ViÊ œvÊÎ"+\Ê N

H

N

(a)

20.59 /Ài>̓i˜ÌÊ œvÊ V>ÌiV…œÊ ÜˆÌ…Ê vœÀ“>`i-

(b)

O

(c)

OH

…Þ`iʈ˜Ê̅iÊ«ÀiÃi˜ViʜvÊ>˜Ê>Vˆ`ÊV>Ì>ÞÃÌÊ«Àœ`ÕViÃÊ >Ê Vœ“«œÕ˜`Ê ÜˆÌ…Ê “œiVՏ>ÀÊ vœÀ“Տ>Ê ÇHÈ"Ó°Ê À>ÜÊ̅iÊÃÌÀÕVÌÕÀiʜvÊ̅ˆÃÊ«Àœ`ÕVÌ°

OCH3

O OH Catechol

klein_c20_001-056v1.4.indd 52

O

O

O

(d)

(e)

30/04/10 16.47

53

Practice Problems

20.64

`i˜ÌˆvÞÊ >Ê œvÊ Ì…iÊ «Àœ`ÕVÌÃÊ vœÀ“i`Ê Ü…i˜Ê ̅iÊ Vœ“«œÕ˜`ÊLiœÜʈÃÊÌÀi>Ìi`Ê܈̅Ê>µÕiœÕÃÊ>Vˆ`\ O

O

(g)

N O

?

excess H3O+

O

O [H+]

(h)

ethylene glycol (1 equivalent)

H

20.65

À>ÜÊ>Ê«>ÕÈLiʓiV…>˜ˆÃ“ÊvœÀÊi>V…ÊœvÊ̅iÊvœœÜˆ˜}Ê ÌÀ>˜ÃvœÀ“>̈œ˜Ã\ H3O+

H

H

H2 N

[H+] OH

O

H3O+

N

?

HCN, KCN

(i) O

(b)

?

O

O

N

(a)

OH

(−H2O)

N

N

?

[H+] HS

?

(j) H

OH

(−H2O)

O

O [H+]

H2O

O

(c)

H

HO

20.67 `i˜ÌˆvÞÊ̅iÊÃÌ>À̈˜}ʓ>ÌiÀˆ>Ãʘii`i`Ê̜ʓ>ŽiÊi>V…ÊœvÊ Ì…iÊvœœÜˆ˜}Ê>ViÌ>Ã\

OH O

O

O

20.66

*Ài`ˆVÌÊ̅iʓ>œÀÊ«Àœ`ÕV̭îÊvœÀÊi>V…ÊœvÊ̅iÊvœœÜˆ˜}Ê Ài>V̈œ˜Ã\Ê

(a)

(b)

O

CH3 N

(a)

NH2

O

[H+] (−H2O)

?

(c)

OEt

O

O

(d)

O

O

20.68

1Ș}Ê i̅>˜œÊ >ÃÊ ÞœÕÀÊ œ˜ÞÊ ÃœÕÀViÊ œvÊ V>ÀLœ˜Ê >̜“Ã]Ê `iÈ}˜Ê>ÊÃޘ̅iÈÃÊvœÀÊ̅iÊvœœÜˆ˜}ÊVœ“«œÕ˜`\Ê

?

1) PhMgBr

(b)

2) H2O

O

O CH3CO3H

(c)

?

O

20.69

O

[H+]

(e)

*Àœ«œÃiÊ>˜ÊivwVˆi˜ÌÊÃޘ̅iÈÃÊvœÀÊi>V…ÊœvÊ̅iÊvœœÜˆ˜}Ê ÌÀ>˜ÃvœÀ“>̈œ˜Ã\

?

CH3CO3H

(d)

O

N H

O

?

O

(a) O

(−H2O)

O

[H+]

(f)

NH2 (−H2O)

klein_c20_001-056v1.4.indd 53

O

?

O

O

O

(b) O

30/04/10 16.47

54

CHAPTER 20

Aldehydes and Ketones

20.70 /…iÊVœ“«œÕ˜`ÊLiœÜʈÃÊLiˆiÛi`Ê̜ÊLiÊ>ÊÜ>ëʫ…iÀœ“œ˜i°Ê À>ÜÊ̅iʓ>œÀÊ«Àœ`ÕVÌÊvœÀ“i`Ê܅i˜Ê̅ˆÃÊVœ“«œÕ˜`ʈÃÊ …Þ`ÀœÞâi`ʈ˜Ê>µÕiœÕÃÊ>Vˆ`\Ê

20.75

*Àœ«œÃiÊ>˜ÊivwVˆi˜ÌÊÃޘ̅iÈÃÊvœÀÊi>V…ÊœvÊ̅iÊvœœÜˆ˜}Ê ÌÀ>˜ÃvœÀ“>̈œ˜Ã\ O

(a) O O O

Br

(b)

20.71

*Àœ«œÃiÊ>˜ÊivwVˆi˜ÌÊÃޘ̅iÈÃÊvœÀÊi>V…ÊœvÊ̅iÊvœœÜˆ˜}Ê ÌÀ>˜ÃvœÀ“>̈œ˜Ã\

O O

(c)

(a) MeO OMe

OH

Br

CN Br

(b)

(d) O NH

O

(c)

(e)

20.72

À>ÜÊ >Ê «>ÕÈLiÊ “iV…>˜ˆÃ“Ê vœÀÊ Ì…iÊ vœœÜˆ˜}Ê ÌÀ>˜ÃvœÀ“>̈œ˜\

Br H

O

O

N

NH2 NH2

(f)

[H2SO4] [–H2O]

O

N

N

20.73 7…i˜ÊVÞVœ…iÝ>˜œ˜iʈÃÊÌÀi>Ìi`Ê܈̅ÊÓ"]Ê>˜ÊiµÕˆˆLÀˆÕ“ʈÃÊiÃÌ>LˆÃ…i`ÊLiÌÜii˜ÊVÞVœ…iÝ>˜œ˜iÊ>˜`ʈÌÃʅÞ`À>Ìi°Ê/…ˆÃÊ iµÕˆˆLÀˆÕ“Ê}Ài>̏ÞÊv>ۜÀÃÊ̅iʎi̜˜i]Ê>˜`ʜ˜ÞÊÌÀ>ViÊ>“œÕ˜ÌÃÊ œvÊ̅iʅÞ`À>ÌiÊV>˜ÊLiÊ`iÌiVÌi`°Ê˜ÊVœ˜ÌÀ>ÃÌ]Ê܅i˜ÊVÞVœ«Àœ«>˜œ˜iʈÃÊÌÀi>Ìi`Ê܈̅ÊÓ"]Ê̅iÊÀiÃՏ̈˜}ʅÞ`À>ÌiÊ«Ài`œ“ˆ˜>ÌiÃÊ >ÌÊiµÕˆˆLÀˆÕ“°Ê-Õ}}iÃÌÊ>˜ÊiÝ«>˜>̈œ˜ÊvœÀÊ̅ˆÃÊVÕÀˆœÕÃʜLÃiÀÛ>̈œ˜°Ê

(g)

(h) O

O

20.74

œ˜Ãˆ`iÀÊ Ì…iÊ Ì…ÀiiÊ Vœ˜Ã̈ÌṎœ˜>Ê ˆÃœ“iÀÃʜvÊ`ˆœÝ>˜iÊ ­ {Hn"Ó®\Ê O

O

O

O O O

1,2-dioxane

1,3-dioxane

1,4-dioxane

"˜iʜvÊ̅iÃiÊVœ˜Ã̈ÌṎœ˜>ÊˆÃœ“iÀÃʈÃÊÃÌ>LiÊ՘`iÀÊL>ÈVÊVœ˜`ˆÌˆœ˜ÃÊ>ÃÊÜiÊ>Ãʓˆ`ÞÊ>Vˆ`ˆVÊVœ˜`ˆÌˆœ˜ÃÊ>˜`ʈÃÊ̅iÀivœÀiÊÕÃi`Ê >ÃÊ>ÊVœ““œ˜Ê܏Ûi˜Ì°Ê˜œÌ…iÀʈܓiÀʈÃʜ˜ÞÊÃÌ>LiÊ՘`iÀÊ L>ÈVÊVœ˜`ˆÌˆœ˜ÃÊLÕÌÊ՘`iÀ}œiÃʅÞ`ÀœÞÈÃÊ՘`iÀʓˆ`ÞÊ>Vˆ`ˆVÊ Vœ˜`ˆÌˆœ˜Ã°Ê/…iÊÀi“>ˆ˜ˆ˜}ʈܓiÀʈÃÊiÝÌÀi“iÞÊ՘ÃÌ>LiÊ>˜`Ê «œÌi˜Ìˆ>ÞÊiÝ«œÃˆÛi°Ê`i˜ÌˆvÞÊi>V…ʈÜ“iÀ]Ê>˜`ÊiÝ«>ˆ˜Ê̅iÊ «Àœ«iÀ̈iÃʜvÊi>V…ÊVœ“«œÕ˜`°

klein_c20_001-056v1.4.indd 54

30/04/10 16.47

55

Integrated Problems

INTEGRATED PROBLEMS 20.76

œ“«œÕ˜`Ê Ê …>ÃÊ “œiVՏ>ÀÊ vœÀ“Տ>Ê ÇH£{"Ê >˜`Ê Ài>VÌÃÊ܈̅ÊÜ`ˆÕ“ÊLœÀœ…Þ`Àˆ`iʈ˜Ê“i̅>˜œÊ̜ÊvœÀ“Ê>˜Ê>Vœ…œ°Ê /…iÊ £Ê ,Ê Ã«iVÌÀÕ“Ê œvÊ Vœ“«œÕ˜`Ê Ê i݅ˆLˆÌÃʜ˜ÞÊÌܜÊÈ}˜>Ã\Ê>Ê`œÕLiÌÊ­IÊrÊ£Ó®Ê>˜`Ê>ÊÃi«ÌiÌÊ­IÊrÊÓ®°Ê/Ài>̈˜}ÊVœ“«œÕ˜`Ê Ê ÜˆÌ…Ê £]Ӈi̅>˜i`ˆÌ…ˆœÊ ­- Ó Ó-®Ê vœœÜi`Ê LÞÊ ,>˜iÞÊ ˜ˆVŽiÊ}ˆÛiÃÊVœ“«œÕ˜`Ê °Ê ­>®Ê œÜʓ>˜ÞÊÈ}˜>ÃÊ܈Ê>««i>Àʈ˜Ê̅iÊ£Ê ,ÊëiVÌÀՓʜvÊ Vœ“«œÕ˜`Ê ¶ ­L®Ê œÜʓ>˜ÞÊÈ}˜>ÃÊ܈Ê>««i>Àʈ˜Ê̅iÊ£Î Ê ,ÊëiVÌÀՓʜvÊ Vœ“«œÕ˜`Ê ¶ ­V®Ê iÃVÀˆLiʅœÜÊޜÕÊVœÕ`ÊÕÃiÊ,ÊëiVÌÀœÃVœ«ÞÊ̜ÊÛiÀˆvÞÊ̅iÊ Vœ˜ÛiÀȜ˜ÊœvÊVœ“«œÕ˜`ÊÊ̜ÊVœ“«œÕ˜`Ê °

20.78 `i˜ÌˆvÞÊ Ì…iÊ ÃÌÀÕVÌÕÀiÃÊ œvÊ Vœ“«œÕ˜`ÃÊ Ê ÌœÊ Ê LiœÜ]Ê >˜`Ê Ì…i˜Ê ˆ`i˜ÌˆvÞÊ Ì…iÊ Ài>}i˜ÌÃÊ Ì…>ÌÊ V>˜Ê LiÊ ÕÃi`Ê ÌœÊ Vœ˜ÛiÀÌÊ VÞVœ…iÝi˜iʈ˜ÌœÊVœ“«œÕ˜`Ê Êˆ˜ÊÕÃÌʜ˜iÊÃÌi«° H3O+

H2CrO4

A

B +

[H ] NH2NH2 (–H2O)

D

20.79

KOH / H2O heat

C

`i˜ÌˆvÞÊ̅iÊÃÌÀÕVÌÕÀiÃʜvÊVœ“«œÕ˜`ÃÊÊÌœÊ ÊLiœÜ\ O

20.77

1)

1Ș}Ê Ì…iÊ ˆ˜vœÀ“>̈œ˜Ê «ÀœÛˆ`i`Ê LiœÜ]Ê `i`ÕViÊ Ì…iÊ ÃÌÀÕVÌÕÀiÃʜvÊVœ“«œÕ˜`ÃÊ]Ê ]Ê ]Ê>˜`Ê \Ê A (C10H12)

Br2 FeBr3

Mg

A

B

H 2) H2O

H

PCC

D 1) EtMgBr

1) O3 2) DMS

(C11H16O)

2) H2O

E

C (C9H10O)

HO

OH

[H+], −H2O

D

20.80 ˜Ê >`i…Þ`iÊ ÜˆÌ…Ê “œiVՏ>ÀÊ vœÀ“Տ>Ê {HÈ"Ê i݅ˆLˆÌÃÊ >˜Ê,ÊÈ}˜>Ê>ÌʣǣxÊV“•£° ­>®Ê *Àœ«œÃiÊÌܜʫœÃÈLiÊÃÌÀÕVÌÕÀiÃÊ̅>ÌÊ>ÀiÊVœ˜ÃˆÃÌi˜ÌÊÜˆÌ…Ê Ì…ˆÃʈ˜vœÀ“>̈œ˜° ­L®Ê iÃVÀˆLiʅœÜÊޜÕÊVœÕ`ÊÕÃiÊ£Î Ê ,ÊëiVÌÀœÃVœ«ÞÊÌœÊ `iÌiÀ“ˆ˜iÊ܅ˆV…ÊœvÊ̅iÊÌܜʫœÃÈLiÊÃÌÀÕVÌÕÀiÃʈÃÊVœÀÀiVÌ°Ê

N

B [H+], (CH3)2NH

AlCl3

C

(–H2O)

20.81

Ê Vœ“«œÕ˜`Ê ÜˆÌ…Ê “œiVՏ>ÀÊ vœÀ“Տ>Ê ™H£ä"Ê i݅ˆLˆÌÃÊ >Ê ÃÌÀœ˜}Ê Ãˆ}˜>Ê >ÌÊ £ÈnÇÊV“•£Êˆ˜ÊˆÌÃÊ,ÊëiVÌÀՓ°Ê/…iÊ £Ê>˜`Ê £Î Ê ,ÊëiVÌÀ>ÊvœÀÊ̅ˆÃÊVœ“«œÕ˜`Ê>ÀiÊÅœÜ˜Ê LiœÜ°Ê`i˜ÌˆvÞÊ̅iÊÃÌÀÕVÌÕÀiʜvÊ̅ˆÃÊVœ“«œÕ˜`°Ê 3

Proton NMR

2 2

9

8

3

7

6

5

4

3

2

1

Chemical Shift (ppm)

Carbon 13 NMR 128.3 132.5 199.9 200

180

127.7

31.3

7.9

136.7 160

140

120

100

80

60

40

20

0

Chemical Shift (ppm)

klein_c20_001-056v1.4.indd 55

30/04/10 16.47

56

CHAPTER 20

Aldehydes and Ketones

20.82 Ê Vœ“«œÕ˜`Ê ÜˆÌ…Ê “œiVՏ>ÀÊ vœÀ“Տ>Ê £ÎH£ä"Ê «Àœ`ÕViÃÊ >Ê ÃÌÀœ˜}Ê Ãˆ}˜>Ê >ÌÊ £ÈÈäÊV“•£Êˆ˜ÊˆÌÃÊ,ÊëiVÌÀՓ°Ê/…iÊ £Î Ê ,ÊëiVÌÀՓÊvœÀÊ̅ˆÃÊVœ“«œÕ˜`ʈÃÊŜܘÊLiœÜ°Ê `i˜ÌˆvÞÊ̅iÊÃÌÀÕVÌÕÀiʜvÊ̅ˆÃÊVœ“«œÕ˜`°Ê Carbon 13 NMR 130.0 128.3 132.4 137.5

196.7 200

190

180

170

160

150

140

130

120

110

100

Chemical Shift (ppm)

ʎi̜˜iÊ܈̅ʓœiVՏ>ÀÊvœÀ“Տ>Ê ™H£n"Êi݅ˆLˆÌÃʜ˜Þʜ˜iÊÈ}˜>Êˆ˜ÊˆÌÃÊ £Ê ,Ê Ã«iVÌÀՓ°Ê*ÀœÛˆ`iÊ>ÊÃÞÃÌi“>̈VÊ­1* ®Ê˜>“iÊvœÀÊ̅ˆÃÊVœ“«œÕ˜`°Ê

20.83

CHALLENGE PROBLEMS 20.84

À>ÜÊ>Ê«>ÕÈLiʓiV…>˜ˆÃ“ÊvœÀÊi>V…ÊœvÊ̅iÊvœœÜˆ˜}ÊÌÀ>˜ÃvœÀ“>̈œ˜Ã\ O

N

O

H3

H

H N

O+

H + H3C + CH3

HO

(a) O OCH3 O

(c)

H

H3O+

(b) O H

H

+

NH2NH2

[H2SO4]

O

H

O [H2SO4]

OH

(d)

N N

O

O

OCH3

OH

OCH3

O

O

[H2SO4]

(e) OH

OH

OCH3 [TsOH]

(f) HO

O

O

OH

20.85 1˜`iÀÊ>Vˆ`‡V>Ì>Þâi`ÊVœ˜`ˆÌˆœ˜Ã]ÊvœÀ“>`i…Þ`iÊ«œÞ“iÀˆâiÃÊ̜ʫÀœ`ÕViÊ>ʘՓLiÀÊ œvÊ Vœ“«œÕ˜`Ã]Ê ˆ˜VÕ`ˆ˜}Ê «>À>vœÀ“>`i…Þ`i°Ê À>ÜÊ >Ê «>ÕÈLiÊ “iV…>˜ˆÃ“Ê vœÀÊ Ì…ˆÃÊ ÌÀ>˜ÃvœÀ“>̈œ˜\

O

O [H3O+]

H

H

O

O

Paraformaldehyde

klein_c20_001-056v1.4.indd 56

30/04/10 16.47